Вы находитесь на странице: 1из 110

Course 311: Abstract Algebra

Academic year 2007-08


D. R. Wilkins
c David R. Wilkins 19972007
Copyright

Contents
1 Topics in Group Theory
1.1 Groups . . . . . . . . . . . . . . . . . . . .
1.2 Examples of Groups . . . . . . . . . . . .
1.3 Elementary Properties of Groups . . . . .
1.4 Subgroups . . . . . . . . . . . . . . . . . .
1.5 Cyclic Groups . . . . . . . . . . . . . . . .
1.6 Cosets and Lagranges Theorem . . . . . .
1.7 Normal Subgroups and Quotient Groups .
1.8 Homomorphisms . . . . . . . . . . . . . .
1.9 The Isomorphism Theorems . . . . . . . .
1.10 Group Actions, Orbits and Stabilizers . . .
1.11 Conjugacy . . . . . . . . . . . . . . . . . .
1.12 The Class Equation of a Finite Group . . .
1.13 Cauchys Theorem . . . . . . . . . . . . .
1.14 The Structure of p-Groups . . . . . . . . .
1.15 The Sylow Theorems . . . . . . . . . . . .
1.16 Some Applications of the Sylow Theorems
1.17 Simple Groups . . . . . . . . . . . . . . .
1.18 Solvable Groups . . . . . . . . . . . . . . .

.
.
.
.
.
.
.
.
.
.
.
.
.
.
.
.
.
.

.
.
.
.
.
.
.
.
.
.
.
.
.
.
.
.
.
.

.
.
.
.
.
.
.
.
.
.
.
.
.
.
.
.
.
.

.
.
.
.
.
.
.
.
.
.
.
.
.
.
.
.
.
.

.
.
.
.
.
.
.
.
.
.
.
.
.
.
.
.
.
.

.
.
.
.
.
.
.
.
.
.
.
.
.
.
.
.
.
.

.
.
.
.
.
.
.
.
.
.
.
.
.
.
.
.
.
.

.
.
.
.
.
.
.
.
.
.
.
.
.
.
.
.
.
.

.
.
.
.
.
.
.
.
.
.
.
.
.
.
.
.
.
.

.
.
.
.
.
.
.
.
.
.
.
.
.
.
.
.
.
.

.
.
.
.
.
.
.
.
.
.
.
.
.
.
.
.
.
.

1
1
2
3
4
5
6
7
10
12
13
14
14
15
16
17
18
21
27

Course 311: Abstract Algebra


Academic year 2007-08
Chapter 2: Rings and Polynomials
D. R. Wilkins
c David R. Wilkins 19972007
Copyright

Contents
2 Rings and Polynomials
2.1 Rings, Integral Domains and Fields .
2.2 Ideals . . . . . . . . . . . . . . . . .
2.3 Quotient Rings and Homomorphisms
2.4 The Characteristic of a Ring . . . . .
2.5 Polynomial Rings . . . . . . . . . . .
2.6 Gausss Lemma . . . . . . . . . . . .
2.7 Eisensteins Irreducibility Criterion .

.
.
.
.
.
.
.

.
.
.
.
.
.
.

.
.
.
.
.
.
.

.
.
.
.
.
.
.

.
.
.
.
.
.
.

.
.
.
.
.
.
.

.
.
.
.
.
.
.

.
.
.
.
.
.
.

.
.
.
.
.
.
.

.
.
.
.
.
.
.

.
.
.
.
.
.
.

.
.
.
.
.
.
.

.
.
.
.
.
.
.

.
.
.
.
.
.
.

30
30
32
33
35
35
38
39

Course 311: Abstract Algebra


Academic year 2007-08
D. R. Wilkins
c David R. Wilkins 19972007
Copyright

Contents
3 Introduction to Galois Theory
3.1 Field Extensions and the Tower Law . . . . . . . . .
3.2 Algebraic Field Extensions . . . . . . . . . . . . . . .
3.3 Algebraically Closed Fields . . . . . . . . . . . . . . .
3.4 Ruler and Compass Constructions . . . . . . . . . . .
3.5 Splitting Fields . . . . . . . . . . . . . . . . . . . . .
3.6 Normal Extensions . . . . . . . . . . . . . . . . . . .
3.7 Separability . . . . . . . . . . . . . . . . . . . . . . .
3.8 Finite Fields . . . . . . . . . . . . . . . . . . . . . . .
3.9 The Primitive Element Theorem . . . . . . . . . . . .
3.10 The Galois Group of a Field Extension . . . . . . . .
3.11 The Galois correspondence . . . . . . . . . . . . . . .
3.12 Quadratic Polynomials . . . . . . . . . . . . . . . . .
3.13 Cubic Polynomials . . . . . . . . . . . . . . . . . . .
3.14 Quartic Polynomials . . . . . . . . . . . . . . . . . .
3.15 The Galois group of the polynomial x4 2 . . . . . .
3.16 The Galois group of a polynomial . . . . . . . . . . .
3.17 Solvable polynomials and their Galois groups . . . . .
3.18 A quintic polynomial that is not solvable by radicals

.
.
.
.
.
.
.
.
.
.
.
.
.
.
.
.
.
.

.
.
.
.
.
.
.
.
.
.
.
.
.
.
.
.
.
.

.
.
.
.
.
.
.
.
.
.
.
.
.
.
.
.
.
.

.
.
.
.
.
.
.
.
.
.
.
.
.
.
.
.
.
.

.
.
.
.
.
.
.
.
.
.
.
.
.
.
.
.
.
.

41
41
42
45
45
50
53
54
56
59
60
62
64
64
66
68
70
71
75

Course 311: Abstract Algebra


Academic year 2007-08
D. R. Wilkins
c David R. Wilkins 19972007
Copyright

Contents
4 Commutative Algebra and Algebraic Geometry
4.1 Modules . . . . . . . . . . . . . . . . . . . . . . .
4.2 Noetherian Modules . . . . . . . . . . . . . . . .
4.3 Noetherian Rings and Hilberts Basis Theorem . .
4.4 Polynomial Rings in Several Variables . . . . . . .
4.5 Algebraic Sets and the Zariski Topology . . . . .
4.6 The Structure of Algebraic Sets . . . . . . . . . .
4.7 Maximal Ideals and Zorns Lemma . . . . . . . .
4.8 Prime Ideals . . . . . . . . . . . . . . . . . . . . .
4.9 Affine Varieties and Irreducibility . . . . . . . . .
4.10 Radical Ideals . . . . . . . . . . . . . . . . . . . .
4.11 Commutative Algebras of Finite Type . . . . . .
4.12 Zariskis Theorem . . . . . . . . . . . . . . . . . .
4.13 Hilberts Nullstellensatz . . . . . . . . . . . . . .

.
.
.
.
.
.
.
.
.
.
.
.
.

.
.
.
.
.
.
.
.
.
.
.
.
.

.
.
.
.
.
.
.
.
.
.
.
.
.

.
.
.
.
.
.
.
.
.
.
.
.
.

.
.
.
.
.
.
.
.
.
.
.
.
.

.
.
.
.
.
.
.
.
.
.
.
.
.

.
.
.
.
.
.
.
.
.
.
.
.
.

77
77
79
81
84
87
90
92
94
95
98
100
101
104

Topics in Group Theory

1.1

Groups

A binary operation on a set G associates to elements x and y of G a third


element x y of G. For example, addition and multiplication are binary
operations of the set of all integers.
Definition A group G consists of a set G together with a binary operation
for which the following properties are satisfied:
(x y) z = x (y z) for all elements x, y, and z of G (the Associative
Law );
there exists an element e of G (known as the identity element of G)
such that e x = x = x e, for all elements x of G;
for each element x of G there exists an element x0 of G (known as the
inverse of x) such that x x0 = e = x0 x (where e is the identity
element of G).
The order |G| of a finite group G is the number of elements of G.
A group G is Abelian (or commutative) if x y = y x for all elements x
and y of G.
One usually adopts multiplicative notation for groups, where the product
x y of two elements x and y of a group G is denoted by xy. The associative
property then requires that (xy)z = x(yz) for all elements x, y and z of G.
The identity element is often denoted by e (or by eG when it is necessary
to specify explicitly the group to which it belongs), and the inverse of an
element x of G is then denoted by x1 .
It is sometimes convenient or customary to use additive notation for certain groups. Here the group operation is denoted by +, the identity element
of the group is denoted by 0, the inverse of an element x of the group is denoted by x. By convention, additive notation is rarely used for non-Abelian
groups. When expressed in additive notation the axioms for a Abelian group
require that (x + y) + z = x + (y + z), x + y = y + x, x + 0 = 0 + x = x and
x + (x) = (x) + x = 0 for all elements x, y and z of the group.
We shall usually employ multiplicative notation when discussing general
properties of groups. Additive notation will be employed for certain groups
(such as the set of integers with the operation of addition) where this notation
is the natural one to use.

1.2

Examples of Groups

The sets of integers, rational numbers, real numbers and complex numbers
are Abelian groups, where the group operation is the operation of addition.
The sets of non-zero rational numbers, non-zero real numbers and nonzero complex numbers are also Abelian groups, where the group operation is
the operation of multiplication.
For each positive integer m the set Zm of congruence classes of integers
modulo m is a group, where the group operation is addition of congruence
classes.
For each positive integer m the set Zm of congruence classes modulo m of
integers coprime to m is a group, where the group operation is multiplication
of congruence classes.
In particular, for each prime number p the set Zp of congruence classes
modulo p of integers not divisible by p is a group, where the group operation
is multiplication of congruence classes.
For each positive integer n the set of all nonsingular n n matrices is a
group, where the group operation is matrix multiplication. These groups are
not Abelian when n 2.
Let E n denote n-dimensional Euclidean space, so that E 2 denotes the
Euclidean plane, and E 3 denotes three-dimensional Euclidean space. A geometrical figure may be represented as a subset S of E n . A symmetry of S is a
transformation T : E n E n of E n which sends straight lines to straight lines,
preserves all lengths and angles, and has the property that T (S) = S. The
collection of all symmetries of a geometrical figure is a group, the symmetry
group of S, the group operation being that of composition of transformations.
For any natural number n greater than 2, the the dihedral group D2n of
order 2n is defined to be the symmetry group of a regular n-sided polygon
in the Euclidean plane. It consists of rotations though an angle of 2j/n
about the centre of the polygon for j = 0, 1, 2, . . . , n 1, together with the
reflections in the n axes of symmetry of the polygon.
The symmetries of a rectangle that is not a square constitute a group of
order 4. This group consists of the identity transformation, reflection in the
axis of symmetry joining the midpoints of the two shorter sides, reflection
in the axis of symmetry joining the two longer sides, and rotation though
an angle of radians (180 ). If I denotes the identity transformation, A
and B denote the reflections in the two axes of symmetry, and C denotes
the rotation through radians then A2 = B 2 = C 2 = I, AB = BA = C,
AC = CA = B and BC = CB = A. This group is Abelian: it is often
referred to as the Klein 4-group (or, in German, Kleinsche Viergruppe).
The symmetries of a regular tetrahedron in 3-dimensional space constitute
2

a group. Any permutation of the vertices of the tetrahedron can be effected


by an appropriate symmetry of the tetrahedron. Moreover each symmetry is
completely determined by the permutation of the vertices which it induces.
Therefore the group of symmetries of a regular tetrahedron is of order 24,
since there are 24 permutations of a set with four elements. It turns out that
this group is non-Abelian.

1.3

Elementary Properties of Groups

In what follows, we describe basic properties of a group G, using multiplicative notation and denoting the identity element of the group by the letter e.
Lemma 1.1 A group G has exactly one identity element e satisfying ex =
x = xe for all x G.
Proof Suppose that f is an element of G with the property that f x = x for
all elements x of G. Then in particular f = f e = e. Similarly one can show
that e is the only element of G satisfying xe = x for all elements x of G.
Lemma 1.2 An element x of a group G has exactly one inverse x1 .
Proof We know from the axioms that the group G contains at least one
element x1 which satisfies xx1 = e and x1 x = e. If z is any element of
G which satisfies xz = e then z = ez = (x1 x)z = x1 (xz) = x1 e = x1 .
Similarly if w is any element of G which satisfies wx = e then w = x1 . In
particular we conclude that the inverse x1 of x is uniquely determined, as
required.
Lemma 1.3 Let x and y be elements of a group G. Then (xy)1 = y 1 x1 .
Proof It follows from the group axioms that
(xy)(y 1 x1 ) = x(y(y 1 x1 )) = x((yy 1 )x1 ) = x(ex1 ) = xx1 = e.
Similarly (y 1 x1 )(xy) = e, and thus y 1 x1 is the inverse of xy, as required.
Note in particular that (x1 )1 = x for all elements x of a group G, since
x has the properties that characterize the inverse of the inverse x1 of x.
Given an element x of a group G, we define xn for each positive integer n
by the requirement that x1 = x and xn = xn1 x for all n > 1. We also define
x0 = e, where e is the identity element of the group, and we define xn to be
the inverse of xn for all positive integers n.
3

Theorem 1.4 Let x be an element of a group G. Then xm+n = xm xn and


xmn = (xm )n for all integers m and n.
Proof The identity xm+n = xm xn clearly holds when m = 0 and when n = 0.
The identity xm+n = xm xn can be proved for all positive integers m and n by
induction on n. The identity when m and n are both negative then follows
from the identity xmn = xn xm on taking inverses. The result when m
and n have opposite signs can easily be deduced from that where m and n
both have the same sign.
The identity xmn = (xm )n follows immediately from the definitions when
n = 0, 1 or 1. The result when n is positive can be proved by induction on
n. The result when n is negative can then be obtained on taking inverses.
If additive notation is employed for an Abelian group then the notation
x is replaced by nx for all integers n and elements x of the group. The
analogue of Theorem 1.4 then states that (m + n)x = mx + nx and (mn)x =
m(n(x)) for all integers m and n.
The associative law may be generalized to products of four or more elements of a group.
n

Example Given four elements x1 , x2 , x3 and x4 of a group, the products


((x1 x2 )x3 )x4 ,

(x1 x2 )(x3 x4 ),

(x1 (x2 x3 ))x4 ,

x1 ((x2 x3 )x4 ),

x1 (x2 (x3 x4 ))

all have the same value. (Note that x1 x2 x3 x4 is by definition the value of the
first of these expressions.)
Two expressions, each specifying a finite product of elements of a group G,
determine the same element of G if the same elements of G occur in both
expressions, and in the same order. This result can be proved by induction
on the number of elements of G making up such a product.

1.4

Subgroups

Definition Let G be a group, and let H be a subset of G. We say that H


is a subgroup of G if the following conditions are satisfied:
the identity element of G is an element of H;
the product of any two elements of H is itself an element of H;
the inverse of any element of H is itself an element of H.
A subgroup H of G is said to be proper if H 6= G.
4

Lemma 1.5 Let x be an element of a group G. Then the set of all elements
of G that are of the form xn for some integer n is a subgroup of G.
Proof Let H = {xn : n Z}. Then the identity element belongs to H, since
it is equal to x0 . The product of two elements of H is itself an element of
H, since xm xn = xm+n for all integers m and n (see Theorem 1.4). Also the
inverse of an element of H is itself an element of H since (xn )1 = xn for
all integers n. Thus H is a subgroup of G, as required.
Definition Let x be an element of a group G. The order of x is the smallest
positive integer n for which xn = e. The subgroup generated by x is the
subgroup consisting of all elements of G that are of the form xn for some
integer n.
Lemma 1.6 Let H and K be subgroups of a group G. Then H K is also
a subgroup of G.
Proof The identity element of G belongs to H K since it belongs to the
subgroups H and K. If x and y are elements of H K then xy is an element
of H (since x and y are elements of H), and xy is an element of K, and
therefore xy is an element of H K. Also the inverse x1 of an element x of
H K belongs to H and to K and thus belongs to H K, as required.
More generally, the intersection of any collection of subgroups of a given
group is itself a subgroup of that group.

1.5

Cyclic Groups

Definition A group G is said to be cyclic, with generator x, if every element


of G is of the form xn for some integer n.
Example The group Z of integers under addition is a cyclic group, generated
by 1.
Example Let n be a positive integer. The set Zn of congruence classes of
integers modulo n is a cyclic group of order n with respect to the operation
of addition.
Example The group of all rotations of the plane about the origin through an
integer multiple of 2/n radians is a cyclic group of order n for all integers n.
This group is generated by an anticlockwise rotation through an angle of
2/n radians.
5

1.6

Cosets and Lagranges Theorem

Definition Let H be a subgroup of a group G. A left coset of H in G is a


subset of G that is of the form xH, where x G and
xH = {y G : y = xh for some h H}.
Similarly a right coset of H in G is a subset of G that is of the form Hx,
where x G and
Hx = {y G : y = hx for some h H}.
Note that a subgroup H of a group G is itself a left coset of H in G.
Lemma 1.7 Let H be a subgroup of a group G. Then the left cosets of H
in G have the following properties:
(i) x xH for all x G;
(ii) if x and y are elements of G, and if y = xa for some a H, then
xH = yH;
(iii) if x and y are elements of G, and if xH yH is non-empty then xH =
yH.
Proof Let x G. Then x = xe, where e is the identity element of G. But
e H. It follows that x xH. This proves (i).
Let x and y be elements of G, where y = xa for some a H. Then
yh = x(ah) and xh = y(a1 h) for all h H. Moreover ah H and a1 h H
for all h H, since H is a subgroup of G. It follows that yH xH and
xH yH, and hence xH = yH. This proves (ii).
Finally suppose that xH yH is non-empty for some elements x and y
of G. Let z be an element of xH yH. Then z = xa for some a H, and
z = yb for some b H. It follows from (ii) that zH = xH and zH = yH.
Therefore xH = yH. This proves (iii).
Lemma 1.8 Let H be a finite subgroup of a group G. Then each left coset
of H in G has the same number of elements as H.
Proof Let H = {h1 , h2 , . . . , hm }, where h1 , h2 , . . . , hm are distinct, and let x
be an element of G. Then the left coset xH consists of the elements xhj for
j = 1, 2, . . . , m. Suppose that j and k are integers between 1 and m for which
xhj = xhk . Then hj = x1 (xhj ) = x1 (xhk ) = hk , and thus j = k, since
h1 , h2 , . . . , hm are distinct. It follows that the elements xh1 , xh2 , . . . , xhm are
distinct. We conclude that the subgroup H and the left coset xH both have
m elements, as required.
6

Theorem 1.9 (Lagranges Theorem) Let G be a finite group, and let H be


a subgroup of G. Then the order of H divides the order of G.
Proof Each element of G belongs to at least one left coset of H in G, and
no element can belong to two distinct left cosets of H in G (see Lemma 1.7).
Therefore every element of G belongs to exactly one left coset of H. Moreover
each left coset of H contains |H| elements (Lemma 1.8). Therefore |G| =
n|H|, where n is the number of left cosets of H in G. The result follows.
Definition Let H be a subgroup of a group G. If the number of left cosets
of H in G is finite then the number of such cosets is referred to as the index
of H in G, denoted by [G: H].
The proof of Lagranges Theorem shows that the index [G: H] of a subgroup H of a finite group G is given by [G: H] = |G|/|H|.
Corollary 1.10 Let x be an element of a finite group G. Then the order of
x divides the order of G.
Proof Let H be the set of all elements of G that are of the form xn for some
integer n. Then H is a subgroup of G (see Lemma 1.5), and the order of
H is the order of x. But the order of H divides G by Lagranges Theorem
(Theorem 1.9). The result follows.
Corollary 1.11 Any finite group of prime order is cyclic.
Proof Let G be a group of prime order, and let x be some element of G
that is not the identity element. Then the order of x is greater than one and
divides the order of G. But then the order of x must be equal to the order
of G, since the latter is a prime number. Thus G is a cyclic group generated
by x, as required.

1.7

Normal Subgroups and Quotient Groups

Let A and B be subsets of a group G. The product AB of the sets A and B


is defined by
AB = {xy : x A and y B}.
We denote {x}A and A{x} by xA and Ax, for all elements x of G and
subsets A of G. The Associative Law for multiplication of elements of G
ensures that (AB)C = A(BC) for all subsets A, B and C of G. We can
therefore use the notation ABC to denote the products (AB)C and A(BC);
7

and we can use analogous notation to denote the product of four or more
subsets of G.
If A, B and C are subsets of a group G, and if A B then clearly
AC BC and CA CB.
Note that if H is a subgroup of the group G and if x is an element of G
then xH is the left coset of H in G that contains the element x. Similarly
Hx is the right coset of H in G that contains the element x.
If H is a subgroup of G then HH = H. Indeed HH H, since the
product of two elements of a subgroup H is itself an element of H. Also
H HH since h = eh for any element h of H, where e, the identity element
of G, belongs to H.
Definition A subgroup N of a group G is said to be a normal subgroup of
G if xnx1 N for all n N and x G.
The notation N / G signifies N is a normal subgroup of G.
Definition A non-trivial group G is said to be simple if the only normal
subgroups of G are the whole of G and the trivial subgroup {e} whose only
element is the identity element e of G.
Lemma 1.12 Every subgroup of an Abelian group is a normal subgroup.
Proof Let N be a subgroup of an Abelian group G. Then
xnx1 = (xn)x1 = (nx)x1 = n(xx1 ) = ne = n
for all n N and x G, where e is the identity element of G. The result
follows.
Example Let S3 be the group of permutations of the set {1, 2, 3}, and let
H be the subgroup of S3 consisting of the identity permutation and the
transposition (1 2). Then H is not normal in G, since (2 3)1 (1 2)(2 3) =
(2 3)(1 2)(2 3) = (1 3) and (1 3) does not belong to the subgroup H.
Proposition 1.13 A subgroup N of a group G is a normal subgroup of G if
and only if xN x1 = N for all elements x of G.
Proof Suppose that N is a normal subgroup of G. Let x be an element
of G. Then xN x1 N . (This follows directly from the definition of a
normal subgroup.) On replacing x by x1 we see also that x1 N x N , and
thus N = x(x1 N x)x1 xN x1 . Thus each of the sets N and xN x1 is
contained in the other, and therefore xN x1 = N .
Conversely if N is a subgroup of G with the property that xN x1 = N
for all x G, then it follows immediately from the definition of a normal
subgroup that N is a normal subgroup of G.
8

Corollary 1.14 A subgroup N of a group G is a normal subgroup of G if


and only if xN = N x for all elements x of G.
Proof Let N be a subgroup of G, and let x be an element of G. If xN x1 =
N then xN = (xN x1 )x = N x. Conversely if xN = N x then xN x1 =
N xx1 = N e = N , where e is the identity element of G. Thus xN = N x if
and only if xN x1 = N . It follows from Proposition 1.13 that a subgroup N
of G is normal if and only if xN = N x for all elements x of G, as required.
Let N be a normal subgroup of G. Corollary 1.14 shows that a subset of
G is a left coset of N in G if and only if it is a right coset of N in G. We
may therefore refer to the left and right cosets of a normal subgroup N as
cosets of N in G (since it is not in this case necessary to distinguish between
left and right cosets).
Lemma 1.15 Let N be a normal subgroup of a group G and let x and y be
elements of G. Then (xN )(yN ) = (xy)N .
Proof If N is a normal subgroup of G then N y = yN , and therefore
(xN )(yN ) = x(N y)N = x(yN )N = (xy)(N N ). But N N = N , since N
is a subgroup of G. Therefore (xN )(yN ) = (xy)N , as required.
Proposition 1.16 Let G be a group, and let N be a normal subgroup of
G. Then the set of all cosets of N in G is a group under the operation of
multiplication. The identity element of this group is N itself, and the inverse
of a coset xN is the coset x1 N for any element x of G.
Proof Let x, y and z be any elements of G. Then the product of the cosets
xN and yN is the coset (xy)N . The subgroup N is itself a coset of N in G,
since N = eN . Moreover
(xN )N = (xN )(eN ) = (xe)N = xN,
N (xN ) = (eN )(xN ) = (ex)N = xN,
(xN )(x1 N ) = (xx1 )N = N,
(x1 N )(xN ) = (x1 x)N = N.
for all elements x of G. Thus the group axioms are satisfied.
Definition Let N be a normal subgroup of a group G. The quotient group
G/N is defined to be the group of cosets of N in G under the operation of
multiplication.
9

Example Consider the dihedral group D8 of order 8, which we represent as


the group of symmetries of a square in the plane with corners at the points
whose Cartesian co-ordinates are (1, 1), (1, 1), (1, 1) and (1, 1). Then
D8 = {I, R, R2 , R3 , T1 , T2 , T3 , T4 },
where I denotes the identity transformation, R denotes an anticlockwise
rotation about the origin through a right angle, and T1 , T2 , T3 and T4 denote
the reflections in the lines y = 0, x = y, x = 0 and x = y respectively. Let
N = {I, R2 }. Then N is a subgroup of D8 . The left cosets of N in D8 are
N , A, B and C, where
A = {R, R3 },

B = {T1 , T3 },

C = {T2 , T4 }.

Moreover N , A, B and C are also the right cosets of N in D8 , and thus N is


a normal subgroup of D8 . On multiplying the cosets A, B and C with one
another we find that AB = BA = C, AC = CA = B and BC = CB = A.
The quotient group D8 /N consists of the set {N, A, B, C}, with the group
operation just described.

1.8

Homomorphisms

Definition A homomorphism : G K from a group G to a group K is a


function with the property that (g1 g2 ) = (g1 ) (g2 ) for all g1 , g2 G,
where denotes the group operation on G and on K.
Example Let q be an integer. The function from the group Z of integers to
itself that sends each integer n to qn is a homomorphism.
Example Let x be an element of a group G. The function that sends each
integer n to the element xn is a homomorphism from the group Z of integers
to G, since xm+n = xm xn for all integers m and n (Theorem 1.4).
Lemma 1.17 Let : G K be a homomorphism. Then (eG ) = eK , where
eG and eK denote the identity elements of the groups G and K. Also (x1 ) =
(x)1 for all elements x of G.
Proof Let z = (eG ). Then z 2 = (eG )(eG ) = (eG eG ) = (eG ) = z. The
result that (eG ) = eK now follows from the fact that an element z of K
satisfies z 2 = z if and only if z is the identity element of K.
Let x be an element of G. The element (x1 ) satisfies (x)(x1 ) =
(xx1 ) = (eG ) = eK , and similarly (x1 )(x) = eK . The uniqueness of
the inverse of (x) now ensures that (x1 ) = (x)1 .
10

An isomorphism : G K between groups G and K is a homomorphism that is also a bijection mapping G onto K. Two groups G and K are
isomorphic if there exists an isomorphism mapping G onto K.
Example Let D6 be the group of symmetries of an equilateral triangle in
the plane with vertices A, B and C, and let S3 be the group of permutations
of the set {A, B, C}. The function which sends a symmetry of the triangle
to the corresponding permutation of its vertices is an isomorphism between
the dihedral group D6 of order 6 and the symmetric group S3 .
Example Let R be the group of real numbers with the operation of addition,
and let R+ be the group of strictly positive real numbers with the operation
of multiplication. The function exp: R R+ that sends each real number x
to the positive real number ex is an isomorphism: it is both a homomorphism
of groups and a bijection. The inverse of this isomorphism is the function
log: R+ R that sends each strictly positive real number to its natural
logarithm.
Here is some further terminology regarding homomorphisms:
A monomorphism is an injective homomorphism.
An epimorphism is a surjective homomorphism.
An endomorphism is a homomorphism mapping a group into itself.
An automorphism is an isomorphism mapping a group onto itself.
Definition The kernel ker of the homomorphism : G K is the set of
all elements of G that are mapped by onto the identity element of K.
Example Let the group operation on the set {+1, 1} be multiplication,
and let : Z {+1, 1} be the homomorphism that sends each integer n
to (1)n . Then the kernel of the homomorphism is the subgroup of Z
consisting of all even numbers.
Lemma 1.18 Let G and K be groups, and let : G K be a homomorphism
from G to K. Then the kernel ker of is a normal subgroup of G.
Proof Let x and y be elements of ker . Then (x) = eK and (y) = eK ,
where eK denotes the identity element of K. But then (xy) = (x)(y) =
eK eK = eK , and thus xy belongs to ker . Also (x1 ) = (x)1 = e1
K = eK ,
and thus x1 belongs to ker . We conclude that ker is a subgroup of K.
Moreover ker is a normal subgroup of G, for if g G and x ker then
(gxg 1 ) = (g)(x)(g)1 = (g)(g 1 ) = eK .
11

If N is a normal subgroup of some group G then N is the kernel of the


quotient homomorphism : G G/N that sends g G to the coset gN . It
follows therefore that a subset of a group G is a normal subgroup of G if and
only if it is the kernel of some homomorphism.
Proposition 1.19 Let G and K be groups, let : G K be a homomorphism from G to K, and let N be a normal subgroup of G. Suppose that
N ker . Then the homomorphism : G K induces a homomorphism
G/N K sending gN G/N to (g). Moreover :
G/N K is injective
:
if and only if N = ker .
Proof Let x and y be elements of G. Now xN = yN if and only if x1 y N .
Also (x) = (y) if and only if x1 y ker . Thus if N ker then (x) =
(y) whenever xN = yN , and thus : G K induces a well-defined function
G/N K sending xN G/N to (x). This function is a homomorphism,
:

since ((xN
)(yN )) = (xyN
) = (xy) = (x)(y) = (xN
)(yN
).
Suppose now that N = ker . Then (x) = (y) if and only if xN = yN .
G/N K is injective. Conversely if :
G/N
Thus the homomorphism :
K is injective then N must be the kernel of , as required.
Corollary 1.20 Let G and K be groups, and let : G K be a homomorphism. Then (G)
= G/ ker .

1.9

The Isomorphism Theorems

Lemma 1.21 Let G be a group, let H be a subgroup of G, and let N be a


normal subgroup of G. Then the set HN is a subgroup of G, where HN =
{hn : h H and n N }.
Proof The set HN clearly contains the identity element of G. Let x and y
be elements of HN . We must show that xy and x1 belong to HN . Now
x = hu and y = kv for some elements h and k of H and for some elements u
and v of N . Then xy = (hk)(k 1 ukv). But k 1 uk N , since N is normal.
It follows that k 1 ukv N , since N is a subgroup and k 1 ukv is the product
of the elements k 1 uk and v of N . Also hk H. It follows that xy HN .
We must also show that x1 HN . Now x1 = u1 h1 = h1 (hu1 h1 ).
Also h1 H, since H is a subgroup of G, and hu1 h1 N , since N
is a normal subgroup of G. It follows that x1 HN , and thus HN is a
subgroup of G, as required.

12

Theorem 1.22 (First Isomorphism Theorem) Let G be a group, let H be a


subgroup of G, and let N be a normal subgroup of G. Then
HN H
.
=
N
N H
Proof Every element of HN/N is a coset of N that is of the form hN for
some h H. Thus if (h) = hN for all h H then : H HN/N is
a surjective homomorphism, and ker = N H. But (H)
= H/ ker

(Corollary 1.20). Therefore HN/N = H/(N H) as required.


Theorem 1.23 (Second Isomorphism Theorem) Let M and N be normal
subgroups of a group G, where M N . Then
G G/M
.
=
N
N/M
Proof There is a well-defined homomorphism : G/M G/N that sends
gM to gN for all g G. Moreover the homomorphism is surjective, and
ker = N/M . But (G/M )
= (G/M )/ ker . Therefore G/N is isomorphic
to (G/M ) / (N/M ), as required.

1.10

Group Actions, Orbits and Stabilizers

Definition A left action of a group G on a set X associates to each g G


and x X an element g.x of X in such a way that g.(h.x) = (gh).x and
1.x = x for all g, h G and x X, where 1 denotes the identity element of
G.
Given a left action of a group G on a set X, the orbit of an element x of
X is the subset {g.x : g G} of X, and the stabilizer of x is the subgroup
{g G : g.x = x} of G.
Lemma 1.24 Let G be a finite group which acts on a set X on the left.
Then the orbit of an element x of X contains [G: H] elements, where [G: H]
is the index of the stabilizer H of x in G.
Proof There is a well-defined function : G/H X defined on the set G/H
of left cosets of H in G which sends gH to g.x for all g G. Moreover this
function is injective, and its image is the orbit of x. The result follows.

13

1.11

Conjugacy

Definition Two elements h and k of a group G are said to be conjugate if


k = ghg 1 for some g G.
One can readily verify that the relation of conjugacy is reflexive, symmetric and transitive and is thus an equivalence relation on a group G. The
equivalence classes determined by this relation are referred to as the conjugacy classes of G. A group G is the disjoint union of its conjugacy classes.
Moreover the conjugacy class of the identity element of G contains no other
element of G.
A group G is Abelian if and only if all its conjugacy classes contain exactly
one element of the group G.
Definition Let G be a group. The centralizer C(h) of an element h of G is
the subgroup of G defined by C(h) = {g G : gh = hg}.
Lemma 1.25 Let G be a finite group, and let h G. Then the number of
elements in the conjugacy class of h is equal to the index [G: C(h)] of the
centralizer C(h) of h in G.
Proof There is a well-defined function f : G/C(h) G, defined on the set
G/C(h) of left cosets of C(h) in G, which sends the coset gC(h) to ghg 1 for
all g G. This function is injective, and its image is the conjugacy class of
h. The result follows.
Let H be a subgroup of a group G. One can easily verify that gHg 1 is
also a subgroup of G for all g G, where gHg 1 = {ghg 1 : h H}.
Definition Two subgroups H and K of a group G are said to be conjugate
if K = gHg 1 for some g G.
The relation of conjugacy is an equivalence relation on the collection of
subgroups of a given group G.

1.12

The Class Equation of a Finite Group

Definition The centre Z(G) of a group G is the subgroup of G defined by


Z(G) = {g G : gh = hg for all h G}.

14

One can verify that the centre of a group G is a normal subgroup of G.


Let G be a finite group, and let Z(G) be the centre of G. Then G \ Z(G)
is a disjoint union of conjugacy classes. Let r be the number of conjugacy
classes contained in G\Z(G), and let n1 , n2 , . . . , nr be the number of elements
in these conjugacy classes. Then ni > 1 for all i, since the centre Z(G) of
G is the subgroup of G consisting of those elements of G whose conjugacy
class contains just one element. Now the group G is the disjoint union of its
conjugacy classes, and therefore
|G| = |Z(G)| + n1 + n2 + + nr .
This equation is referred to as the class equation of the group G.
Definition Let g be an element of a group G. The centralizer C(g) of g is
the subgroup of G defined by C(g) = {h G : hg = gh}.
Proposition 1.26 Let G be a finite group, and let p be a prime number.
Suppose that pk divides the order of G for some positive integer k. Then
either pk divides the order of some proper subgroup of G, or else p divides
the order of the centre of G.
Proof Choose elements g1 , g2 , . . . , gr of G\Z(G), where Z(G) is the centre of
G, such that each conjugacy class included in G \ Z(G) contains exactly one
of these elements. Let ni be the number of elements in the conjugacy class
of gi and let C(gi ) be the centralizer of gi for each i. Then C(gi ) is a proper
subgroup of G, and |G| = ni |C(gi )|. Thus if pk divides |G| but does not divide
the order of any proper subgroup of G then p must divide ni for i = 1, 2, . . . , r.
Examination of the class equation |G| = |Z(G)| + n1 + n2 + + nr now
shows that p divides |Z(G)|, as required.

1.13

Cauchys Theorem

Theorem 1.27 (Cauchy) Let G be an finite group, and let p be a prime


number that divides the order of G. Then G contains an element of order p.
Proof We prove the result by induction on the order of G. Thus suppose
that every finite group whose order is divisible by p and less than |G| contains
an element of order p. If p divides the order of some proper subgroup of G
then that subgroup contains the required element of order p. If p does not
divide the order of any proper subgroup of G then Proposition 1.26 ensures
that p divides the order of the centre Z(G) of G, and thus Z(G) cannot be
a proper subgroup of G. But then G = Z(G) and the group G is Abelian.
15

Thus let G be an Abelian group whose order is divisible by p, and let


H be a proper subgroup of G that is not contained in any larger proper
subgroup. If |H| is divisible by p then the induction hypothesis ensures that
H contains the required element of order p, since |H| < |G|. Suppose then
that |H| is not divisible by p. Choose g G \ H, and let C be the cyclic
subgroup of G generated by g. Then HC = G, since HC 6= H and HC
is a subgroup of G containing H. It follows from the First Isomorphism
Theorem (Theorem 1.22) that G/H
= C/H C. Now p divides |G/H|,
since |G/H| = |G|/|H| and p divides |G| but not |H|. Therefore p divides
|C|. Thus if m = |C|/p then g m is the required element of order p. This
completes the proof of Cauchys Theorem.

1.14

The Structure of p-Groups

Definition Let p be a prime number. A p-group is a finite group whose


order is some power pk of p.
Lemma 1.28 Let p be a prime number, and let G be a p-group. Then there
exists a normal subgroup of G of order p that is contained in the centre of G.
Proof Let |G| = pk . Then pk divides the order of G but does not divide the
order of any proper subgroup of G. It follows from Proposition 1.26 that p
divides the order of the centre of G. It then follows from Cauchys Theorem
(Theorem 1.27) that the centre of G contains some element of order p. This
element generates a cyclic subgroup of order p, and this subgroup is normal
since its elements commute with every element of G.
Proposition 1.29 Let G be a p-group, where p is some prime number, and
let H be a proper subgroup of G. Then there exists some subgroup K of G
such that H / K and K/H is a cyclic group of order p.
Proof We prove the result by induction on the order of G. Thus suppose
that the result holds for all p-groups whose order is less than that of G. Let
Z be the centre of G. Then ZH is a well-defined subgroup of G, since Z is
a normal subgroup of G.
Suppose that ZH 6= H. Then H is a normal subgroup of ZH. The
quotient group ZH/H is a p-group, and contains a subgroup K1 of order p
(Lemma 1.28). Let K = {g ZH : gH K1 }. Then H / K and K/H
= K1 ,
and therefore K is the required subgroup of G.
Finally suppose that ZH = H. Then Z H. Let H1 = {hZ : h H}.
Then H1 is a subgroup of G/Z. But G/Z is a p-group, and |G/Z| < |G|,
since |Z| p (Lemma 1.28). The induction hypothesis ensures the existence
16

of a subgroup K1 of G/Z such that H1 / K1 and K1 /H1 is cyclic of order p.


Let K = {g G : gZ K1 }. Then H / K and K/H
= K1 /H1 . Thus K is
the required subgroup of G.
Repeated applications of Proposition 1.29 yield the following result.
Corollary 1.30 Let G be a finite group whose order is a power of some
prime number p. Then there exist subgroups G0 , G1 , . . . , Gn of G, where G0
is the trivial subgroup and Gn = G, such that Gi1 / Gi and Gi /Gi1 is a
cyclic group of order p for i = 1, 2, . . . , n.

1.15

The Sylow Theorems

Definition Let G be a finite group, and let p be a prime number dividing


the order |G| of G. A p-subgroup of G is a subgroup whose order is some
power of p. A Sylow p-subgroup of G is a subgroup whose order is pk , where
k is the largest natural number for which pk divides |G|.
Theorem 1.31 (First Sylow Theorem) Let G be a finite group, and let p be a
prime number dividing the order of G. Then G contains a Sylow p-subgroup.
Proof We prove the result by induction on the order of G. Thus suppose
that all groups whose order is less than that of G contain the required Sylow
p-subgroups. Let k be the largest positive integer for which pk divides |G|.
If pk divides the order of some proper subgroup H of G then the induction
hypothesis ensures that H contains the required Sylow p-subgroup of order
pk . If pk does not divide the order of any proper subgroup of G then p
divides the order of the centre Z(G) of G (Proposition 1.26). It follows from
Cauchys Theorem (Theorem 1.27) that Z(G) contains an element of order
p, and this element generates a normal subgroup N of G of order p. The
induction hypothesis then ensures that G/N has a Sylow p-subgroup L of
order pk1 , since |G/N | = |G|/p. Let K = {g G : gN L}. Then
|K| = p|L| = pk , and thus K is the required Sylow p-subgroup of G.
Theorem 1.32 (Second Sylow Theorem) Let G be a finite group, and let
p be a prime number dividing the order of G. Then all Sylow p-subgroups
of G are conjugate, and any p-subgroup of G is contained in some Sylow psubgroup of G. Moreover the number of Sylow p-subgroups in G divides the
order of G and is congruent to 1 modulo p.

17

Proof Let K be a Sylow p-subgroup of G, and let X be the set of left cosets
of K in G. Let H be a p-subgroup of G. Then H acts on X on the left,
where h(gK) = hgK for all h H and g G. Moreover h(gK) = gK if and
only if g 1 hg K. Thus an element gK of X is fixed by H if and only if
g 1 Hg K.
Let |G| = pk m, where k and m are positive integers and m is coprime to
p. Then |K| = pk . Now the number of left cosets of K in G is |G|/|K|. Thus
the set X has m elements. Now the number of elements in any orbit for the
action of H on X divides the order of H, since it is the index in H of the
stabilizer of some element of that orbit (Lemma 1.24). But then the number
of elements in each orbit must be some power of p, since H is a p-group.
Thus if an element of X is not fixed by H then the number of elements in its
orbit is divisible by p. But X is a disjoint union of orbits under the action
of H on X. Thus if m0 denotes the number of elements of X that are fixed
by H then m m0 is divisible by p.
Now m is not divisible by p. It follows that m0 6= 0, and m0 is not divisible
by p. Thus there exists at least one element g of G such that g 1 Hg K. But
then H is contained in the Sylow p-subgroup gKg 1 . Thus every p-subgroup
is contained in a Sylow p-subgroup of G, and this Sylow p-subgroup is a
conjugate of the given Sylow p-subgroup K. In particular any two Sylow
p-subgroups are conjugate.
It only remains to show that the number of Sylow p-subgroups in G
divides the order of |G| and is congruent to 1 modulo p. On applying the
above results with H = K, we see that g 1 Kg = K for some g G if and
only if gK is a fixed point for the action of K on X. But the number of
elements g of G for which gK is a fixed point is m0 |K|, where m0 is the
number of fixed points in X. It follows that the number of elements g of
G for which g 1 Kg = K is pk m0 . But every Sylow p-subgroup of G is of
the form g 1 Kg for some g G. It follows that the number n of Sylow
p-subgroups in G is given by n = |G|/pk m0 = m/m0 . In particular n divides
|G|. Now we have already shown that m m0 is divisible by p. It follows
that m0 is coprime to p, since m is coprime to p. Also m m0 is divisible
by m0 , since (m m0 )/m0 = n 1. Putting these results together, we see
that m m0 is divisible by m0 p, and therefore n 1 is divisible by p. Thus
n divides |G| and is congruent to 1 modulo p, as required.

1.16

Some Applications of the Sylow Theorems

Theorem 1.33 Let p and q be prime numbers, where p < q and q 6 1


(mod p). Then any group of order pq is cyclic.

18

Proof Let G be a group of order pq. It follows from the First Sylow Theorem
that G contains Sylow subgroups Np and Nq of orders p and q respectively.
Now the number np of Sylow p-subgroups divides pq and satisfies np 1
(mod p), by the Second Sylow Theorem. Clearly np cannot be divisible by p,
and therefore either np = 1 or np = q. But q 6 1 (mod p). It follows that
np = 1. Thus the group G has just one subgroup of order p.
Now, given any element g of G, the subgroups Np and gNp g 1 are of
order p. It follows that gNp g 1 = Np for all elements g of G. Thus Np is a
normal subgroup of G.
A similar argument shows that Nq is also a normal subgroup of G, since
p < q, and therefore p 6 1 (mod q).
Now Np Nq is a subgroup of both Np and Nq . It follows from Lagranges
Theorem that the order of Np Nq divides both of the prime numbers p and
q, and therefore |Np Nq | = 1 and Np Nq = {e}, where e is the identity
element of G.
Let x Np and y Nq . Then yx1 y 1 Np and xyx1 Nq , since
Np and Nq are normal subgroups of G. But then xyx1 y 1 Np Nq ,
since xyx1 y 1 = x(yx1 y 1 ) = (xyx1 )y 1 , and therefore xyx1 y 1 = e.
Thus xy = yx for all x Np and y Nq . It follows easily from this that
the function : Np Nq G which sends (x, y) Np Nq to xy is a
homomorphism. This homomorphism is injective, for if xy = e for some
x Np and y Nq , then x = y 1 , and hence x Np Nq , from which it
follows that x = e and y = e. But any injective homomorphism between two
finite groups of the same order is necessarily an isomorphism. Therefore the
function : Np Nq G is an isomorphism, and thus G
= Np Nq .
Now any group whose order is prime number must be cyclic. Therefore
the groups Np and Nq are cyclic. Let x be an element of Np that generates
Np , and let y be an element of Nq that generates Nq . Then (x, y)n = (xn , y n )
for all integers n. It follows from this that the order of (x, y) cannot be equal
to 1, p or q, and must therefore be equal to pq. Thus Np Nq is a cyclic
group generated by (x, y), and therefore G is a cyclic group, generated by
xy, as required.
Example Any finite group whose order is 15, 33, 35, 51, 65, 69, 85, 87, 91
or 95 is cyclic.
Theorem 1.34 Let G be a group of order 2p where p is a prime number
greater than 2. Then either the group G is cyclic, or else the group G is isomorphic to the dihedral group D2p of symmetries of a regular p-sided polygon
in the plane.

19

Proof It follows from the First Sylow Theorem, or from Cauchys Theorem, that the group G contains elements x and y whose orders are 2 and p
respectively. The subgroup N generated by y is then a Sylow p-subgroup
of G. Now it follows from the Second Sylow Theorem that the number of
Sylow p-subgroups of G divides 2p and is congruent to 1 modulo p. There
can therefore be only one such Sylow p-subgroup, since 2, p and 2p are not
congruent to 1 modulo p. Now if g is any element of G then gN g 1 is a
Sylow p-subgroup of G, and therefore gN g 1 = N . We deduce that N is a
normal subgroup of G, of order p.
Now consider the element xyx1 of G. This must be an element of the
normal subgroup N of G generated by y. Therefore xyx1 = y k for some
integer k. Moreover k is not divisible by p, since xyx1 is not the identity
element e of G. Then
2

y k = (y k )k = (xyx1 )k = xy k x1 = x(xyx1 )x1 = x2 yx2 .


But x2 = x2 = e, since x is an element of G of order 2. It follows that
2
2
y k = y, and thus y k 1 = e. But then p divides k 2 1, since y is an
element of order p. Moreover k 2 1 = (k 1)(k + 1). It follows that either p
divides k 1, in which case xyx1 = y, or else p divides k + 1, in which case
xyx1 = y 1 .
In the case when xyx1 = y we see that xy = yx, and one can then
readily verify that the group G is a cyclic group of order 2p generated by xy.
In the case when xyx1 = y 1 the group G is isomorphic to the dihedral
group D2p of order 2p. In this case the elements x and y generate G (since
they generate a subgroup of G whose order divides 2p but is greater than p,
and must therefore be equal to 2p). Under the isomorphism with the dihedral
group D2p the element x corresponds to a reflection in one of the axes of
symmetry of the regular p-sided polygon, and the element y corresponds to a
rotation of that polygon about its centre through an angle of 2/p radians.
Theorem 1.35 Let p and q be prime numbers with p < q, and let d be the
smallest positive integer for which pd 1 (mod q). If G is a group of order
pk q, where 0 < k < d then G contains a normal subgroup of order q. If G is
a group of order pd q then either G contains a normal subgroup of order q or
else G contains a normal subgroup of order pd .
Proof It follows from the First Sylow Theorem (or directly from Cauchys
Theorem) that the group G contains at least one Sylow q-subgroup K, and
this is of order q. If |G| = pk q then the number nq of such Sylow q-subgroups
divides pk q and satisfies nq 1 (mod q), by the Second Sylow Theorem.
20

It follows that nq is coprime to q, and therefore nq = pj for some integer j


satisfying 0 j k.
If k < d then none of the integers p, p2 , . . . , pk are congruent to 1 modulo
q, and therefore j = 0 and nq = 1. In this case there is just one Sylow
q-subgroup K, and this is a normal subgroup. (Given any element g of G,
the subgroup gKg 1 is a Sylow q-subgroup, and therefore gKg 1 = K.)
If k = d then none of the integers pj with 0 < j < d are congruent to 1
modulo q, and therefore either nq = 1 or nq = pd . If nq = 1 then there is just
one Sylow q-subgroup K, and this is a normal subgroup.
If nq > 1 then nq = pd , and thus there are pd Sylow q-subgroups, and
these are of order q. Now if Ki and Kj are two distinct subgroups of order q
then Ki Kj is a proper subgroup of both Ki and Kj , and its order is a
proper divisor of the order q of Ki and Kj , by Lagranges Theorem. But q
is a prime number. It follows that Ki Kj = {e}, where e is the identity
element of G. We deduce from this that no element of G of order q can belong
to more than one subgroup of order q. But each subgroup of G of order q
contains q 1 elements of order q (namely all elements of that subgroup with
the exception of the identity element). It follows that the group G contains
pd (q 1) elements of order q. Now |G| = pd q. It follows that G contains
exactly pd elements that are not of order q. But it follows from the First
Sylow Theorem that G contains at least one Sylow p-subgroup H, and this is
of order pd . This subgroup must therefore contain all the elements of G that
are not of order q. It follows that the group G cannot contain more than
one such Sylow p-subgroup. This Sylow p-subgroup H is therefore a normal
subgroup of G of order pd , as required.

1.17

Simple Groups

Definition A non-trivial group G is said to be simple if the only normal


subgroups of G are the whole of G and the trivial subgroup {e} whose only
element is the identity element e of G.
Lemma 1.36 Any non-trivial Abelian simple group is a cyclic group whose
order is a prime number.
Proof Let G be a non-trivial Abelian simple group, and let x be an element
of G that is not equal to the identity element e of G. All subgroups of an
Abelian group are normal subgroups. Therefore the subgroup of G generated
by x is a normal subgroup of G, and must therefore be the whole of G.
Therefore G is a cyclic group, generated by the element x. Moreover all
elements of G other than the identity element are generators of G, and are
21

therefore of order p, where p = |G|. Let d be a divisor of p. Then xd is an


element of order p/d, since p/d is the smallest positive integer k for which
xdk = e. It follows that either d = 1 or d = p (since the group G contains no
element whose order is greater than 1 but less than p). It follows that the
order p of G is a prime number, as required.
Using the Sylow Theorems and related results, we can prove that any
finite simple group whose order is less than 60 is a cyclic group of prime
order.
Now the prime numbers less than 60 are the following: 2, 3, 5, 7, 11, 13,
17, 19, 23, 29, 31, 37, 41, 43, 47, 53 and 59. All groups of these orders are
simple groups, and are cyclic groups.
If p is a prime number greater than 2 then any group of order 2p is either
a cyclic group or else is isomorphic to the dihedral group D2p of order 2p
(Theorem 1.34). In either case such a group contains a normal subgroup of
order p, and therefore not a simple group. In particular, there are no simple
groups of orders 6, 10, 14, 22, 26, 34, 38, 46 or 58.
If G is a group of order pk for some prime number p and for some integer k
satisfying k 2, then G contains a normal subgroup of order p (Lemma 1.28).
It follows that such a group is not simple. In particular, there are no simple
groups of orders 4, 8, 16, 32, 9, 27, 25 and 49.
Let G be a group of order pq, where p and q are prime numbers and
p < q. Any Sylow q-subgroup of G is of order q, and the number of such
Sylow q-subgroups must divide pq and be congruent to 1 modulo q. Now
p cannot be congruent to 1 modulo q, since 1 < p < q. Therefore G has
just one Sylow q-subgroup, and this is a normal subgroup of G of order q.
It follows that such a group is not a simple group. In particular there are
no simple groups of orders 15, 21, 33, 35, 39, 51, 55 or 57. (In particular it
follows from Theorem 1.33 that any group whose order is 15, 33, 35, or 51 is
a cyclic group.)
It only remains to verify that there are no simple groups of orders 12, 18,
20, 24, 28, 30, 36, 40, 42, 44, 45, 48, 50, 52, 54 or 56.
We can deal with many of these on applying Theorem 1.35. On applying
this theorem with p = 2, q = 3 and d = 2, we see that there are no simple
groups of orders 6 or 12. On applying the theorem with p = 2, q = 5 and
d = 4, we see that there are no simple groups of orders 10, 20, 40 or 80.
On applying the theorem with p = 2, q = 7 and d = 3, we see that there
are no simple groups of orders 14, 28 or 56. On applying the theorem with
p = 2, q = 11 we see that there are no simple groups of orders 22, 44 etc.,
on applying the theorem with p = 2, q = 13 we see that there are no simple
groups of orders 26, 52 etc., and on applying the theorem with p = 3 and
22

q = 5, we see that there are no simple groups of orders 15, 45 etc.


It now remains to verify that there are no simple groups of orders 18, 24,
30, 36, 42, 48, 50 or 54.
Using the Second Sylow Theorem, we see that any group of order 18 has
just one Sylow 3-subgroup. This Sylow 3-subgroup is then a normal group
of order 9, and therefore no group of order 18 is simple. Similarly a group of
order 50 has just one Sylow 5-subgroup, which is then a normal subgroup of
order 25, and therefore no group of order 50 is simple. Also a group of order
54 has just one Sylow 3-subgroup, which is then a normal subgroup of order
27, and therefore no group of order 54 is simple.
On applying the Second Sylow Theorem, we see the number of Sylow
7-subgroups of any group of order 42 must divide 42 and be congruent to 1
modulo 7. This number must then be coprime to 7 and therefore divide 6,
since 42 = 7 6. But no divisor of 6 greater than 1 is coprime to 1 modulo
7. It follows that any group of order 42 has just one Sylow 7-subgroup, and
this subgroup is therefore a normal subgroup of order 7. Thus no group of
order 42 is simple.
On applying the Second Sylow Theorem, we see that if a group of order 30
has more than one subgroup of order 3 then it must have 10 such subgroups,
and must therefore have 20 elements of order 3 (since each subgroup of order 3 contains two elements of order 3, and the intersection of two distinct
subgroups of order 3 must be the trivial subgroup). Similarly if a group of
order 30 has more than one subgroup of order 5 then it must have 6 such
subgroups, and must therefore have 24 elements of order 5. Obviously such
a group cannot have both 20 elements of order 3 and 24 elements of order 5.
Therefore it either has a single subgroup of order 3 or a single subgroup of
order 5. This subgroup is normal. Therefore no group of order 30 is simple.
In order to show that there are no simple groups of order less than 60,
apart from the cyclic groups whose order is prime, it only remains to verify
that there are no simple groups of orders 24, 36 and 48. In order to deal with
these remaining cases, we need to make use of the following result.
Lemma 1.37 Let H and K be subgroups of a finite group G. Then
|H K|

|H| |K|
.
|G|

Proof Let : H K G be the function with (h, k) = hk for all h


H and k K. (This function is not in general a homomorphism.) Let
(h1 , k1 ) and (h2 , k2 ) be elements of H K. Then h1 k1 = h2 k2 if and only
1
1
1
if h1
and
2 h1 = k2 k1 , in which case h2 h1 H K. But then h2 = h1 x
k2 = xk1 for some element x of H K. Thus (h1 , k1 ) = (h2 , k2 ) if and
23

only if (h2 , k2 ) = (h1 x1 , xk1 ) for some element x of H K. It follows that


each element of the range (H K) of the function is the image of exactly
|H| |K|
|H K| elements of H K. It follows from this that (H K) has
|H K|
elements. But (H K) is a subset of G. Therefore
|H| |K|
|G|.
|H K|
The required inequality now follows directly.
Let G be a finite group, and let H be a subgroup of index 2 in G (i.e., a
subgroup with half as many elements as G). Then H is a normal subgroup
of G. Indeed the subsets H and G \ H of G are the left cosets and are also
the right cosets of H in G, and therefore the left cosets of H in G coincide
with the right cosets.
Example We now show that there are no simple groups of order 24. Let G
be a group of order 24. Then G contains a Sylow 2-subgroup H of order 8. If
this is the only Sylow 2-subgroup, then it is a normal subgroup, and therefore
the group G is not simple. Otherwise the group G contains at least two
distinct subgroups H and K of order 8. It then follows from Lemma 1.37
that |H K| 38 . But |H K| divides 8, by Lagranges Theorem, since
H K is a subgroup of H and of K. Therefore |H K| = 4. It follows
that H K is a subgroup of index 2 in H and K, and is therefore a normal
subgroup of both H and K. Let
J = {g G : g(H K)g 1 = H K}.
Then J is a subgroup of G, and H K is a normal subgroup of J. Moreover
H and K are subgroups of J, and therefore |J| is divisible by 8, by Lagranges
Theorem. But J is a subgroup of G, and hence |J| divides 24. Also |J| > 8,
since H (and K) are proper subgroups of J. It follows that |J| = 24, and
therefore J = G. But then H K is a normal subgroup of G of order 4, and
therefore G is not simple.
An analogous argument shows that there are no simple groups of order 48:
a group G of order 48 contains either a single Sylow 2-subgroup of order 16,
which is then a normal subgroup of G, or else it contains a normal subgroup
of order 8 which is the intersection of two distinct Sylow 2-subgroups of G.
The following result will be needed in order to show that there are no
simple groups of order 36. (It may be obtained as an immediate corollary of
Proposition 1.29.)
24

Lemma 1.38 Let G be a group of order p2 where p is a prime number, and


let H be a subgroup of G of order p. Then H is a normal subgroup of G.
Proof Let J = {g G : gHg 1 = H}. Then J is a subgroup of G and H is
a normal subgroup of J. We shall show that J = G.
Now the centre Z(G) of G is contained in J. Moreover it follows from
Lemma 1.28 that |Z(G)| is divisible by p. Were it the case that |J| = p then
J = H = Z(G). But then J would consist of all elements of G for which
gZ(G)g 1 = Z(G), and thus would be the whole of G, which is impossible.
It follows that |J| = p2 (since |J| > p and |J| divides p2 ). But then J = G,
and hence H is a normal subgroup of G, as required.
Example We now show that there are no simple groups of order 36. Let G
be a group of order 36. Then G contains a Sylow 3-subgroup H of order 9. If
this is the only Sylow 3-subgroup, then it is a normal subgroup, and therefore
the group G is not simple. Otherwise the group G contains at least two
distinct subgroups H and K of order 9. It then follows from Lemma 1.37
that |H K| 94 . But |H K| divides 9, by Lagranges Theorem, since
H K is a subgroup of H and of K. Therefore |H K| = 3. On applying
Lemma 1.38 we see that H K is a normal subgroup of H and of K.
Let
J = {g G : g(H K)g 1 = H K}.
Then J is a subgroup of G, and H K is a normal subgroup of J. Moreover
H and K are subgroups of J, and therefore |J| is divisible by 9, by Lagranges
Theorem. But J is a subgroup of G, and hence |J| divides 36. Also |J| > 9,
since H (and K) are proper groups of J. It follows that either |J| = 18 or
36. If |J| = 36 then J = G and H K is a normal subgroup of G of order 3.
If |J| = 18 then J is a subgroup of G of index 2, and is therefore a normal
subgroup of order 18. We conclude that any group of order 36 contains at
least one non-trivial normal subgroup. Therefore there are no simple groups
of order 36.
We have now shown that there are indeed no simple groups of order less
than 60, other than the cyclic groups of prime order.
There is a simple group of order 60 which is simple but is not cyclic. This
group is the alternating group A5 , consisting of all even permutations of a
set with five elements.
Lemma 1.39 The alternating group A5 is simple.

25

Proof We regard A5 as the group even permutations of the set {1, 2, 3, 4, 5}.
There are 60 such permutations: the identity permutation, twenty 3-cycles,
twenty-four 5-cycles, and fifteen permutations that are products of two disjoint transpositions. (Such a product of disjoint transpositions is a permutation (a1 a2 )(a3 a4 ) that interchanges a1 with a2 and a3 with a4 for some
distinct elements a1 , a2 , a3 and a4 of the set {1, 2, 3, 4, 5}.)
Now each 3-cycle in A5 generates a Sylow 3-subgroup of order 3, and these
subgroups are all conjugate to one another by the Second Sylow Theorem.
It follows that any normal subgroup of A5 that contains at least one 3-cycle
must contain all twenty 3-cycles, and thus its order must therefore be at
least 21 (since it must also contain the identity element). Similarly each
5-cycle in A5 generates a Sylow 5-subgroup of order 5, and these subgroups
are all conjugate to one another. Therefore any normal subgroup of A5 that
contains at least one 5-cycle must contain all twenty four 5-cycles, and thus
its order must be at least 25.
Now if A5 were to contain a subgroup of order 30, this subgroup would be
the kernel of a non-constant homomorphism : A5 {1, 1} from A5 to the
multiplicative group consisting of the numbers 1 and 1. But any 3-cycle
or 5-cycle would have to belong to the kernel of this homomorphism, and
therefore this kernel would contain at least 45 elements, which is impossible.
We conclude that A5 cannot contain any subgroup of order 30. It follows
from Lagranges Theorem that any normal subgroup of A5 that contains at
least one 3-cycle or 5-cycle must be the whole of A5 .
The group A5 contains 5 Sylow 2-subgroups, which are of order 4. One
of these consists of the identity permutation, together with the three permutations (1 2)(3 4), (1 3)(2 4) and (1 4)(2 3). (Each of these permutations fixes
the element 5.) There are four other such Sylow 2-subgroups, and all of the
Sylow 2-subgroups are conjugate to one another. It follows that A5 does not
contain any normal subgroup of order 4. Moreover A5 cannot contain any
normal subgroup of order 2, since any element of order 2 belongs to one of
the five Sylow 2-subgroups of order 4, and is therefore conjugate to elements
of order 2 in the other Sylow 2-subgroups.
Now any subgroup of A5 whose order is divisible by 3 must contain a
3-cycle by Cauchys Theorem. (Theorem 1.27.) Similarly any subgroup of
A5 whose order is divisible by 5 must contain a 5-cycle. It follows that the
order of any proper normal subgroup of A5 cannot be divisible by 3 or 5.
But this order must divide 60. Therefore the order of any proper normal
subgroup of A5 must be at most 4. But we have seen that A5 cannot contain
any normal subgroup of order 4 or 2. Therefore any proper normal subgroup
of A5 is trivial, and therefore A5 is simple.

26

1.18

Solvable Groups

The concept of a solvable group was introduced into mathematics by Evariste


Galois, in order to state and prove his fundamental general theorems concerning the solvability of polynomial equations. We now investigate the basic
properties of such solvable groups.
Definition A group G is said to be solvable (or soluble) if there exists a finite
sequence G0 , G1 , . . . , Gn of subgroups of G, where G0 = {1} and Gn = G,
such that Gi1 is normal in Gi and Gi /Gi1 is Abelian for i = 1, 2, . . . , n.
Example The symmetric group 4 is solvable. Indeed let V4 be the Kleinsche Viergruppe consisting of the identity permutation and the permutations (12)(34), (13)(24) and (14)(23), and let A4 be the alternating group
consisting of all even permutations of {1, 2, 3, 4}. Then {} / V4 / A4 / 4 , V4
is Abelian, A4 /V4 is cyclic of order 3, and 4 /A4 is cyclic of order 2.
In order to prove certain basic results concerning solvable groups, we
need to make use of the Isomorphism Theorems for groups, which may be
described as follows.
Lemma 1.40 Let G be a group, let H1 and H2 be subgroups of G, where
H1 / H2 , and let J1 = H1 N , J2 = H2 N , K1 = H1 N/N and K2 =
H2 N/N , where N is some normal subgroup of G. Then J1 / J2 and K1 / K2 .
Moreover there exists a normal subgroup of H2 /H1 isomorphic to J2 /J1 , and
the quotient of H2 /H1 by this normal subgroup is isomorphic to K2 /K1 .
Proof It is a straightforward exercise to verify that J1 / J2 and K1 / K2 . Let
: H2 K2 be the surjective homomorphism sending h H2 to the coset hN .
Now induces a well-defined surjective homomorphism : H2 /H1 K2 /K1 ,
since (H1 ) K1 . Also 1 (K1 ) = H2 (H1 N ). But H2 (H1 N ) = H1 (H2
N ), for if a H1 , b N and ab H2 then b H2 N . Therefore
ker = 1 (K1 )/H1 = H1 (H2 N )/H1
= H2 N/H1 N = J2 /J1
by the First Isomorphism Theorem (Theorem 1.22). Moreover the quotient
of H2 /H1 by the normal subgroup ker is isomorphic to the image K2 /K1
of . Thus ker is the required normal subgroup of H2 /H1 .
Proposition 1.41 Let G be a group, and let H be a subgroup of G. Then
(i) if G is solvable then any subgroup H of G is solvable;
(ii) if G is solvable then G/N is solvable for any normal subgroup N of G;
27

(iii) if N is a normal subgroup of G and if both N and G/N are solvable


then G is solvable.
Proof Suppose that G is solvable. Let G0 , G1 , . . . , Gm be a finite sequence
of subgroups of G, where G0 = {1}, Gn = G, and Gi1 / Gi and Gi /Gi1 is
Abelian for i = 1, 2, . . . , m.
We first show that the subgroup H is solvable. Let Hi = H Gi for
i = 0, 1, . . . , m. Then H0 = {1} and Hm = H. If u Hi and v Hi1 then
uvu1 H, since H is a subgroup of G. Also uvu1 Gi1 , since u Gi1 ,
v Gi and Gi1 is normal in Gi . Therefore uvu1 Hi1 . Thus Hi1 is a
normal subgroup of Hi for i = 1, 2, . . . , m. Moreover
Hi
Gi H
Gi1 (Gi H)

=
=
Hi1
Gi1 (Gi H)
Gi1
by the First Isomorphism Theorem (Theorem 1.22), and thus Hi /Hi1 is
isomorphic to a subgroup of the Abelian group Gi /Gi1 . It follows that
Hi /Hi1 must itself be an Abelian group. We conclude therefore that the
subgroup H of G is solvable.
Now let N be a normal subgroup of G, and let Ki = Gi N/N for all i.
Then K0 is the trivial subgroup of G/N and Km = G/N . It follows from
Lemma 1.40 that Ki1 / Ki and Ki /Ki1 is isomorphic to the quotient of
Gi /Gi1 by some normal subgroup. But a quotient of any Abelian group
must itself be Abelian. Thus each quotient group Ki /Ki1 is Abelian, and
thus G/N is solvable.
Finally suppose that G is a group, N is a normal subgroup of G and
both N and G/N are solvable. We must prove that G is solvable. Now the
solvability of N ensures the existence of a finite sequence G0 , G1 , . . . , Gm of
subgroups of N , where G0 = {1}, Gm = N , and Gi1 / Gi and Gi /Gi1 is
Abelian for i = 1, 2, . . . , m. Also the solvability of G/N ensures the existence
of a finite sequence K0 , K1 , . . . , Kn of subgroups of G/N , where K0 = N/N ,
Kn = G/N , and Ki1 / Ki and Ki /Ki1 is Abelian for i = 1, 2, . . . , n.
Let Gm+i be the preimage of Ki under the the quotient homomorphism
: G G/N , for i = 1, 2, . . . , n. The Second Isomorphism Theorem (Theorem 1.23) ensures that Gm+i /Gm+i1
= Ki /Ki1 for all i > 0. Therefore
G0 , G1 , . . . , Gm+n is a finite sequence of subgroups of G, where G0 = {1},
Gn = G, and Gi1 / Gi and Gi /Gi1 is Abelian for i = 1, 2, . . . , m + n. Thus
the group G is solvable, as required.
Example The alternating group A5 is simple. It follows that A5 is not
solvable, since the definition of solvable groups ensures that that any simple
solvable group is cyclic, and A5 is not cyclic. Now if n 5 the symmetric
28

group n of all permutations of a set of n elements contains a subgroup


isomorphic to A5 . (Take as this subgroup the set of all even permutations of
five of the elements permuted by the elements of n .) Moreover any subgroup
of a solvable group is solvable (Proposition 1.41.) It follows therefore that
the symmetric group n is not solvable when n 5.

29

Rings and Polynomials

2.1

Rings, Integral Domains and Fields

Definition A ring consists of a set R on which are defined operations of


addition and multiplication satisfying the following axioms:
x+y = y+x for all elements x and y of R (i.e., addition is commutative);
(x + y) + z = x + (y + z) for all elements x, y and z of R (i.e., addition
is associative);
there exists an an element 0 of R (known as the zero element) with the
property that x + 0 = x for all elements x of R;
given any element x of R, there exists an element x of R with the
property that x + (x) = 0;
x(yz) = (xy)z for all elements x, y and z of R (i.e., multiplication is
associative);
x(y + z) = xy + xz and (x + y)z = xz + yz for all elements x, y and z
of R (the Distributive Law ).
Lemma 2.1 Let R be a ring. Then x0 = 0 and 0x = 0 for all elements x of
R.
Proof The zero element 0 of R satisfies 0 + 0 = 0. Using the Distributive
Law, we deduce that x0 + x0 = x(0 + 0) = x0 and 0x + 0x = (0 + 0)x = 0x.
Thus if we add (x0) to both sides of the identity x0 + x0 = x0 we see that
x0 = 0. Similarly if we add (0x) to both sides of the identity 0x + 0x = 0x
we see that 0x = 0.
Lemma 2.2 Let R be a ring. Then (x)y = (xy) and x(y) = (xy) for
all elements x and y of R.
Proof It follows from the Distributive Law that xy +(x)y = (x+(x))y =
0y = 0 and xy + x(y) = x(y + (y)) = x0 = 0. Therefore (x)y = (xy)
and x(y) = (xy).
A subset S of a ring R is said to be a subring of R if 0 S, a + b S,
a S and ab S for all a, b S.
A ring R is said to be commutative if xy = yx for all x, y R. Not every
ring is commutative: an example of a non-commutative ring is provided by
the ring of n n matrices with real or complex coefficients when n > 1.
30

A ring R is said to be unital if it possesses a (necessarily unique) non-zero


multiplicative identity element 1 satisfying 1x = x = x1 for all x R.
Definition A unital commutative ring R is said to be an integral domain if
the product of any two non-zero elements of R is itself non-zero.
Definition A field consists of a set K on which are defined operations of
addition and multiplication satisfying the following axioms:
x+y = y+x for all elements x and y of K (i.e., addition is commutative);
(x + y) + z = x + (y + z) for all elements x, y and z of K (i.e., addition
is associative);
there exists an an element 0 of K known as the zero element with the
property that x + 0 = x for all elements x of K;
given any element x of K, there exists an element x of K with the
property that x + (x) = 0;
xy = yx for all elements x and y of K (i.e., multiplication is commutative);
x(yz) = (xy)z for all elements x, y and z of K (i.e., multiplication is
associative);
there exists a non-zero element 1 of K with the property that 1x = x
for all elements x of K;
given any non-zero element x of K, there exists an element x1 of K
with the property that xx1 = 1;
x(y + z) = xy + xz and (x + y)z = xz + yz for all elements x, y and z
of K (the Distributive Law ).
An examination of the relevant definitions shows that a unital commutative ring R is a field if and only if, given any non-zero element x of R, there
exists an element x1 of R such that xx1 = 1. Moreover a ring R is a field
if and only if the set of non-zero elements of R is an Abelian group with
respect to the operation of multiplication.
Lemma 2.3 A field is an integral domain.

31

Proof A field is a unital commutative ring. Let x and y be non-zero elements


of a field K. Then there exist elements x1 and y 1 of K such that xx1 = 1
and yy 1 = 1. Then xyy 1 x1 = 1. It follows that xy 6= 0, since 0(y 1 x1 ) =
0 and 1 6= 0.
The set Z of integers is an integral domain with respect to the usual
operations of addition and multiplication. The sets Q, R and C of rational,
real and complex numbers are fields.

2.2

Ideals

Definition Let R be a ring. A subset I of R is said to be an ideal of R if


0 I, a + b I, a I, ra I and ar I for all a, b I and r R. An
ideal I of R is said to be a proper ideal of R if I 6= R.
Note that an ideal I of a unital ring R is proper if and only if 1 6 I.
Indeed if 1 I then r I for all r R, since r = r1.
Lemma 2.4 A unital commutative ring R is a field if and only if the only
ideals of R are {0} and R.
Proof Suppose that R is a field. Let I be a non-zero ideal of R. Then
there exists x I satisfying x 6= 0. Moreover there exists x1 R satisfying
xx1 = 1 = x1 x. Therefore 1 I, and hence I = R. Thus the only ideals
of R are {0} and R.
Conversely, suppose that R is a unital commutative ring with the property
that the only ideals of R are {0} and R. Let x be a non-zero element of R,
and let Rx denote the subset of R consisting of all elements of R that are of
the form rx for some r R. It is easy to verify that Rx is an ideal of R. (In
order to show that yr Rx for all y Rx and r R, one must use the fact
that the ring R is commutative.) Moreover Rx 6= {0}, since x Rx. We
deduce that Rx = R. Therefore 1 Rx, and hence there exists some element
x1 of R satisfying x1 x = 1. This shows that R is a field, as required.
The intersection of any collection of ideals of a ring R is itself an ideal
of R. For if a and b are elements of R that belong to all the ideals in the
collection, then the same is true of 0, a + b, a, ra and ar for all r R.
Let X be a subset of the ring R. The ideal of R generated by X is defined
to be the intersection of all the ideals of R that contain the set X. Note that
this ideal is well-defined and is the smallest ideal of R containing the set X
(i.e., it is contained in every other ideal that contains the set X).
32

We denote by (f1 , f2 , . . . , fk ) the ideal of R generated by any finite subset


{f1 , f2 , . . . , fk } of R. We say that an ideal I of the ring R is finitely generated
if there exists a finite subset of I which generates the ideal I.
Lemma 2.5 Let R be a unital commutative ring, and let X be a subset of
R. Then the ideal generated by X coincides with the set of all elements of
R that can be expressed as a finite sum of the form r1 x1 + r2 x2 + + rk xk ,
where x1 , x2 , . . . , xk X and r1 , r2 , . . . , rk R.
Proof Let I be the subset of R consisting of all these finite sums. If J is any
ideal of R which contains the set X then J must contain each of these finite
sums, and thus I J. Let a and b be elements of I. It follows immediately
from the definition of I that 0 I, a + b I, a I, and ra I for all
r R. Also ar = ra, since R is commutative, and thus ar I. Thus I
is an ideal of R. Moreover X I, since the ring R is unital and x = 1x
for all x X. Thus I is the smallest ideal of R containing the set X, as
required.
Each integer n generates an ideal nZ of the ring Z of integers. This ideal
consists of those integers that are divisible by n.
Lemma 2.6 Every ideal of the ring Z of integers is generated by some nonnegative integer n.
Proof The zero ideal is of the required form with n = 0. Let I be some
non-zero ideal of Z. Then I contains at least one strictly positive integer
(since m I for all m I). Let n be the smallest strictly positive integer
belonging to I. If j I then we can write j = qn + r for some integers q
and r with 0 r < n. Now r I, since r = j qn, j I and qn I.
But 0 r < n, and n is by definition the smallest strictly positive integer
belonging to I. We conclude therefore that r = 0, and thus j = qn. This
shows that I = nZ, as required.

2.3

Quotient Rings and Homomorphisms

Let R be a ring and let I be an ideal of R. If we regard R as an Abelian


group with respect to the operation of addition, then the ideal I is a (normal)
subgroup of R, and we can therefore form a corresponding quotient group
R/I whose elements are the cosets of I in R. Thus an element of R/I is of
the form I + x for some x R, and I + x = I + x0 if and only if x x0 I. If

33

x, x0 , y and y 0 are elements of R satisfying I + x = I + x0 and I + y = I + y 0


then
(x + y) (x0 + y 0 ) = (x x0 ) + (y y 0 ),
xy x0 y 0 = xy xy 0 + xy 0 x0 y 0 = x(y y 0 ) + (x x0 )y 0 .
But x x0 and y y 0 belong to I, and also x(y y 0 ) and (x x0 )y 0 belong
to I, since I is an ideal. It follows that (x + y) (x0 + y 0 ) and xy x0 y 0
both belong to I, and thus I + x + y = I + x0 + y 0 and I + xy = I + x0 y 0 .
Therefore the quotient group R/I admits well-defined operations of addition
and multiplication, given by
(I + x) + (I + y) = I + x + y,

(I + x)(I + y) = I + xy

for all I +x R/I and I +y R/I. One can readily verify that R/I is a ring
with respect to these operations. We refer to the ring R/I as the quotient of
the ring R by the ideal I.
Example Let n be an integer satisfying n > 1. The quotient Z/nZ of the
ring Z of integers by the ideal nZ generated by n is the ring of congruence
classes of integers modulo n. This ring has n elements, and is a field if and
only if n is a prime number.
Definition A function : R S from a ring R to a ring S is said to be a
homomorphism (or ring homomorphism) if and only if (x+y) = (x)+(y)
and (xy) = (x)(y) for all x, y R. If in addition the rings R and S are
unital then a homomorphism : R S is said to be unital if (1) = 1 (i.e.,
maps the identity element of R onto that of S).
Let R and S be rings, and let : R S be a ring homomorphism. Then
the kernel ker of the homomorphism is an ideal of R, where
ker = {x R : (x) = 0}.
The image (R) of the homomorphism is a subring of S; however it is not
in general an ideal of S.
An ideal I of a ring R is the kernel of the quotient homomorphism that
sends x R to the coset I + x.
Definition An isomorphism : R S between rings R and S is a homomorphism that is also a bijection between R and S. The inverse of an
isomorphism is itself an isomorphism. Two rings are said to be isomorphic
if there is an isomorphism between them.
34

The verification of the following result is a straightforward exercise.


Proposition 2.7 Let : R S be a homomorphism from a ring R to a
ring S, and let I be an ideal of R satisfying I ker . Then there exists a
unique homomorphism : R/I S such that (I + x) = (x) for all x R.
Moreover : R/I S is injective if and only if I = ker .
Corollary 2.8 Let : R S be ring homomorphism. Then (R) is isomorphic to R/ ker .

2.4

The Characteristic of a Ring

Let R be a ring, and let r R. We may define n.r for all natural numbers n
by recursion on n so that 1.r = r and n.r = (n 1).r + r for all n > 0. We
define also 0.r = 0 and (n).r = (n.r) for all natural numbers n. Then
(m + n).r = m.r + n.r,
(mn).r = m.(n.r),

n.(r + s) = n.r + n.s,


(m.r)(n.s) = (mn).(rs)

for all integers m an n and for all elements r and s of R.


In particular, suppose that R is a unital ring. Then the set of all integers n
satisfying n.1 = 0 is an ideal of Z. Therefore there exists a unique nonnegative integer p such that pZ = {n Z : n.1 = 0} (see Lemma 2.6). This
integer p is referred to as the characteristic of the ring R, and is denoted by
charR.
Lemma 2.9 Let R be an integral domain. Then either charR = 0 or else
charR is a prime number.
Proof Let p = charR. Clearly p 6= 1. Suppose that p > 1 and p = jk, where
j and k are positive integers. Then (j.1)(k.1) = (jk).1 = p.1 = 0. But R is
an integral domain. Therefore either j.1 = 0, or k.1 = 0. But if j.1 = 0 then
p divides j and therefore j = p. Similarly if k.1 = 0 then k = p. It follows
that p is a prime number, as required.

2.5

Polynomial Rings

Let R be a unital commutative ring. The set of all polynomials


c0 + c1 x + c2 x2 + + cn xn
in an indeterminate x with coefficients c0 , . . . , cn in the ring R themselves
constitute a ring, which we shall denote by R[x]. If the coefficient cn of
35

highest power of x is non-zero then the polynomial is said to be of degree n,


and the coefficient cn is referred to as the leading coefficient of the polynomial.
The polynomial is said to be monic if the leading coefficient cn is equal to
the multiplicative identity element 1 of the ring R.
Two polynomials with coefficients in the ring R are equal if and only if
they are of the same degree and corresponding coefficients are equal. Polynomials may be added, subtracted and multiplied in the usual fashion.
We now consider various properties of polynomials whose coefficients belong to a field K (such as the field of rational numbers, real numbers or
complex numbers).
Lemma 2.10 Let K be a field, and let f K[x] be a non-zero polynomial
with coefficients in K. Then, given any polynomial h K[x], there exist
unique polynomials q and r in K[x] such that h = f q + r and either r = 0
or else deg r < deg f .
Proof If deg h < deg f then we may take q = 0 and r = h. In general we
prove the existence of q and r by induction on the degree deg h of h. Thus
suppose that deg h deg f and that any polynomial of degree less than deg h
can be expressed in the required form. Now there is some element c of K
for which the polynomials h(x) and cf (x) have the same leading coefficient.
Let h1 (x) = h(x) cxm f (x), where m = deg h deg f . Then either h1 = 0
or deg h1 < deg h. The inductive hypothesis then ensures the existence
of polynomials q1 and r such that h1 = f q1 + r and either r = 0 or else
deg r < deg f . But then h = f q + r, where q(x) = cxm + q1 (x). We now
verify the uniqueness of q and r. Suppose that f q + r = f q + r, where
q, r K[x] and either r = 0 or deg r < deg f . Then (q q)f = r r. But
deg((q q)f ) deg f whenever q 6= q, and deg(r r) < deg f whenever
r 6= r. Therefore the equality (q q)f = r r cannot hold unless q = q and
r = r. This proves the uniqueness of q and r.
Any polynomial f with coefficients in a field K generates an ideal (f )
of the polynomial ring K[x] consisting of all polynomials in K[x] that are
divisible by f .
Lemma 2.11 Let K be a field, and let I be an ideal of the polynomial ring
K[x]. Then there exists f K[x] such that I = (f ), where (f ) denotes the
ideal of K[x] generated by f .
Proof If I = {0} then we can take f = 0. Otherwise choose f I such
that f 6= 0 and the degree of f does not exceed the degree of any non-zero
polynomial in I. Then, for each h I, there exist polynomials q and r in K[x]
36

such that h = f q + r and either r = 0 or else deg r < deg f . (Lemma 2.10).
But r I, since r = h f q and h and f both belong to I. The choice of f
then ensures that r = 0 and h = qf . Thus I = (f ).
Definition Polynomials f1 , f2 , . . . , fk with coefficients in some field K. are
said to be coprime if there is no non-constant polynomial that divides all of
them.
Theorem 2.12 Let f1 , f2 , . . . , fk be coprime polynomials with coefficients in
some field K. Then there exist polynomials g1 , g2 , . . . , gk with coefficients in
K such that
f1 (x)g1 (x) + f2 (x)g2 (x) + + fk (x)gk (x) = 1.
Proof Let I be the ideal in K[x] generated by f1 , f2 , . . . , fk . It follows from
Lemma 2.11 that the ideal I is generated by some polynomial d. Then d
divides all of f1 , f2 , . . . , fk and is therefore a constant polynomial, since these
polynomials are coprime. It follows that I = K[x]. But the ideal I of K[x]
generated by f1 , f2 , . . . , fk coincides with the subset of K[x] consisting of all
polynomials that may be represented as finite sums of the form
f1 (x)g1 (x) + f2 (x)g2 (x) + + fk (x)gk (x)
for some polynomials g1 , g2 , . . . , gk . It follows that the constant polynomial
with value 1 may be expressed as a sum of this form, as required.
Definition A non-constant polynomial f with coefficients in a field K is said
to be irreducible over K if it is not divisible by any non-constant polynomial
of lower degree with coefficients in K.
Any polynomial with coefficients in a field K may be factored as a product
of irreducible polynomials. This is easily proved by induction on the degree
of the polynomial, for if a non-constant polynomial is not itself irreducible,
then it can be factored as a product of polynomials of lower degree.
Lemma 2.13 Let K be a field. Then the ring K[x] of polynomials with
coefficients in K contains infinitely many irreducible polynomials.
Proof Let f1 , f2 , . . . , fk K[x] be irreducible polynomials, and let
g = f1 f2 fk + 1.
Then g is not divisible by f1 , f2 , . . . , fk , and therefore no irreducible factor
of g is divisible by any of f1 , f2 , . . . , fk . It follows that K[x] must contain
irreducible polynomials distinct from f1 , f2 , . . . , fk . Thus the number of irreducible polynomials in K[x] cannot be finite.
37

The proof of Lemma 2.13 is a direct analogue of Euclids proof of the


existence of infinitely many prime numbers.
Proposition 2.14 Let f , g and h be polynomials with coefficients in some
field K. Suppose that f is irreducible over K and that f divides the product
gh. Then either f divides g or else f divides h.
Proof Suppose that f does not divide g. We must show that f divides
h. Now the only polynomials that divide f are constant polynomials and
multiples of f . No multiple of f divides g. Therefore the only polynomials
that divide both f and g are constant polynomials. Thus f and g are coprime.
It follows from Proposition 2.12 that there exist polynomials u and v with
coefficients in K such that 1 = ug + vf . Then h = ugh + vf h. But f divides
ugh + vf h, since f divides gh. It follows that f divides h, as required.
Proposition 2.15 Let K be a field, and let (f ) be the ideal of K[x] generated
by an irreducible polynomial f with coefficients in K. Then K[x]/(f ) is a
field.
Proof Let I = (f ). Then the quotient ring K[x]/I is commutative and has
a multiplicative identity element I +1. Let g K[x]. Suppose that I +g 6= I.
Now the only factors of f are constant polynomials and constant multiples
of f , since f is irreducible. But no constant multiple of f can divide g, since
g 6 I. It follows that the only common factors of f and g are constant
polynomials. Thus f and g are coprime. It follows from Proposition 2.12
that there exist polynomials h, k K[x] such that f h + gk = 1. But then
(I +k)(I +g) = I +1 in K[x]/I, since f h I. Thus I +k is the multiplicative
inverse of I + g in K[x]/I. We deduce that every non-zero element of K[x]/I
is invertible, and thus K[x]/I is a field, as required.

2.6

Gausss Lemma

We shall show that a polynomial with integer coefficients is irreducible over


Q if and only if it cannot be expressed as a product of polynomials of lower
degree with integer coefficients.
Definition A polynomial with integer coefficients is said to be primitive if
there is no prime number that divides all the coefficients of the polynomial
Lemma 2.16 (Gausss Lemma) Let g and h be polynomials with integer
coefficients. If g and h are both primitive then so is gh.
38

Proof Let g(x) = b0 + b1 x + b2 x2 + + br xr and h(x) = c0 + c1 x + c2 x2 +


+ cs xs , and let g(x)h(x) = a0 + a1 x + a2 x2 + + ar+s xr+s . Let p be a
prime number. Then the polynomials g and h must both have at least one
coefficient that is not divisible by p. Let j and k be the smallest values of i
for which p does not divide bi and ci respectively. Then aj+k bj ck is divisible
j1
k1
P
P
by p, since aj+k bj ck =
bi cj+ki +
bj+ki ci , where p divides bi for all
i=0

i=0

i < j and p divides ci for all i < k. But p does not divide bj ck since p does
not divide either bj or ck . Therefore p does not divide the coefficient aj+k of
gh. This shows that the polynomial gh is primitive, as required.
Proposition 2.17 A polynomial with integer coefficients is irreducible over
the field Q of rational numbers if and only if it cannot be factored as a product
of polynomials of lower degree with integer coefficients.
Proof Let f be a polynomial with integer coefficients. If f is irreducible
over Q then f clearly cannot be factored as a product of polynomials of
lower degree with integer coefficients. Conversely suppose that f cannot be
factored in this way. Let f (x) = g(x)h(x), where g and h are polynomials
with rational coefficients. Then there exist positive integers r and s such that
the polynomials rg(x) and sh(x) have integer coefficients. Let the positive
integers u and v be the highest common factors of the coefficients of the
polynomials rg(x) and sh(x) respectively. Then rg(x) = ug (x) and sh(x) =
vh (x), where g and h are primitive polynomials with integer coefficients.
Then (rs)f (x) = (uv)g (x)h (x). We now show that f (x) = mg (x)h (x)
for some integer m. Let l be the smallest divisor of rs such that lf (x) =
mg (x)h (x) for some integer m. We show that l = 1. Suppose that it
were the case that l > 1. Then there would exist a prime factor p of l.
Now p could not divide m, since otherwise (l/p)f (x) = (m/p)g (x)h (x),
which contradicts the definition of l. Theorefore p would have to divide each
coefficient of g (x)h (x), which is impossible, since it follows from Gausss
Lemma (Lemma 2.16) that the product g h of the primitive polynomials
g and h is itself a primitive polynomial. Therefore l = 1 and f (x) =
mg (x)h (x). Now f does not factor as a product of polynomials of lower
degree with integer coefficients. Therefore either deg f = deg g = deg g, or
else deg f = deg h = deg h, Thus f is irreducible over Q, as required.

2.7

Eisensteins Irreducibility Criterion

Proposition 2.18 (Eisensteins Irreducibility Criterion) Let


f (x) = a0 + a1 x + a2 x2 + + an xn
39

be a polynomial of degree n with integer coefficients, and let p be a prime


number. Suppose that
p does not divide an ,
p divides a0 , a1 , . . . , an1 ,
p2 does not divide a0 .
Then the polynomial f is irreducible over the field Q of rational numbers.
Proof Suppose that f (x) = g(x)h(x), where g and h are polynomials with
integer coefficients. Let g(x) = b0 + b1 x + b2 x2 + + br xr and h(x) =
c0 +c1 x+c2 x2 + +cs xs . Then a0 = b0 c0 . Now a0 is divisible by p but is not
divisible by p2 . Therefore exactly one of the coefficients b0 and c0 is divisible
by p. Suppose that p divides b0 but does not divide c0 . Now p does not divide
all the coefficients of g(x), since it does not divide all the coefficients of f (x).
Let j be the smallest value of i for which p does not divide bi . Then p divides
j1
P
aj bj c0 , since aj bj c0 =
bi cji and bi is divisible by p when i < j. But
i=0

bj c0 is not divisible by p, since p is prime and neither bj nor c0 is divisible by


p. Therefore aj is not divisible by p, and hence j = n and deg g n = deg f .
Thus deg g = deg f and deg h = 0. Thus the polynomial f does not factor
as a product of polynomials of lower degree with integer coefficients, and
therefore f is irreducible over Q (Proposition 2.17).

40

Introduction to Galois Theory

3.1

Field Extensions and the Tower Law

Let K be a field. An extension L: K of K is an embedding of K in some


larger field L.
Definition Let L: K and M : K be field extensions. A K-homomorphism
: L M is a homomorphism of fields which satisfies (a) = a for all a K.
A K-monomorphism is an injective K-homomorphism. A K-isomorphism is
a bijective K-homomorphism. A K-automorphism of L is a K-isomorphism
mapping L onto itself.
Two extensions L1 : K and L2 : K of a field K are said to be K-isomorphic
(or isomorphic) if there exists a K-isomorphism : L1 L2 between L1 and
L2 .
If L: K is a field extension then we can regard L as a vector space over
the field K. If L is a finite-dimensional vector space over K then we say that
the extension L: K is finite. The degree [L: K] of a finite field extension L: K
is defined to be the dimension of L considered as a vector space over K.
Proposition 3.1 (The Tower Law) Let M : L and L: K be field extensions.
Then the extension M : K is finite if and only if M : L and L: K are both
finite, in which case [M : K] = [M : L][L: K].
Proof Suppose that M : K is a finite field extension. Then L, regarded as a
vector space over K, is a subspace of the finite-dimensional vector space M ,
and therefore L is itself a finite-dimensional vector space over K. Thus L: K
is finite. Also there exists a finite subset of M which spans M as a vector
space over K, since M : K is finite, and this finite subset must also span M
over L, and thus M : L must be finite.
Conversely suppose that M : L and L: K are both finite extensions. Let
x1 , x2 , . . . , xm be a basis for L, considered as a vector space over the field K,
and let y1 , y2 , . . . , yn be a basis for M , considered as a vector space over the
field L. Note that m = [L: K] and n = [M : L]. We claim that the set of
all products xi yj with i = 1, 2, . . . , m and j = 1, 2, . . . , n is a basis for M ,
considered as a vector space over K.
First we show that the elements xi yj are linearly independent over K.
m P
n
P
Suppose that
ij xi yj = 0, where ij K for all i and j. Then
i=1 j=1

m
P

ij xi L for all j, and y1 , y2 , . . . , yn are linearly independent over L,

i=1

41

and therefore

m
P

ij xi = 0 for j = 1, 2, . . . , n. But x1 , x2 , . . . , xm are linearly

i=1

independent over K. It follows that ij = 0 for all i and j. This shows that
the elements xi yj are linearly independent over K.
Now y1 , y2 , . . . , yn span M as a vector space over L, and therefore any
n
P
element z of M can be written in the form z =
j yj , where j L for
j=1

all j. But each j can be written in the form j =


for all i and j. But then z =

m P
n
P

m
P

ij xi , where ij K

i=1

ij xi yj . This shows that the products

i=1 j=1

xi yj span M as a vector space over K, and thus


{xi yj : 1 i m and 1 j n}
is a basis of M , considered as a vector space over K. We conclude that the
extension M : K is finite, and
[M : K] = mn = [M : L][L: K],
as required.
Let L: K be a field extension. If A is any subset of L, then the set K A
generates a subfield K(A) of L which is the intersection of all subfields of L
that contain K A. (Note that any intersection of subfields of L is itself a
subfield of L.) We say that K(A) is the field obtained from K by adjoining
the set A.
We denote K({1 , 2 , . . . , k }) by K(1 , 2 , . . . , k ) for any finite subset
{1 , 2 , . . . , k } of L. In particular K() denotes the field obtained by adjoining some element of L to K. A field extension L: K is said to be simple
if there exists some element of L such that L = K().

3.2

Algebraic Field Extensions

Definition Let L: K be a field extension, and let be an element of L. If


there exists some non-zero polynomial f K[x] with coefficients in K such
that f () = 0, then is said to be algebraic over K; otherwise is said to
be transcendental over K. A field extension L: K is said to be algebraic if
every element of L is algebraic over K.
Lemma 3.2 A finite field extension is algebraic.

42

Proof Let L: K be a finite field extension, and let n = [L: K]. Let L.
Then either the elements 1, , 2 , . . . , n are not all distinct, or else these
elements are linearly dependent over the field K (since a linearly independent subset of L can have at most n elements.) Therefore there exist
c0 , c1 , c2 , . . . , cn K, not all zero, such that
c0 + c1 + c2 2 + + cn n = 0.
Thus is algebraic over K. This shows that the field extension L: K is
algebraic, as required.
Definition A polynomial f with coefficients in some field or unital ring is
said to be monic if its leading coefficient (i.e., the coefficient of the highest
power of x occurring in f (x) with a non-zero coefficient) is equal to 1.
Lemma 3.3 Let K be a field and let be an element of some extension
field L of K. Suppose that is algebraic over K. Then there exists a unique
irreducible monic polynomial m K[x], with coefficients in K, characterized
by the following property: f K[x] satisfies f () = 0 if and only if m divides
f in K[x].
Proof Let I = {f K[x] : f () = 0}. Then I is a non-zero ideal of K[x].
Now there exists some polynomial m with coefficients in K which generates
the ideal I (Lemma 2.11). Moreover, by dividing m by its leading coefficient,
if necessary, we can ensure that m is a monic polynomial. Then f K[x]
satisfies f () = 0 if and only if m divides f .
Suppose that m = gh where g, h K[x]. Then 0 = m() = g()h().
But then either g() = 0, in which case m divides g, or else h() = 0, in
which case m divides h. The polynomial m is thus irreducible over K.
The polynomial m is uniquely determined since if some monic polynomial m also satisfies the required conditions then m and m divide one another
and therefore m = m.
Definition Let K be a field and let L be an extension field of K. Let be
an element of L that is algebraic over K. The minimum polynomial m of
over K is the unique irreducible monic polynomial m K[x] with coefficients
in K characterized by the following property: f K[x] satisfies f () = 0 if
and only if m divides f in K[x].
Note that if f K[x] is an irreducible monic polynomial, and if is a
root of f in some extension field L of K, then f is the minimum polynomial
of over K.
43

Theorem 3.4 A simple field extension K(): K is finite if and only if


is algebraic over K, in which case [K(): K] is the degree of the minimum
polynomial of over K.
Proof Suppose that the field extension K(): K is finite. It then follows
from Lemma 3.2 that is algebraic over K.
Conversely suppose that is algebraic over K. Let R = {f () : f
K[x]}. Now f () = 0 if and only if the minimum polynomial m of over
K divides f . It follows that f () = 0 if and only if f (m), where (m) is
the ideal of K[x] generated by m. The ring homomorphism from K[x] to R
that sends f K[x] to f () therefore induces an isomorphism between the
quotient ring K[x]/(m) and the ring R. But K[x]/(m) is a field, since m is
irreducible (Proposition 2.15). Therefore R is a subfield of K() containing
K {}, and hence R = K().
Let z K(). Then z = g() for some g K[x]. But then there exist
polynomials l and f belonging to K[x] such that g = lm + f and either f = 0
or deg f < deg m (Lemma 2.10). But then z = f () since m() = 0.
Suppose that z = h() for some polynomial h K[x], where either h = 0
or deg h < deg m. Then m divides hf , since is a zero of hf . But if hf
were non-zero then its degree would be less than that of m, and thus h f
would not be divisible by m. We therefore conclude that h = f . Thus any
element z of K() can be expressed in the form z = f () for some uniquely
determined polynomial f K[x] satisfying either f = 0 or deg f < deg m.
Thus if n = deg m then 1, , 2 . . . , n1 is a basis of K() over K. It follows
that the extension K(): K is finite and [K(): K] = deg m, as required.
Corollary 3.5 A field extension L: K is finite if and only if there exists
a finite subset {1 , 2 , . . . , k } of L such that i is algebraic over K for
i = 1, 2, . . . , k and L = K(1 , 2 , . . . , k ).
Proof Suppose that the field extension L: K is a finite. Then it is algebraic
(Lemma 3.2). Thus if {1 , 2 , . . . , k } is a basis for L, considered as a vector
space over K, then each i is algebraic and L = K(1 , 2 , . . . , k ).
Conversely suppose that L = K(1 , 2 , . . . , k ), where i is algebraic over
K for i = 1, 2, . . . , k. Let Ki = K(1 , 2 , . . . , i ) for i = 1, 2, . . . , k. Clearly
Ki1 (i ) Ki for all i > 1, since Ki1 Ki and i Ki . Also Ki
Ki1 (i ), since Ki1 (i ) is a subfield of L containing K {1 , 2 , . . . , i }
We deduce that Ki = Ki1 (i ) for i = 2, 3, . . . , k. Moreover i is clearly
algebraic over Ki1 since it is algebraic over K, and K Ki1 . It follows
from Theorem 3.4 that the field extension Ki : Ki1 is finite for each i. Using
the Tower Law (Proposition 3.1), we deduce that L: K is a finite extension,
as required.
44

Corollary 3.6 Let M : L and L: K be algebraic field extensions. Then M : K


is an algebraic field extension.
Proof Let be an element of M . We must show that is algebraic over
K. Now there exists some non-zero polynomial f L[x] with coefficients
in L such that f () = 0, since M : L is algebraic. Let 1 , 2 , . . . , k be
the coefficients of f (x), and let L0 = K(1 , 2 , . . . , k ). Now each i is
algebraic over K (since L: K is algebraic). Thus L0 : K is finite. Moreover
is algebraic over L0 , since the coefficients of the polynomial f belong to L0 ,
and thus L0 (): L0 is finite (Theorem 3.4). It follows from the Tower Law
(Proposition 3.1) that L0 (): K is finite. But then K(): K is finite, and
hence is algebraic over K, as required.

3.3

Algebraically Closed Fields

Definition A field K is said to be algebraically closed if, given any nonconstant polynomial f K[x] with coefficients in K, there exists some K
satisfying f () = 0.
The field C of complex numbers is algebraically closed. This result is the
Fundamental Theorem of Algebra.
Lemma 3.7 Let K be an algebraically closed field, and let L: K be an algebraic extension of K. Then L = K.
Proof Let L, and let m K[x] be the minimal polynomial of over
K. Then the polynomial m (x) has a root a in K, and is therefore divisible
by the polynomial x a. It follows that m (x) = x a, since m (x) is an
irreducible monic polynomial. But then = a, and therefore K. This
shows that every element of L belongs to K, and thus L = K, as required.

3.4

Ruler and Compass Constructions

One can make use of the Tower Law in order to prove the impossibility of
performing a number of geometric constructions in a finite number of steps
using straightedge and and compasses alone. These impossible constructions
include the following:
the trisection of an arbitrary angle;
the construction of the edge of a cube having twice the volume of some
given cube;
45

the construction of a square having the same area as a given circle.


Definition Let P0 and P1 be the points of the Euclidean plane given by
P0 = (0, 0) and P1 = (1, 0). We say that a point P of the plane is constructible
using straightedge and compasses alone if P = Pn for some finite sequence
P0 , P1 , . . . , Pn of points of the plane, where P0 = (0, 0), P1 = (1, 0) and, for
each j > 1, the point Pj is one of the following:
the intersection of two distinct straight lines, each passing through at
least two points belonging to the set {P0 , P1 , . . . , Pj1 };
the point at which a straight line joining two points belonging to the
set {P0 , P1 , . . . , Pj1 } intersects a circle which is centred on a point of
this set and passes through another point of the set;
the point of intersection of two distinct circles, where each circle is
centred on a point of the set {P0 , P1 , . . . , Pj1 } and passes through
another point of the set.
Constructible points of the plane are those that can be constructed from
the given points P0 and P1 using straightedge (i.e., unmarked ruler) and
compasses alone.
Theorem 3.8 Let (x, y) be a constructible point of the Euclidean plane.
Then [Q(x, y): Q] = 2r for some non-negative integer r.
Proof Let P = (x, y) and let P0 , P1 , . . . , Pn be a finite sequence of points
of the plane with the properties listed above. Let K0 = K1 = Q and
Kj = Kj1 (xj , yj ) for j = 2, 3, . . . , n, where Pj = (xj , yj ). Straightforward
coordinate geometry shows that, for each j, the real numbers xj and yj are
both roots of linear or quadratic polynomials with coefficients in Kj1 . It
follows that [Kj1 (xj ): Kj1 ] = 1 or 2 and [Kj1 (xj , yj ): Kj1 (xj )] = 1 or 2
for each j. It follows from the Tower Law (Proposition 3.1) that [Kn : Q] = 2s
for some non-negative integer s. But [Kn : Q] = [Kn : Q(x, y)][Q(x, y): Q]. We
deduce that [Q(x, y): Q] divides 2s , and therefore [Q(x, y): Q] = 2r for some
non-negative integer r.
One can apply this criterion to show that there is no geometrical construction that enables one to trisect an arbitrary angle using straightedge
and compasses alone. The same method can be used to show the impossibility of duplicating a cube or squaring a circle using straightedge and
compasses alone.
46

Example We show that there is no geometrical construction for the trisection of an angle of 3 radians (i.e., 60 ) using straightedge and compasses
alone. Let a = cos 9 and b = sin 9 . Now the point (cos 3 , sin 3 ) (i.e, the

point ( 12 , 12 3)) is constructible. Thus if an angle of 3 radians could be trisected using straightedge and compasses alone, then the point (a, b) would
be constructible. Now
cos 3 = cos cos 2 sin sin 2 = cos (cos2 sin2 ) 2 sin2 cos
= 4 cos3 3 cos
for any angle . On setting = 9 we deduce that 4a3 3a = 12 and thus
8a3 6a 1 = 0. Now 8a3 6a 1 = f (2a 1), where f (x) = x3 + 3x2 3.
An immediate application of Eisensteins criterion for irreducibility shows
that the polynomial f is irreducible over the field Q of rational numbers, and
thus [Q(a): Q] = [Q(2a 1): Q] = 3. It now follows from Theorem 3.8 that
the point (cos 9 , sin 9 ) is not constructible using straightedge and compasses
alone. Therefore it is not possible to trisect an angle of 3 radians using
straightedge and compasses alone. It follows that there is no geometrical
construction for the trisection of an arbitrary angle using straightedge and
compasses alone.
Example It is not difficult to see
that if it were possible to
construct two

3
3
points in the plane a distance 2 apart, then the point ( 2, 0) would be
constructible.
But it follows from Theorem 3.8 that this is impossible,

3
since 2is a root of the irreducible monic polynomial x3 2, and therefore [Q( 3 2), Q] = 3. We conclude that there is no geometric construction
using straightedge and compasses alone that will construct from a line segment in the plane a second line segment such that a cube with the second
line segment as an edge will have twice the volume of a cube with the first
line segment as an edge.
Example It can be shown
that is not algebraic over the field Q of rational

numbers. Therefore is not algebraic over Q. It then follows from Theorem 3.8 it is not possible to give a geometrical construction for obtaining a
square with the same area as a given circle, using straightedge and compasses
alone. (Thus it is not possible to square the circle using straightedge and
compasses alone.)
Lemma 3.9 If the endpoints of any line segment in the plane are constructible, then so is the midpoint.

47

Proof Let P and Q be constructible points in the plane. Let S and T be the
points where the circle centred on P and passing through Q intersects the
circle centred on Q and passing through P . Then S and T are constructible
points in the plane, and the point R at which the line ST intersects the
line P Q is the midpoint of the line segment P Q. Thus this midpoint is a
constructible point.
Lemma 3.10 If any three vertices of a parallelogram in the plane are constructible, then so is the fourth vertex.
Proof Let the vertices of the parallelogram listed in anticlockwise (or in
clockwise) order be A, B, C and D, where A, B and D are constructible
points. We must show that C is also constructible. Now the midpoint E of
the line segment BD is a constructible point, and the circle centred on E
and passing though A will intersect the line AE in the point C. Thus C is a
constructible point, as required.
Theorem 3.11 Let K denote the set of all real numbers x for which the
point (x, 0) is constructible using straightedge and compasses alone. Then K
is a subfield of the field of real numbers, and a point (x, y) of the plane is
constructible using straightedge and compass
alone if and only if x K and
y K. Moreover if x K and x > 0 then x K.
Proof Clearly 0 K and 1 K. Let x and y be real numbers belonging to
K. Then (x, 0) and (y, 0) are constructible points of the plane. Let M be the
midpoint of the line segment whose endpoints are (x, 0) and (y, 0). Then M
is constructible (Lemma 3.9), and M = ( 21 (x + y), 0). The circle centred on
M and passing through the origin intersects the x-axis at the origin and at
the point (x + y, 0). Therefore (x + y, 0) is a constructible point, and thus
x + y K. Also the circle centred on the origin and passing through (x, 0)
intersects the x-axis at (x, 0). Thus (x, 0) is a constructible point, and
thus x K.
We claim that if x K then the point (0, x) is constructible. Now if x K
and x 6= 0 then (x, 0) and (x, 0) are constructible points, and the circle
centred on (x, 0) and passing through (x, 0) intersects the circle centred on
(x, 0) and passing through
(x, 0) in two

points that lie on the y-axis. These


two points (namely (0, 3x) and (0, 3x)) are constructible, and therefore
the circle centred on the origin and passing though (x, 0) intersects the y-axis
in two constructible points which are (0, x) and (0, x). Thus if x K then
the point (0, x) is constructible.
Let x and y be real numbers belonging to K. Then the points (x, 0),
(0, y) and (0, 1) are constructible. The point (x, y 1) is then constructible,
48

since it is the fourth vertex of a parallelogram which has three vertices at the
constructible points (x, 0), (0, y) and (0, 1) (Lemma 3.10). But the line which
passes through the two constructible points (0, y) and (x, y 1) intersects
the x-axis at the point (xy, 0). Therefore the point (xy, 0) is constructible,
and thus xy K.
Now suppose that x K, y K and y 6= 0. The point (x, 1 y) is
constructible, since it is the fourth vertex of a parallelogram with vertices
at the constructible points (x, 0), (0, y) and (0, 1). The line segment joining
the constructible points (0, 1) and (x, 1 y) intersects the x-axis at the point
(xy 1 , 0). Thus xy 1 K.
The above results show that K is a subfield of the field of real numbers.
Moreover if x K and y K then the point (x, y) is constructible, since it is
the fourth vertex of a rectangle with vertices at the constructible points (0, 0),
(x, 0) and (0, y). Conversely, suppose that the point (x, y) is constructible.
We claim that the point (x, 0) is constructible and thus x K. This result is
obviously true if y = 0. If y 6= 0 then the circles centred on the points (0, 0)
and (1, 0) and passing through (x, y) intersect in the two points (x, y) and
(x, y). The point (x, 0) is thus the point at which the line passing through
the constructible points (x, y) and (x, y) intersects the x-axis, and is thus
itself constructible. The point (0, y) is then the fourth vertex of a rectangle
with vertices at the constructible points (0, 0), (x, 0) and (x, y), and thus is
itself constructible. The circle centred on the origin and passing though (0, y)
intersects the x-axis at (y, 0). Thus (y, 0) is constructible, and thus y K.
We have thus shown that a point (x, y) is constructible using straightedge
and compasses alone if and only if x K and y K.
Suppose that x K and that x > 0. Then 21 (1 x) K. Thus if
C = (0, 12 (1 x)) then C is a constructible point. Let (u, 0) be the point at
which the circle centred on C and passing through the constructible point
(0, 1) intersects the x-axis. (The circle does intersect the x-axis since it passes
through (0, 1) and (0, x), and x > 0.) The radius of this circle is 12 (1 + x)),
and therefore 14 (1 x)2 + u2 = 14 (1 + x)2 (Pythagoras Theorem.) But then
2
u
= x. But (u, 0) is a constructible point. Thus if x K and x > 0 then
x K, as required.
The above theorems can be applied to the problem of determining whether
or not it is possible to construct a regular n-sided polygon with a straightedge
and compass, given its centre and one of its vertices. The impossibility
of trisecting an angle of 60 shows that a regular 18-sided polygon is not
constructible using straightedge and compass. Now if one can construct a
regular n-sided polygon then one can easily construct a regular 2n-sided
polygon by bisecting the angles of the n-sided polygon. Thus the problem
49

reduces to that of determining which regular polygons with an odd number


of sides are constructible. Moreover it is not difficult to reduce down to the
case where n is a power of some odd prime number.
Gauss discovered that a regular 17-sided polygon was constructible in
1796, when he was 19 years old. Techniques of Galois Theory show that the
regular n-sided polygon is constructible using straightedge and compass if
and only if n = 2s p1 p2 pt , where p1 , p2 , . . . , pt are distinct Fermat primes:
a Fermat prime is a prime number that is of the form 2k +1 for some integer k.
If k = uv, where u and v are positive integers and v is odd, then 2k + 1 =
wv + 1 = (w + 1)(wv1 wv2 + w + 1), where w = 2u , and hence
m
2k + 1 is not prime. Thus any Fermat prime is of the form 22 + 1 for some
non-negative integer m. Fermat observed in 1640 that Fm is prime when
m 4. These Fermat primes have the values F0 = 3, F1 = 5, F2 = 17,
F3 = 257 and F4 = 65537. Fermat conjectured that all the numbers Fm were
prime. However it has been shown that Fm is not prime for any integer m
between 5 and 16. Moreover F16 = 265536 + 1 1020000 . Note that the five
Fermat primes 3, 5, 17, 257 and 65537 provide only 32 constructible regular
polygons with an odd number of sides.
It is not difficult to see that the geometric problem of constructing a
regular n-sided polygon using straightedge and compasses is equivalent to
the algebraic problem of finding a formula to express the nth roots of unity
in the complex plane in terms of integers or rational numbers by means of
algebraic formulae which involve finite addition, subtraction, multiplication,
division and the successive extraction of square roots. Thus the problem is
closely related to that of expressing the roots of a given polynomial in terms
of its coefficients by means of algebraic formulae which involve only finite
addition, subtraction, multiplication, division and the successive extraction
of pth roots for appropriate prime numbers p.

3.5

Splitting Fields

Definition Let L: K be a field extension, and let f K[x] be a polynomial


with coefficients in K. The polynomial f is said to split over L if f is a
constant polynomial or if there exist elements 1 , 2 , . . . , n of L such that
f (x) = c(x 1 )(x 2 ) (x n ),
where c K is the leading coefficient of f .
We see therefore that a polynomial f K[x] splits over an extension
field L of K if and only if f factors in L[x] as a product of constant or linear
factors.
50

Definition Let L: K be a field extension, and let f K[x] be a polynomial


with coefficients in K. The field L is said to be a splitting field for f over K
if the following conditions are satisfied:
the polynomial f splits over L;
the polynomial f does not split over any proper subfield of L that
contains the field K.
Lemma 3.12 Let M : K be a field extension, and let f K[x] be a polynomial with coefficients in K. Suppose that the polynomial f splits over M .
Then there exists a unique subfield L of M which is a splitting field for f
over K.
Proof Let L be the intersection of all subfields M 0 of M containing K with
the property that the polynomial f splits over M 0 . One can readily verify
that L is the unique splitting field for f over K contained in M .
The Fundamental Theorem of Algebra ensures that a polynomial f Q[x]
with rational coefficients always splits over the field C of complex numbers.
Thus some unique subfield L of C is a splitting field for f over Q.
Note that if the polynomial f K[x] splits over an extension field M of
K, and if 1 , 2 , . . . , n are the roots of the polynomial f in M , then the
unique splitting field of f over K contained in M is the field K(1 , 2 , . . . , n )
obtaining on adjoining the roots of f to K.

Example The field Q( 2) is a splitting field for the polynomial x2 2 over


Q.
We shall prove below that splitting fields always exist and that any two
splitting field extensions for a given polynomial over a field K are isomorphic.
Given any homomorphism : K M of fields, we define
(a0 + a1 x + + an xn ) = (a0 ) + (a1 )x + + (an )xn
for all polynomials a0 + a1 x + + an xn with coefficients in K. Note that
(f + g) = (f ) + (g) and (f g) = (f ) (g) for all f, g K[x].
Theorem 3.13 (Kronecker) Let K be a field, and let f K[x] be a nonconstant polynomial with coefficients in K. Then there exists an extension
field L of K and an element of L for which f () = 0.

51

Proof Let g be an irreducible factor of f , and let L = K[x]/(g), where (g)


is the ideal of K[x] generated by g. For each a K let i(a) = a + (g). Then
i: K L is a monomorphism. We embed K in L on identifying a K with
i(a).
Now L is a field, since g is irreducible (Proposition 2.15). Let = x+(g).
Then g() is the image of the polynomial g under the quotient homomorphism from K[x] to L, and therefore g() = 0. But g is a factor of the
polynomial f . Therefore f () = 0, as required.
Corollary 3.14 Let K be a field and let f K[x]. Then there exists a
splitting field for f over K.
Proof We use induction on the degree deg f of f . The result is trivially true
when deg f = 1 (since f then splits over K itself). Suppose that the result
holds for all fields and for all polynomials of degree less than deg f . Now it
follows from Theorem 3.13 that there exists a field extension K1 : K of K and
an element of K1 satisfying f () = 0. Moreover f (x) = (x )g(x) for
some polynomial g with coefficients in K(). Now deg g < deg f . It follows
from the induction hypothesis that there exists a splitting field L for g over
K(). Then f splits over L.
Suppose that f splits over some field M , where K M L. Then
M and hence K() M . But M must also contain the roots of g,
since these are roots of f . It follows from the definition of splitting fields
that M = L. Thus L is the required splitting field for the polynomial f over
K.
Any two splitting fields for a given polynomial with coefficients in a field K
are K-isomorphic. This result is a special case of the following theorem.
Theorem 3.15 Let K1 and K2 be fields, and let : K1 K2 be an isomorphism between K1 and K2 . Let f K1 [x] be a polynomial with coefficients
in K1 , and let L1 and L2 be splitting fields for f and (f ) over K1 and K2
respectively. Then there exists an isomorphism : L1 L2 which extends
: K1 K2 .
Proof We prove the result by induction on [L1 : K1 ]. The result is trivially
true when [L1 : K1 ] = 1. Suppose that [L1 : K1 ] > 1 and the result holds for
splitting field extensions of lower degree. Choose a root of f in L1 \K1 , and
let m be the minimum polynomial of over K1 . Then m divides f and (m)
divides (f ), and therefore (m) splits over L2 . Moreover the polynomial
(m) is irreducible over K2 , since : K1 K2 induces an isomorphism
between the polynomial rings K1 [x] and K2 [x]. Choose a root of (m).
52

Let g and h be polynomials with coefficients in K1 . Now g() = h()


if and only if m divides g h. Similarly (g)() = (h)() if and only if
(m) divides (g) (h). Therefore (g)() = (h)() if and only if
g() = h(), and thus there is a well-defined isomorphism : K1 () K2 ()
which sends g() to (g)() for any polynomial g with coefficients in K.
Now L1 and L2 are splitting fields for the polynomials f and (f ) over the
fields K1 () and K2 () respectively, and [L1 : K1 ()] < [L1 : K1 ]. The induction hypothesis therefore ensures the existence of an isomorphism : L1 L2
extending : K1 () K2 (). Then : L1 L2 is the required extension of
: K1 K2 .
Corollary 3.16 Let L: K be a splitting field extension, and let and be
elements of L. Then there exists a K-automorphism of L sending to if
and only if and have the same minimum polynomial over K.
Proof Suppose that there exists a K-automorphism of L which sends
to . Then h() = (h()) for all polynomials h K[x] with coefficients in
K. Therefore h() = 0 if and only if h() = 0. It follows that and must
have the same minimum polynomial over K.
Conversely suppose that and are elements of L that have the same
minimum polynomial m over K. Let h1 and h2 be polynomials with coefficients in K. Now h1 () = h2 () if and only if h1 h2 is divisible by the
minimum polynomial m. It follows that h1 () = h2 () if and only if h1 () =
h2 (). Therefore there is a well-defined K-isomorphism : K() K()
that sends h() to h() for all polynomials h with coefficients in K. Then
() = .
Now L is the splitting field over K for some polynomial f with coefficients
in K. The field L is then a splitting field for f over both K() and K(). It
follows from Theorem 3.15 that the K-isomorphism : K() K() extends
to a K-automorphism of L that sends to , as required.

3.6

Normal Extensions

Definition A field extension L: K is said to be normal if every irreducible


polynomial in K[x] with at least one root in L splits over L.
Note that a field extension L: K is normal if and only if, given any element of L, the minimum polynomial of over K splits over L.
Theorem 3.17 Let K be a field, and let L be an extension field of K. Then
L is a splitting field over K for some polynomial with coefficients in K if and
only if the field extension L: K is both finite and normal.
53

Proof Suppose that L: K is both finite and normal. Then there exist algebraic elements 1 , 2 , . . . , n of L such that L = K(1 , 2 , . . . , n ) (Corollary 3.5). Let f (x) = m1 (x)m2 (x) mn (x), where mj K[x] is the minimum polynomial of j over K for j = 1, 2, . . . , n. Then mj splits over L since
mj is irreducible and L: K is normal. Thus f splits over L. It follows that
L is a splitting field for f over K, since L is obtained from K by adjoining
roots of f .
Conversely suppose that L is a splitting field over K for some polynomial
f K[x]. Then L is obtained from K by adjoining the roots of f , and
therefore the extension L: K is finite. (Corollary 3.5).
Let g K[x] be irreducible, and let M be a splitting field for the polynomial f g over L. Then L M and the polynomials f and g both split over
M . Let and be roots of g in M . Now the polynomial f splits over the
fields L() and L(). Moreover if f splits over any subfield of M containing
K() then that subfield must contain L (since L is a splitting field for f over
K) and thus must contain L(). We deduce that L() is a splitting field for
f over K(). Similarly L() is a splitting field for f over K().
Now there is a well-defined K-isomorphism : K() K() which sends
h() to h() for all polynomials h with coefficients in K, since two such polynomials h1 and h2 take the same value at a root of the irreducible polynomial g if and only if their difference h1 h2 is divisible by g. This isomorphism
: K() K() extends to an K-isomorphism : L() L() between L()
and L(), since L() and L() are splitting fields for f over the field K() and
K() respectively (Theorem 3.15). Thus the extensions L(): K and L(): K
are isomorphic, and [L(): K] = [L(): K]. But [L(): K] = [L(): L][L: K]
and [L(): K] = [L(): L][L: K] by the Tower Law (Theorem 3.1). It follows
that [L(): L] = [L(): L]. In particular L if and only if L. This
shows that that any irreducible polynomial with a root in L must split over
L, and thus L: K is normal, as required.

3.7

Separability

Let K be a field. We recall that nk is defined inductively for all integers n


and for all elements k of K so that 0k = 0 and (n + 1)k = nk + k for all
n Z and k K. Thus 1k = k, 2k = k + k, 3k = k + k + k etc., and
(n)k = (nk) for all n Z.
Definition Let K be a field, and let f K[x] be a polynomial with coeffin
P
cients c0 , c1 , . . . , cn in K, where f (x) =
cj xj . The formal derivative Df
j=0

54

of f is defined by the formula (Df )(x) =

n
P

jcj xj1 .

j=1

(The definition of formal derivative given above is a purely algebraic definition, applying to polynomials with coefficients in any field whatsoever,
which corresponds to the formula for the derivative of a polynomial with real
coefficients obtained by elementary calculus.)
Let K be a field. One can readily verify by straightforward calculation
that D(f + g) = Df + Dg and D(f g) = (Df )g + f (Dg) for all f K[x]. If
f is a constant polynomial then Df = 0.
Let K be a field, and let f K[x]. An element of an extension field L
of K is said to be a repeated zero if (x )2 divides f (x).
Proposition 3.18 Let K be a field, and let f K[x]. The polynomial f
has a repeated zero in a splitting field for f over K if and only if there exists
a non-constant polynomial with coefficients in K that divides both f and its
formal derivative Df in K[x].
Proof Suppose that f K[x] has a repeated root in a splitting field L.
Then f (x) = (x )2 h(x) for some polynomial h L[x]. But then
(Df )(x) = 2(x )h(x) + (x )2 (Dh)(x)
and hence (Df )() = 0. It follows that the minimum polynomial of over
K is a non-constant polynomial with coefficients in K which divides both f
and Df .
Conversely let f K[x] be a polynomial with the property that f and
Df are both divisible by some non-constant polynomial g K[x]. Let L be
a splitting field for f over K. Then g splits over L (since g is a factor of f ).
Let L be a root of g. Then f () = 0, and hence f (x) = (x )e(x)
for some polynomial e L[x]. On differentiating, we find that (Df )(x) =
e(x) + (x )De(x). But (Df )() = 0, since g() = 0 and g divides Df
in K[x]. It follows that e() = (Df )() = 0, and thus e(x) = (x )h(x)
for some polynomial h L[x]. But then f (x) = (x )2 h(x), and thus the
polynomial f has a repeated root in the splitting field L, as required.
Definition Let K be a field. An irreducible polynomial in K[x] is said to
be separable over K if it does not have repeated roots in a splitting field. A
polynomial in K[x] is said to separable over K if all its irreducible factors
are separable over K. A polynomial is said to be inseparable if it is not
separable.

55

Corollary 3.19 Let K be a field. An irreducible polynomial f is inseparable


if and only if Df = 0.
Proof Let f K[x] be an irreducible polynomial. Suppose that f is inseparable. Then f has a repeated root in a splitting field, and it follows
from Proposition 3.18 that there exists a non-constant polynomial g in K[x]
dividing both f and its formal derivative Df . But then g = cf for some
non-zero element c of K, since f is irreducible, and thus f divides Df . But
if Df were non-zero then deg Df < deg f , and thus f would not divide Df .
Thus Df = 0.
Conversely if Df = 0 then f divides both f and Df . It follows from
Proposition 3.18 that f has a repeated root in a splitting field, and is thus
inseparable.
Definition An algebraic field extension L: K is said to be separable over K
if the minimum polynomial of each element of L is separable over K.
Suppose that K is a field of characteristic zero. Then n.k 6= 0 for all
n Z and k K satisfying n 6= 0 and k 6= 0. It follows from the definition
of the formal derivative that Df = 0 if and only if f K[x] is a constant
polynomial. The following result therefore follows immediately from Corollary 3.19.
Corollary 3.20 Suppose that K is a field of characteristic zero. Then every
polynomial with coefficients in K is separable over K, and thus every field
extension L: K of K is separable.

3.8

Finite Fields

Lemma 3.21 Let K be a field of characteristic p, where p > 0. Then (x +


y)p = xp + y p and (xy)p = xp y p for all x, y K. Thus the function x 7 xp
is a monomorphism mapping the field K into itself.
p

Proof The Binomial Theorem tells us that (x + y) =

p  
X
p

xj y pj , where

j=0
 
 
p
p
p(p 1) (p j + 1)
for j = 1, 2, . . . , p. The de= 1 and
=
j!
0
j
nominator of each binomial coefficient must divide the numerator, since this
coefficient is an integer. Now the characteristic p of K is a prime number.
Moreover if 0 < j < p then p is a factor of the numerator but is not a factor
of the denominator. It follows from the Fundamental Theorem of Arithmetic

56

 
p
that p divides
for all j satisfying 0 < j < p. But px = 0 for all x K,
j
since charK = p. Therefore (x + y)p = xp + y p for all x, y K. The identity
(xy)p = xp y p is immediate from the commutativity of K.
Let K be a field of characteristic p, where p > 0. The monomorphism
x 7 xp is referred to as the Frobenius monomorphism of K. If K is finite then
this monomorphism is an automorphism of K, since any injection mapping
a finite set into itself must be a bijection.
Theorem 3.22 A field K has pn elements if and only if it is a splitting field
n
for the polynomial xp x over its prime subfield Fp , where Fp
= Z/pZ.
Proof Suppose that K has q elements, where q = pn . If K \ {0} then
q1 = 1, since the set of non-zero elements of K is a group of order q 1
with respect to multiplication. It follows that q = for all K. Thus
all elements of K are roots of the polynomial xq x. This polynomial must
therefore split over K, since its degree is q and K has q elements. Moreover
the polynomial cannot split over any proper subfield of K. Thus K is a
splitting field for this polynomial.
Conversely suppose that K is a splitting field for the polynomial f over
Fp , where f (x) = xq x and q = pn . Let () = q for all K.
Then : K K is a monomorphism, being the composition of n successive
applications of the Frobenius monomorphism of K. Moreover an element
of K is a root of f if and only if () = . It follows from this that
the roots of f constitute a subfield of K. This subfield is the whole of
K, since K is a splitting field. Thus K consists of the roots of f . Now
Df (x) = qxq1 1 = 1, since q is divisible by the characteristic p of Fp . It
follows from Proposition 3.18 that the roots of f are distinct. Therefore f
has q roots, and thus K has q elements, as required.
Let K be a finite field of characteristic p. Then K has pn elements, where
n = [K: Fp ], since any vector space of dimension n over a field of order p must
have exactly pn elements. The following result is now a consequence of the
existence of splitting fields (Corollary 3.14) and the uniqueness of splitting
fields up to isomorphism (Theorem 3.15)
Corollary 3.23 There exists a finite field GF(pn ) of order pn for each prime
number p and positive integer n. Two finite fields are isomorphic if and only
if they have the same number of elements.

57

The field GF(pn ) is referred to as the Galois field of order pn .


The non-zero elements of a field constitute a group under multiplication.
We shall prove that all finite subgroups of the group of non-zero elements of
a field are cyclic. It follows immediately from this that the group of non-zero
elements of a finite field is cyclic.
For each positive integer n, we denote by (n) the number of integers
x
X
satisfying 0 x < n that are coprime to n. We show that the sum
(d)
d|n

of (d) taken over all divisors of a positive integer n is equal to n.


Lemma 3.24 Let n be a positive integer. Then

(d) = n.

d|n

Proof If x is an integer satisfying


X0 x < n then (x, n) = n/d for some
divisor d of n. It follows that n =
nd , where nd is the number of integers x
d|n

satisfying 0 x < n for which (x, n) = n/d. Thus it suffices to show that
nd = (d) for each divisor d of n.
Let d be a divisor of n, and let a = n/d. Given any integer x satisfying
0 x < n that is divisible by a, there exists an integer y satisfying 0 y < d
such that x = ay. Then (x, n) = (ay, ad) = a(y, d). It follows that the
integers x satisfying 0 x < n for which (x, n) = a are those of the form
ay, where y is an integer, 0 y < d and (y, d) = 1. It follows that there
are exactly (d) integersX
x satisfying 0 x < n for which (x, n) = n/d, and
thus nd = (d) and n =
(d), as required.
d|n

The set of all non-zero elements of a field is a group with respect to the
operation of multiplication.
Theorem 3.25 Let G be a finite subgroup of the group of non-zero elements
of a field. Then the group G is cyclic.
Proof Let n be the order of the group G. It follows from Lagranges Theorem
that the order of every element of G divides n. For each divisor dX
of n, let (d)
denote the number of elements of G that are of order d. Clearly
(d) = n.
d|n

Let g be an element of G of order d, where d is a divisor of n. The elements


1, g, g 2 , . . . , g d1 are distinct elements of G and are roots of the polynomial
xd 1. But a polynomial of degree d with coefficients in a field has at most
d roots in that field. Therefore every element x of G satisfying xd = 1 is g k
58

for some uniquely determined integer k satisfying 0 k < d. If k is coprime


to d then g k has order d, for if (g k )n = 1 then d divides kn and hence d
divides n. Conversely if g k has order d then d and k are coprime, for if e is
a common divisor of k and d then (g k )d/e = g d(k/e) = 1, and hence e = 1.
Thus if there exists at least one element g of G that is of order d then the
elements of G that are of order d are the elements g k for those integers k
satisfying 0 k < d that are coprime to d. It follows that if (d) > 0 then
(d) = (d), where (d) is the number of integers k satisfying 0 k < d
that are coprime to d.
X
Now 0 (d) (d) for each divisor d of n. But
(d) = n and
d|n

(d) = n. It follows that (d) = (d) for each divisor d of n. In

d|n

particular (n) = (n) 1. Thus there exists an element of G whose order


is the order n of G. This element generates G, and thus G is cyclic, as
required.
Corollary 3.26 The group of non-zero elements of a finite field is cyclic.

3.9

The Primitive Element Theorem

Theorem 3.27 (Primitive Element Theorem) Every finite separable field


extension is simple.
Proof Let L: K be a finite separable field extension. Suppose that K is a
finite field. Then L is also a finite field, since it is a finite-dimensional vector
space over K. The group of non-zero elements of L is therefore generated by
a single non-zero element of L (Corollary 3.26). But then L = K() and
thus L: K is simple. This proves the Primitive Element Theorem in the case
where the field K is finite.
Next suppose that L = K(, ), where K is infinite, and are algebraic
over K and L: K is separable. Let N be a splitting field for the polynomial
f g, where f and g are the minimum polynomials of and respectively over
K. Then f and g both split over N . Let 1 , 2 , . . . , q be the roots of f in
N , and let 1 , 2 , . . . , r be the roots of g in N , where 1 = and 1 = .
The separability of L: K ensures that k 6= j when k 6= j.
Now K is infinite. We can therefore choose c K so that c 6= (i
)/( j ) for any i and j with j 6= 1. Let h(x) = f ( cx), where
= + c. Then h is a polynomial in the indeterminate x with coefficients
in K() which satisfies h() = f () = 0. Moreover h(j ) 6= 0 whenever
j 6= 1, since cj 6= i for all i and j with j 6= 1. Thus is the only
59

common root of g and h. It follows that x is a highest common factor of


g and h in the polynomial ring K()[x], and therefore K(). But then
K(), since = c and c K. It follows that L = K().
It now follows by induction on m that if L = K(1 , 2 , . . . , m ), where K
is infinite, 1 , 2 , . . . , m are algebraic over K, and L: K is separable, then
the extension L: K is simple. Thus all finite separable field extensions are
simple, as required.

3.10

The Galois Group of a Field Extension

Definition The Galois group (L: K) of a field extension L: K is the group


of all automorphisms of the field L that fix all elements of the subfield K.
Lemma 3.28 If L: K is a finite separable field extension then |(L: K)|
[L: K].
Proof It follows from the Primitive Element Theorem (Theorem 3.27) that
there exists some element of L such that L = K(). Let be an element
of L. Then = g() for some polynomial g with coefficients in K. But then
() = g(()) for all (L: K), since the coefficients of g are fixed by .
It follows that each automorphism in (L: K) is uniquely determined once
() is known.
Let f be the minimum polynomial of over K. Then
f (()) = (f ()) = 0
for all (L: K) since the coefficients of f are in K and are therefore fixed
by . Thus () is a root of f . It follows that the order |(L: K)| of the
Galois group is bounded above by the number of roots of f that belong to
L, and is thus bounded above by the degree deg f of f . But deg f = [L: K]
(Theorem 3.4). Thus |(L: K)| [L: K], as required.
Definition Let L be a field, and let G be a group of automorphisms of L.
The fixed field of G is the subfield K of L defined by
K = {a L : (a) = a for all G}.
Proposition 3.29 Let L be a field, let G be a finite group of automorphisms
of L, and let K be the fixed field of G. Then each element of L is algebraic
over K, and the minimum polynomial of over K is the polynomial
(x 1 )(x 2 ) (x k ),
where 1 , 2 , . . . , k are distinct and are the elements of the orbit of under
the action of G on L.
60

Proof Let f (x) = (x 1 )(x 2 ) (x k ). Then the polynomial f is


invariant under the action of G, since each automorphism in the group G
permutes the elements 1 , 2 , . . . , k and therefore permutes the factors of
f amongst themselves. It follows that the coefficients of the polynomial f
belong to the fixed field K of G. Thus is algebraic over K, as it is a root
of the polynomial f .
Now, given any root i of f , there exists some G such that i =
(). Thus if g K[x] is a polynomial with coefficients in K which satisfies
g() = 0 then g(i ) = (g()) = 0, since the coefficients of g are fixed by .
But then f divides g. Thus f is the minimum polynomial of over K, as
required.
Definition A field extension is said to be a Galois extension if it is finite,
normal and separable.
Theorem 3.30 Let L be a field, let G be a finite subgroup of the group of
automorphisms of L, and let K be the fixed field of G. Then the field extension
L: K is a Galois extension. Moreover G is the Galois group (L: K) of L: K
and |G| = [L: K].
Proof It follows from Proposition 3.29 that, for each L, the minimum
polynomial of over K splits over L and has no multiple roots. Thus the
extension L: K is both normal and separable.
Let M be any field satisfying K M L for which the extension M : K
is finite. The extension M : K is separable, since L: K is separable. It follows
from the Primitive Element Theorem (Theorem 3.27) that the extension
M : K is simple. Thus M = K() for some L. But then [M : K] is equal
to the degree of the minimum polynomial of over K (Theorem 3.4). It
follows from Proposition 3.29 that [M : K] is equal to the number of elements
in the orbit of under the action of G on L. Therefore [M : K] divides |G|
for any intermediate field M for which the extension M : K is finite.
Now let the intermediate field M be chosen so as to maximize [M : K].
If L then is algebraic over K, and therefore [M (): M ] is finite. It
follows from the Tower Law (Theorem 3.1) that [M (): K] is finite, and
[M (): K] = [M (): M ][M : K]. But M has been chosen so as to maximize
[M : K]. Therefore [M (): K] = [M : K], and [M (): M ] = 1. Thus M .
We conclude that M = L. Thus L: K is finite and [L: K] divides |G|.
The field extension L: K is a Galois extension, since it has been shown to
be finite, normal and separable. Now G (L: K) and |(L: K)| [L: K]
(Lemma 3.28). Therefore |(L: K)| [L: K] |G| |(L: K)|, and thus
G = (L: K) and |G| = [L: K], as required.
61

Theorem 3.31 Let (L: K) be the Galois group of a finite field extension
L: K. Then |(L: K)| divides [L: K]. Moreover |(L: K)| = [L: K] if and only
if L: K is a Galois extension, in which case K is the fixed field of (L: K).
Proof Let M be the fixed field of (L: K). It follows from Theorem 3.30
that L: M is a Galois extension and |(L: K)| = [L: M ]. Now [L: K] =
[L: M ][M : K] by the Tower Law (Theorem 3.1). Thus |(L: K)| divides
[L: K]. If |(L: K)| = [L: K] then M = K. But then L: K is a Galois
extension and K is the fixed field of (L: K).
Conversely suppose that L: K is a Galois extension. We must show that
|(L: K)| = [L: K]. Now the extension L: K is both finite and separable. It
follows from the Primitive Element Theorem (Theorem 3.27) that there exists
some element of L such that L = K(). Let f be the minimum polynomial
of over K. Then f splits over L, since f is irreducible and the extension
L: K is normal. Let 1 , 2 , . . . , n be the roots of f in L, where 1 = and
n = deg f . If is a K-automorphism of L then f (()) = (f ()) = 0, since
the coefficients of the polynomial f belong to K and are therefore fixed by
. Thus () = j for some j. We claim that, for each root j of f , there is
exactly one K-automorphism j of L satisfying j () = j .
Let g(x) and h(x) be polynomials with coefficients in K. Suppose that
g() = h(). Then g h is divisible by the minimum polynomial f of .
It follows that g(j ) = h(j ) for any root j of f . Now every element of
L is of the form g() for some g K[x], since L = K(). We deduce
therefore that there is a well-defined function j : L L with the property
that j (g()) = g(j ) for all g K[x]. The definition of this function ensures
that it is the unique automorphism of the field L that fixes each element of
K and sends to j .
Now the roots of the polynomial f in L are distinct, since f is irreducible
and L: K is separable. Moreover the order of the Galois group (L: K) is
equal to the number of roots of f , since each root determines a unique element
of the Galois group. Therefore |(L: K)| = deg f . But deg f = [L: K] since
L = K() and f is the minimum polynomial of over K (Theorem 3.4).
Thus |(L: K)| = [L: K], as required.

3.11

The Galois correspondence

Proposition 3.32 Let K, L and M be fields satisfying K M L. Suppose that L: K is a Galois extension. Then so is L: M . If in addition M : K
is normal, then M : K is a Galois extension.
Proof Let L and let fK K[x] and fM M [x] be the minimum
polyomials of over K and M respectively. Then fK splits over L, since fK
62

is irreducible over K and L: K is a normal extension. Also the roots of fK in


L are distinct, since L: K is a separable extension. But fM divides fK , since
fK () = 0 and the coefficients of fK belong to M . It follows that fM also
splits over L, and its roots are distinct. We deduce that the finite extension
L: M is both normal and separable, and is therefore a Galois extension.
The finite extension M : K is clearly separable, since L: K is separable.
Thus if M : K is a normal extension then it is a Galois extension.
Proposition 3.33 Let L: K be a Galois extension, and let M be a field
satisfying K M L. Then the extension M : K is normal if and only if
(M ) = M for all (L: K).
Proof Let be an element of M , and let f K[x] be the minimum polynomial of over K. Now K is the fixed field of the Galois group (L: K), since
the field extension L: K is a Galois extension (Theorem 3.31). It follows that
the polynomial f splits over L, and the roots of f are the elements of the
orbit of under the action of (L: K) on L (Proposition 3.29). Therefore f
splits over M if and only if () M for all (L: K). Now the extension
M : K is normal if and only if the minimum polynomial of any element of M
over K splits over M . It follows that the extension M : K is normal if and
only if (M ) M for all (L: K). But if (M ) M for all (L: K)
then 1 (M ) M and M = ( 1 (M )) (M ) and thus (M ) = M
for all (L: K). Therefore the extension M : K is normal if and only if
(M ) = M for all (L: K).
Corollary 3.34 Let L: K be a Galois extension, and let M be a field satisfying K M L. Suppose that the extension M : K is normal. Then the
restriction |M to M of any K-automorphism of L is a K-automorphism
of M .
Proof Let (L: K) be a K-automorphism of L. We see from Proposition 3.33 that (M ) = M . Similarly 1 (M ) = M . It follows that the
restrictions |M : M M and 1 |M : M M of and 1 to M are Khomomorphisms mapping M into itself. Moreover 1 |M : M M is the
inverse of |M : M M . Thus |M : M M is an isomorphism, and is thus
a K-automorphism of M , as required.
Theorem 3.35 (The Galois Correspondence) Let L: K be a Galois extension
of a field K. Then there is a natural bijective correspondence between fields M
satisfying K M L and subgroups of the Galois group (L: K) of the
extension L: K. If M is a field satisfying K M L then the subgroup
of (L: K) corresponding to M is the Galois group (L: M ) of the extension
63

L: M . If G is a subgroup of (L: K) then the subfield of L corresponding to


G is the fixed field of G. Moreover the extension M : K is normal if and only
if (L: M ) is a normal subgroup of the Galois group (L: K), in which case
(M : K)
= (L: K)/(L: M ).
Proof Let M be a subfield of L containing K. Then L: M is a Galois extension (Proposition 3.32). The existence of the required bijective correspondence between fields M satisfying K M L and subgroups of the Galois
group (L: K) follows immediately from Theorem 3.30 and Theorem 3.31.
Let M be a field satisfying K M L. Now the extension M : K is
normal if and only if (M ) = M for all (L: K). (Proposition 3.33).
Let H = (L: M ). Then M = (M ) if and only if H = H 1 , since M
and (M ) are the fixed fields of H and H 1 respectively, and there is a
bijective correspondence between subgroups of the Galois group (L: K) and
their fixed fields. Thus the extension M : K is normal if and only if (L: M )
is a normal subgroup of (L: K).
Finally suppose that M : K is a normal extension. For each (L: K),
let () be the restriction |M of to M . Then : (L: K) (M : K) is a
group homomorphism whose kernel is (L: M ). We can apply Theorem 3.30
to the extension M : K to deduce that ((L: K)) = (M : K), since the
fixed field of ((L: K)) is K. Therefore the homomorphism : (L: K)
(M : K) induces the required isomorphism between (L: K)/(L: M ) and
(M : K).

3.12

Quadratic Polynomials

We consider the problem of expressing the roots of a polynomial of low degree


in terms of its coefficients. Then the well-known procedure for locating the
roots of a quadratic polynomial with real or complex coefficients generalizes
to quadratic polynomials with coefficients in a field K whose characteristic
does not equal 2. Given a quadratic polynomial ax2 + bx + c with coefficients
a and b belonging to some such field K, let us adjoin to K an element satisfying 2 = b2 4ac. Then the polynomial splits over K(), and its roots are
(b )/(2a). We shall describe below analogous procedures for expressing
the roots of cubic and quartic polynomials in terms of their coefficients.

3.13

Cubic Polynomials

Consider a cubic polynomial x3 + ax2 + bx + c, where the coefficients a, b and


c belong to some field K of characteristic zero. If f (x) = x3 + ax2 + bx + c
2 3
then f (x 31 a) = x3 px q, where p = 13 a2 b and q = 13 ba 27
a c. It
64

therefore suffices to restrict our attention to cubic polynomials of the form


x3 px q, where p and q belong to K.
Let f (x) = x3 px q, and let u and v be elements of some splitting
field for f over K. Then
f (u + v) = u3 + v 3 + (3uv p)(u + v) q.
Suppose that 3uv = p. Then f (u + v) = u3 + p3 /(27u3 ) q. Thus f (u +
p/(3u)) = 0 if and only if u3 is a root of the quadratic polynomial x2 xq +
p3 /27. Now the roots of this quadratic polynomial are
r
q
q2
p3

,
4
27
2
and the product of these roots is p3 /27. Thus if one of these roots is equal to
u3 then the other is equal to v 3 , where v = p/(3u). It follows that the roots
of the cubic polynomial f are
s
s
r
r
2
3
3 q
3 q
q
p
q2
p3
+

2
4
27
2
4
27
where the two cube roots must be chosen so as to ensure that their product
is equal to 13 p. It follows that the cubic polynomial x3 px q splits over the
1 3
field K(, , ), where 2 = 14 q 2 27
p and 3 = 12 q +  and where satisfies
3
= 1 and 6= 1. The roots of the polynomial in this extension field are ,
and , where
=+

p
,
3

= + 2

p
,
3

= 2 + 3

p
.
3

Now let us consider the possibilities for the Galois group (L: K), where
L is a splitting field for f over K. Now L = K(, , ), where , and
are the roots of f . Also a K-automorphism of L must permute the roots
of f amongst themselves, and it is determined by its action on these roots.
Therefore (L: K) is isomorphic to a subgroup of the symmetric group 3
(i.e., the group of permutations of a set of 3 objects), and thus the possibilities
for the order of (L: K) are 1, 2, 3 and 6. It follows from Corollary 3.16 that
f is irreducible over K if and only if the roots of f are distinct and the
Galois group acts transitively on the roots of f . By considering all possible
subgroups of 3 it is not difficult to see that f is irreducible over K if and
only if |(L: K)| = 3 or 6. If f splits over K then |(L: K)| = 1. If f factors
in K[x] as the product of a linear factor and an irreducible quadratic factor
then |(L: K)| = 2.
65

Let = ()()( ). Then 2 is invariant under any permutation


of and , and therefore 2 is fixed by all automorphisms in the Galois
group (L: K). Therefore 2 K. The element 2 of K is referred to as
the discriminant of the polynomial f . A straightforward calculation shows
that if f (x) = x3 px q then 2 = 4p3 27q 2 . Now changes sign under
any permutation of the roots , and that transposes two of the roots
whilst leaving the third root fixed. But K if and only if is fixed by all
elements of the Galois group (L: K), in which case the Galois group must
induce only cyclic permutations of the roots , and . Therefore (L: K)
is isomorphic to the cyclic group of order 3 if and only if f is irreducible
and the discriminant 4p3 27q 2 of f has a square root in the field K. If f
is irreducible but the discriminant does not have a square root in K then
(L: K) is isomorphic to the symmetric group 3 , and |(L: K)| = 6.

3.14

Quartic Polynomials

We now consider how to locate the roots of a quartic polynomial with coefficients in a field K of characteristic zero. A substitution of the form x 7 xc,
where c K, will reduce the problem to that of locating the roots , ,
and of a quartic polynomial f of the form f (x) = x4 px2 qx r in some
splitting field L.
Now the roots , , and of the quartic polynomial
x4 px2 qx r,
must satisfy the equation
(x )(x )(x )(x ) = x4 px2 qx r.
Equating coefficients of x, we find that
+ + + = 0,
and
p = ( + + + + + ),
q = + + + ,
r = .
Let
= ( + )( + ) = ( + )2 = ( + )2 ,
= ( + )( + ) = ( + )2 = ( + )2 ,
= ( + )( + ) = ( + )2 = ( + )2 .
66

We shall show that + + , + + and can all be expressed in


terms of p, q and r.
To do this we eliminate from the above expressions using the identity
+ + + = 0. We find
p =
=
q =
=
r =

( + + )( + + )
2 + 2 + 2 + + + ,
( + + )( + + )
( 2 + 2 + 2 + 2 + 2 + 2 ) 2,
2 + 2 + 2 .

Then


2
2
2
+ + = ( + ) + ( + ) + ( + )


= 2 2 + 2 + 2 + + +
2

+ +

p2

= 2p,
= ( + )4 + ( + )4 + ( + )4
= 4 + 4 3 + 6 2 2 + 4 3 + 4
+ 4 + 4 3 + 6 2 2 + 4 3 + 4
+ 4 + 4 3 + 6 2 2 + 4 3 + 4
= 2( 4 + 4 + 4 ) + 4( 3 + 3 + 3 + 3 + 3 + 3 )
+ 6( 2 2 + 2 2 + 2 2 ),
= 4 + 4 + 4 + 3( 2 2 + 2 2 + 2 2 )
+ 4( 2 + 2 + 2 )
+ 2( 3 + 3 + 3 + 3 + 3 + 3 ).

Therefore
2 + 2 + 2 = 2p2 8( 2 + 2 + 2 )
= 2p2 8r.
But
4p2 = ( + + )2 = 2 + 2 + 2 + 2( + + )
Therefore
+ + = 2p2 21 (2 + 2 + 2 )
= p2 + 4r.
67

Finally, we note that



2
= ( + )( + )( + ) .
Now
( + )( + )( + ) = 2 + 2 + 2 + 2 + 2 + 2 + 2
= q.
( + )( + )( + ) = ( + )( + )( + ) = q.
Therefore
= (q)2 = q 2 .
Thus , and are the roots of the resolvent cubic
x3 + 2px2 + (p2 + 4r)x + q 2 .

One can then verify that the roots of f take the form 12 ( + +

), where these square roots are chosen to ensure that =


q. (It should be noted that there are four possible ways in which the square
roots can be chosen to satisfy this condition; these yield all four roots of the
polynomial f .) We can therefore determine the roots of f in an appropriate
splitting field once we have expressed the quantities , and in terms of
the coefficients of the polynomial.
Remark Any permutation of the roots of the quartic
x4 px2 qx r,
will permute the roots , and of the resolvent cubic
g(x) = (x )(x )(x )
amongst themselves, and will therefore permute the factors of g. Therefore
the coefficients of g are fixed by all elements of the Galois group (L: K)
and therefore must belong to the ground field K. As we have seen from the
calculations above, these coefficients can be expressed in terms of p, q, r.

3.15

The Galois group of the polynomial x4 2

We shall apply the Galois correspondence to investigate the structure of the


splitting field for the polynomial x4 2 over the field Q of rational numbers.
68

A straightforward application of Eisensteins Irreducibility Criterion (Proposition 2.18) shows that the polynomial x4 2 is irreducible over Q. Let be
4
the unique positive real number satisfying 4 = 2. Then the roots of x
2
in the field C of complex numbers are , i, and i, where i = 1.
Thus if L = Q(, i) then L is a splitting field for the polynomial x4 2 over
Q.
Now the polynomial x4 2 is the minimum polynomial of over Q, since
this polynomial is irreducible. We can therefore apply Theorem 3.4 to deduce
that [Q(): Q] = 4. Now i does not belong to Q(), since Q() R. Therefore
the polynomial x2 + 1 is the minimum polynomial of i over Q(). Another
application of Theorem 3.4 now shows that [L: Q()] = [Q(, i): Q()] = 2. It
follows from the Tower Law (Theorem 3.1) that [L: Q] = [L: Q()][Q(): Q] =
8. Moreover the extension L: Q is a Galois extension, and therefore its Galois
group (L: Q) is a group of order 8 (Theorem 3.31).
Another application of the Tower Law now shows that [L: Q(i)] = 4,
since [L: Q] = [L: Q(i)][Q(i): Q] and [Q(i): Q] = 2. Therefore the minimum
polynomial of over Q(i) is a polynomial of degree 4 (Theorem 3.4). But is
a root of x4 2. Therefore x4 2 is irreducible over Q(i), and is the minimum
polynomial of over Q(i). Corollary 3.16 then ensures the existence of an
automorphism of L that sends L to i and fixes each element of Q(i).
Similarly there exists an automorphism of L that sends i to i and fixes
each element of Q(). (The automorphism is in fact the restriction to L
of the automorphism of C that sends each complex number to its complex
conjugate.)
Now the automorphisms , 2 , 3 and 4 fix i and therefore send to
i, , i and respectively. Therefore 4 = , where is the identity
automorphism of L. Similarly 2 = . Straightforward calculations show
that = 3 , and ( )2 = ( 2 )2 = ( 3 )2 = . It follows easily from this
that (L: Q) = {, , 2 , 3 , , , 2 , 3 }, and (L: Q) is isomorphic to the
dihedral group of order 8 (i.e., the group of symmetries of a square in the
plane).
The Galois correspondence is a bijective correspondence between the subgroups of (L: Q) and subfields of L that contain Q. The subfield of L corresponding to a given subgroup of (L: Q) is set of all elements of L that
are fixed by all the automorphisms in the subgroup. One can verify that
the correspondence between subgroups of (L: Q) and their fixed fields is as

69

follows:

3.16

Subgroup of (L: Q)

Fixed field

(L: K)
{, , 2 , 3 }
{, 2 , , 2 }
{, 2 , , 3 }
{, 2 }
{, }
{, 2 }
{, }
{, 3 }
{}

Q
Q(i)

Q( 2)
Q(i
2)
Q( 2, i)
Q()
Q(i)
Q((1 i)/)
Q((1 + i)/)
Q(, i)

The Galois group of a polynomial

Definition Let f be a polynomial with coefficients in some field K. The


Galois group K (f ) of f over K is defined to be the Galois group (L: K) of
the extension L: K, where L is some splitting field for the polynomial f over
K.
We recall that all splitting fields for a given polynomial over a field K
are K-isomorphic (see Theorem 3.15), and thus the Galois groups of these
splitting field extensions are isomorphic. The Galois group of the given polynomial over K is therefore well-defined (up to isomorphism of groups) and
does not depend on the choice of splitting field.
Lemma 3.36 Let f be a polynomial with coefficients in some field K and
let M be an extension field of K. Then M (f ) is isomorphic to a subgroup
of K (f ).
Proof Let N be a splitting field for f over M . Then N contains a splitting
field L for f over K. An element of (N : M ) is an automorphism of N
that fixes every element of M and therefore fixes every element of K. Its
restriction |L to L is then a K-automorphism of L (Corollary 3.34). Moreover ( )|L = (|L ) ( |L ) for all , (N : M ). Therefore there is a
group homomorphism from (N : M ) to (L: K) which sends an automorphism (N : M ) to its restriction |L to L.
Now if (N : M ) is in the kernel of this group homomorphism from
(N : M ) to (L: K) then |L must be the identity automorphism of L. But
f splits over L, and therefore all the roots of f are elements of L. It follows
that () = for each root of f . The fixed field of must therefore
be the whole of N , since M is contained in the fixed field of , and N is
70

a splitting field for f over M . Thus must be the identity automorphism


of N . We conclude therefore that the group homomorphism from (N : M )
to (L: K) sending (N : M ) to |L is injective, and therefore maps
(N : M ) isomorphically onto a subgroup of (L: K). The result therefore
follows from the definition of the Galois group of a polynomial.
Let f be a polynomial with coefficients in some field K and let the roots
of f is some splitting field L be 1 , 2 , . . . , n . An element of (L: K) is
a K-automorphism of L, and therefore permutes the roots of f . Moreover
two automorphism and in the Galois group (L: K) are equal if and only
if (j ) = (j ) for j = 1, 2, . . . , n, since L = K(1 , 2 , . . . , n ). Thus the
Galois group of a polynomial can be represented as a subgroup of the group
of permutations of its roots. We deduce immediately the following result.
Lemma 3.37 Let f be a polynomial with coefficients in some field K. Then
the Galois group of f over K is isomorphic to a subgroup of the symmetric
group n , where n is the degree of f .

3.17

Solvable polynomials and their Galois groups

Definition We say that a polynomial with coefficients in a given field is


solvable by radicals if the roots of the polynomial in a splitting field can be
constructed from its coefficients in a finite number of steps involving only the
operations of addition, subtraction, multiplication, division and extraction
of nth roots for appropriate natural numbers n.
It follows from the definition above that a polynomial with coefficients in
a field K is solvable by radicals if and only if there exist fields K0 , K1 , . . . , Km
such that K0 = K, the polynomial f splits over Km , and, for each integer i
between 1 and m, the field Ki is obtained on adjoining to Ki1 an element i
with the property that ipi Ki1 for some positive integer pi . Moreover we
can assume, without loss of generality that p1 , p2 , . . . , pm are prime numbers,
since an nth root of an element of a given field can be adjoined that field
by successively adjoining powers n1 , n2 , . . . , nk of chosen such that n/n1
is prime, ni /ni1 is prime for i = 2, 3, . . . , k, and nk = 1.
We shall prove that a polynomial with coefficients in a field K of characteristic zero is solvable by radicals if and only if its Galois group K (f ) over
K is a solvable group.
Let L be a field, and let p be a prime number that is not equal to the
characteristic of L. Suppose that the polynomial xp 1 splits over L. Then
the polynomial xp 1 has distinct roots, since its formal derivative pxp1 is
71

non-zero at each root of xp 1. An element of L is said to be a primitive


pth root of unity if p = 1 and 6= 1. The primitive pth roots of unity are
the roots of the polynomial xp1 +xp2 + +1, since xp 1 = (x 1)(xp1 +
xp2 + + 1). Also the group of pth roots of unity in L is a cyclic group
over order p which is generated by any primitive pth root of unity.
Lemma 3.38 Let K be a field, and let p be a prime number that is not
equal to the characteristic of K. If is a primitive pth root of unity in
some extension field of K then the Galois group of the extension K(): K is
Abelian.
Proof Let L = K(). Then L is a splitting field for the polynomial xp 1.
Let and be K-automorphisms of L. Then () and () are roots of
xp 1 (since the automorphisms and permute the roots of this polynomial)
and therefore there exist non-negative integers q and r such that () = q
and () = r . Then ( ()) = qr = (()). But there is at most one
K-automorphism of L sending to qr . It follows that = . Thus
the Galois group (L: K) is Abelian, as required.
Lemma 3.39 Let K be a field of characteristic zero and let M be a splitting
field for the polynomial xp c over K, where p is some prime number and
c K. Then the Galois group (M : K) of the extension M : K is solvable.
Proof The result is trivial when c = 0, since M = K in this case.
Suppose c 6= 0. The roots of the polynomial xp c are distinct, and each
pth root of unity is the ratio of two roots of xp c. Therefore M = K(, ),
where p = c and is some primitive pth root of unity. Now K(): K
is a normal extension, since K() is a splitting field for the polynomial
xp 1 over K (Theorem 3.17). On applying the Galois correspondence
(Theorem 3.35), we see that (M : K()) is a normal subgroup of (M : K),
and (M : K)/(M : K()) is isomorphic to (K(): K). But (K(): K) is
Abelian (Lemma 3.38). It therefore suffices to show that (M : K()) is also
Abelian.
Now the field M is obtained from K() by adjoining an element satisfying p = c. Therefore each automorphism in (M : K()) is uniquely
determined by the value of (). Moreover () is also a root of xp c, and
therefore () = j for some integer j. Thus if and are automorphisms
of M belonging to (M : K()), and if () = j and () = k , then
( ()) = (()) = j+k , since () = () = . Therefore = .
We deduce that (M : K()) is Abelian, and thus (M : K) is solvable, as
required.
72

Lemma 3.40 Let f be a polynomial with coefficients in a field K of characteristic zero, and let K 0 = K(), where K 0 satisfies p K for some
prime number p. Then K (f ) is solvable if and only if K 0 (f ) is solvable.
Proof Let N be a splitting field for the polynomial f (x)(xp c) over K,
where c = p . Then N contains a splitting field L for f over K and a
splitting field M for xp c over K. Then N : K, L: K and M : K are Galois
extensions. The Galois correspondence (Theorem 3.35) ensures that (N : L)
and (N : M ) are normal subgroups of (N : K). Moreover (L: K) is isomorphic to (N : K)/(N : L), and (M : K) is isomorphic to (N : K)/(N : M ).
Now M and N are splitting fields for the polynomial xp c over the fields K
and L respectively. It follows from Lemma 3.39 that (M : K) and (N : L)
are solvable. But if H is a normal subgroup of a finite group G then G is solvable if and only both H and G/H are solvable (Proposition 1.41). Therefore
(N : K) is solvable if and only if (N : M ) is solvable. Also (N : K) is solvable if and only if (L: K) is solvable. It follows that (N : M ) is solvable if
and only if (L: K) is solvable. But (N : M )
= M (f ) and (L: K)
= K (f ),
since L and N are splitting fields for f over K and M respectively. Thus
M (f ) is solvable if and only if K (f ) is solvable.
Now M is also a splitting field for the polynomial xp c over K 0 , since
0
K = K(), where is a root of the polynomial xp c. The above argument therefore shows that M (f ) is solvable if and only if K 0 (f ) is solvable.
Therefore K (f ) is solvable if and only if K 0 (f ) is solvable, as required.
Theorem 3.41 Let f be a polynomial with coefficients in a field K of characteristic zero. Suppose that f is solvable by radicals. Then the Galois group
K (f ) of f is a solvable group.
Proof The polynomial f is solvable by radicals. Therefore there exist fields
K0 , K1 , . . . , Km such that K0 = K, the polynomial f splits over Km , and, for
each integer i between 1 and m, the field Ki is obtained on adjoining to Ki1
an element i with the property that ipi Ki1 for some prime number pi .
Now Km (f ) is solvable, since it is the trivial group consisting of the identity
automorphism of Km only. Also Lemma 3.40 ensures that, for each i > 0,
Ki (f ) is solvable if and only if Ki1 (f ) is solvable. It follows that K (f ) is
solvable, as required.
Lemma 3.42 Let p be a prime number, let K be a field whose characteristic
is not equal to p, and let L: K be a Galois extension of K of degree p. Suppose
that the polynomial xp 1 splits over K. Then there exists L such that
L = K() and p K.
73

Proof The Galois group (L: K) is a cyclic group of order p, since its order is
equal to the degree p of the extension L: K. Let be a generator of (L: K),
let be an element of L \ K, and let
j = 0 + j 1 + 2j 2 + + (p1)j p1
for j = 0, 1, . . . , p 1, where 0 = , i = (i1 ) for i = 1, 2, . . . , p 1,
and is a primitive pth root of unity contained in K. Now (j ) = j j
for j = 0, 1, . . . , p 1, since () = , (p1 ) = 0 and p = 1. Therefore
(jp ) = jp and hence jp K for j = 0, 1, 2, . . . , p 1. But
0 + 1 + 2 + + p1 = p,
since j is a root of the polynomial 1 + x + x2 + + xp1 for all integers
j that are not divisible by p. Moreover p L \ K, since L \ K and
p 6= 0 in K. Therefore at least one of the elements 0 , 1 , . . . , p1 belongs
to L \ K. Let = j , where j L \ K. It follows from the Tower Law
(Theorem 3.1) that [K(), K] divides [L: K]. But [L: K] = p and p is prime.
It follows that L = K(). Moreover p K, as required.
Theorem 3.43 Let f be a polynomial with coefficients in a field K of characteristic zero. Suppose that the Galois group K (f ) of f over K is solvable.
Then f is solvable by radicals.
Proof Let be a primitive pth root of unity. Then K() (f ) is isomorphic
to a subgroup of K (f ) (Lemma 3.36) and is therefore solvable (Proposition 1.41). Moreover f is solvable by radicals over K if and only if f is
solvable by radicals over K(), since K() is obtained from K by adjoining
an element whose pth power belongs to K. We may therefore assume,
without loss of generality, that K contains a primitive pth root of unity for
each prime p that divides |K (f )|.
The result is trivial when |K (f )| = 1, since in that case the polynomial f
splits over K. We prove the result by induction on the degree |K (f )| of the
Galois group. Thus suppose that the result holds when the order of the Galois
group is less than |K (f )|. Let L be a splitting field for f over K. Then L: K
is a Galois extension and (L: K)
= K (f ). Now the solvable group (L: K)
contains a normal subgroup H for which the corresponding quotient group
(L: K)/H is a cyclic group of order p for some prime number p dividing
|(L: K)|. Let M be the fixed field of H. Then (L: M ) = H and (M : K)
=
(L: K)/H. (Theorem 3.35), and therefore [M : K] = |(L: K)/H| = p. It
follows from Lemma 3.42 that M = K() for some element M satisfying
p K. Moreover M (f )
= H, and H is solvable, since any subgroup of
74

a solvable group is solvable (Proposition 1.41). The induction hypothesis


ensures that f is solvable by radicals when considered as a polynomial with
coefficients in M , and therefore the roots of f lie in some extension field of
M obtained by successively adjoining radicals. But M is obtained from K by
adjoining the radical . Therefore f is solvable by radicals, when considered
as a polynomial with coefficients in K, as required.
On combining Theorem 3.41 and Theorem 3.43, we see that a polynomial
with coefficients in a field K of characteristic zero is solvable by radicals if
and only if its Galois group K (f ) over K is a solvable group.

3.18

A quintic polynomial that is not solvable by radicals

Lemma 3.44 Let p be a prime number and let f be a polynomial of order p


with rational coefficients. Suppose that f has exactly p 2 real roots and is
irreducible over the field Q of rational numbers. Then the Galois group of f
over Q is isomorphic to the symmetric group p .
Proof If is a root of f then [Q(): Q] = p since f is irreducible and
deg f = p (Theorem 3.4). Thus if L is a splitting field extension for f over
Q then [L: Q] = [L: Q()][Q(): Q] by the Tower Law (Proposition 3.1) and
therefore [L: Q] is divisible by p. But [L: Q] is the order of the Galois group G
of f , and therefore |G| is divisible by p. It follows from a basic theorem of
Cauchy that G must contain at least one element of order p. Moreover an
element of G is determined by its action on the roots of f . Thus an element
of G is of order p if and only if it cyclically permutes the roots of f .
The irreducibility of f ensures that f has distinct roots (Corollary 3.20).
Let 1 and 2 be the two roots of f that are not real. Then 1 and 2 are
complex conjugates of one another, since f has real coefficients. We have
already seen that G contains an element of order p which cyclically permutes
the roots of f . On taking an appropriate power of this element, we obtain
an element of G that cyclically permutes the roots of f and sends 1 to
2 . We label the real roots 3 , 4 , . . . , p of f so that j = (j1 ) for
j = 2, 3, 4, . . . , p. Then (p ) = 1 . Now complex conjugation restricts to a
Q-automorphism of L that interchanges 1 and 2 but fixes j for j > 2.
But if 2 j p then 1j j1 transposes the roots j1 and j and fixes
the remaining roots. But transpositions of this form generate the whole of
the group of permutations of the roots. Therefore every permutation of the
roots of f is realised by some element of the Galois group G of f , and thus
G
= p , as required.
75

Example Consider the quintic polynomial f where f (x) = x5 6x + 3.


Eisensteins Irreducibility Criterion (Proposition 2.18) can be used to show
that f is irreducible over Q. Now f (2) = 17, f (1) = 8, f (1) = 2
and f (2) = 23. The Intermediate Value Theorem ensures that f has at
least 3 distinct real roots. If f had at least 4 distinct real roots then Rolles
Theorem would ensure that the number of distinct real roots of f 0 and f 00
would be at least 3 and 2 respectively. But zero is the only root of f 00 since
f 00 (x) = 20x3 . Therefore f must have exactly 3 distinct real roots. It follows
from Lemma 3.44 that the Galois group of f is isomorphic to the symmetric
group 5 . This group is not solvable. Theorem 3.41 then ensures that the
polynomial f is not solvable by radicals over the field of rational numbers.
The above example demonstrates that there cannot exist any general
formula for obtaining the roots of a quintic polynomial from its coefficients in
a finite number of steps involving only addition, subtraction, multiplication,
division and the extraction of nth roots. For if such a general formula were
to exist then every quintic polynomial with rational coefficients would be
solvable by radicals.

76

Commutative Algebra and Algebraic Geometry

4.1

Modules

Definition Let R be a unital commutative ring. A set M is said to be a


module over R (or R-module) if
(i) given any x, y M and r R, there are well-defined elements x + y
and rx of M ,
(ii) M is an Abelian group with respect to the operation + of addition,
(iii) the identities
r(x + y) = rx + ry,
(rs)x = r(sx),

(r + s)x = rx + sx,
1x = x

are satisfied for all x, y M and r, s R.


Example If K is a field, then a K-module is by definition a vector space
over K.
Example Let (M, +) be an Abelian group, and let x M . If n is a positive
integer then we define nx to be the sum x + x + + x of n copies of x. If
n is a negative integer then we define nx = (|n|x), and we define 0x = 0.
This enables us to regard any Abelian group as a module over the ring Z of
integers. Conversely, any module over Z is also an Abelian group.
Example Any unital commutative ring can be regarded as a module over
itself in the obvious fashion.
Let R be a unital commutative ring, and let M be an R-module. A
subset L of M is said to be a submodule of M if x + y L and rx L for
all x, y L and r R. If M is an R-module and L is a submodule of M
then the quotient group M/L can itself be regarded as an R-module, where
r(L + x) L + rx for all L + x M/L and r R. The R-module M/L is
referred to as the quotient of the module M by the submodule L.
Note that a subset I of a unital commutative ring R is a submodule of R
if and only if I is an ideal of R.
Let M and N be modules over some unital commutative group R. A
function : M N is said to be a homomorphism of R-modules if (x+y) =
77

(x)+(y) and (rx) = r(x) for all x, y M and r R. A homomorphism


of R-modules is said to be an isomorphism if it is invertible. The kernel
ker and image (M ) of any homomorphism : M N are themselves Rmodules. Moreover if : M N is a homomorphism of R-modules, and if L
is a submodule of M satisfying L ker , then induces a homomorphism
: M/L N . This induced homomorphism is an isomorphism if and only if
L = ker and N = (M ).
Definition Let M1 , M2 , . . . , Mk be modules over a unital commutative ring
R. The direct sum M1 M2 Mk is defined to be the set of ordered
k-tuples (x1 , x2 , . . . , xk ), where xi Mi for i = 1, 2, . . . , k. This direct sum
is itself an R-module:
(x1 , x2 , . . . , xk ) + (y1 , y2 , . . . , yk ) = (x1 + y1 , x2 + y2 , . . . , xk + yk ),
r(x1 , x2 , . . . , xk ) = (rx1 , rx2 , . . . , rxk )
for all xi , yi Mi and r R.
If K is any field, then K n is the direct sum of n copies of K.
Definition Let M be a module over some unital commutative ring R. Given
any subset X of M , the submodule of M generated by the set X is defined
to be the intersection of all submodules of M that contain the set X. It
is therefore the smallest submodule of M that contains the set X. An Rmodule M is said to be finitely-generated if it is generated by some finite
subset of itself.
Lemma 4.1 Let M be a module over some unital commutative ring R, and
let {x1 , x2 , . . . , xk } be a finite subset of M . Then the submodule of M generated by this set consists of all elements of M that are of the form
r1 x 1 + r2 x 2 + + rk x k
for some r1 , r2 , . . . , rk R.
Proof The subset of M consisting of all elements of M of this form is clearly
a submodule of M . Moreover it is contained in every submodule of M that
contains the set {x1 , x2 , . . . , xk }. The result follows.

78

4.2

Noetherian Modules

Definition Let R be a unital commutative ring. An R-module M is said to


be Noetherian if every submodule of M is finitely-generated.
Proposition 4.2 Let R be a unital commutative ring, and let M be a module
over R. Then the following are equivalent:
(i) (Ascending Chain Condition) if L1 L2 L3 is an ascending
chain of submodules of M then there exists an integer N such that
Ln = LN for all n N ;
(ii) (Maximal Condition) every non-empty collection of submodules of M
has a maximal element (i.e., an submodule which is not contained in
any other submodule belonging to the collection);
(iii) (Finite Basis Condition) M is a Noetherian R-module (i.e., every submodule of M is finitely-generated).
Proof Suppose that M satisfies the Ascending Chain Condition. Let C be
a non-empty collection of submodules of M . Choose L1 C. If C were to
contain no maximal element then we could choose, by induction on n, an
ascending chain L1 L2 L3 of submodules belonging to C such that
Ln 6= Ln+1 for all n, which would contradict the Ascending Chain Condition.
Thus M must satisfy the Maximal Condition.
Next suppose that M satisfies the Maximal Condition. Let L be an submodule of M , and let C be the collection of all finitely-generated submodules
of M that are contained in L. Now the zero submodule {0} belongs to C,
hence C contains a maximal element J, and J is generated by some finite
subset {a1 , a2 , . . . , ak } of M . Let x L, and let K be the submodule generated by {x, a1 , a2 , . . . , ak }. Then K C, and J K. It follows from the
maximality of J that J = K, and thus x J. Therefore J = L, and thus L
is finitely-generated. Thus M must satisfy the Finite Basis Condition.
Finally suppose that M satisfies the Finite Basis Condition. Let L1
L2 L3 be an ascending chain of submodules of M , and let L be the
+
S
union
Ln of the submodules Ln . Then L is itself an submodule of M .
n=1

Indeed if a and b are elements of L then a and b both belong to Ln for some
sufficiently large n, and hence a + b, a and ra belong to Ln , and thus to L,
for all r M . But the submodule L is finitely-generated. Let {a1 , a2 , . . . , ak }
be a generating set of L. Choose N large enough to ensure that ai LN for
i = 1, 2, . . . , k. Then L LN , and hence LN = Ln = L for all n N . Thus
M must satisfy the Ascending Chain Condition, as required.
79

Proposition 4.3 Let R be a unital commutative ring, let M be an R-module,


and let L be a submodule of M . Then M is Noetherian if and only if L and
M/L are Noetherian.
Proof Suppose that the R-module M is Noetherian. Then the submodule L
is also Noetherian, since any submodule of L is also a submodule of M and
is therefore finitely-generated. Also any submodule K of M/L is of the form
{L + x : x J} for some submodule J of M satisfying L J. But J
is finitely-generated (since M is Noetherian). Let x1 , x2 , . . . , xk be a finite
generating set for J. Then
L + x1 , L + x2 , . . . , L + xk
is a finite generating set for K. Thus M/L is Noetherian.
Conversely, suppose that L and M/L are Noetherian. We must show that
M is Noetherian. Let J be any submodule of M , and let (J) be the image of
J under the quotient homomorphism : M M/L, where (x) = L + x for
all x M . Then (J) is a submodule of the Noetherian module M/L and is
therefore finitely-generated. It follows that there exist elements x1 , x2 , . . . , xk
of J such that (J) is generated by
L + x1 , L + x2 , . . . , L + xk .
Also J L is a submodule of the Noetherian module L, and therefore there
exists a finite generating set y1 , y2 , . . . , ym for J L. We claim that
{x1 , x2 , . . . , xk , y1 , y2 , . . . , ym }
is a generating set for J.
Let z J. Then there exist r1 , r2 , . . . , rk R such that
(z) = r1 (L + x1 ) + r2 (L + x2 ) + + rk (L + xk ) = L + r1 x1 + r2 x2 + + rk xk .
But then z (r1 x1 +r2 x2 + +rk xk ) J L (since L = ker ), and therefore
there exist s1 , s2 , . . . , sm such that
z (r1 x1 + r2 x2 + + rk xk ) = s1 y1 + s2 y2 + + sm ym ,
and thus
z=

k
X

ri x i +

i=1

m
X

si y i .

j=1

This shows that the submodule J of M is finitely-generated. We deduce that


M is Noetherian, as required.
80

Corollary 4.4 The direct sum M1 M2 Mk of Noetherian modules


M1 , M2 , . . . Nk over some unital commutative ring R is itself a Noetherian
module over R.
Proof The result follows easily by induction on k once it has been proved
in the case k = 2.
Let M1 and M2 be Noetherian R-modules. Then M1 {0} is a Noetherian
submodule of M1 M2 isomorphic to M1 , and the quotient of M1 M2 by
this submodule is a Noetherian R-module isomorphic to M2 . It follows from
Proposition 4.3 that M1 M2 is Noetherian, as required.
One can define also the concept of a module over a non-commutative
ring. Let R be a unital ring (not necessarily commutative), and let M be an
Abelian group. We say that M is a left R-module if each r R and m M
determine an element rm of M , and the identities
r(x + y) = rx + ry,

(r + s)x = rx + sx,

(rs)x = r(sx),

1x = x

are satisfied for all x, y M and r, s R. Similarly we say that M is a right


R-module if each r R and m M determine an element mr of M , and the
identities
(x + y)r = xr + yr,

x(r + s) = xr + xs,

x(rs) = (xr)s,

x1 = x

are satisfied for all x, y M and r, s R. (If R is commutative then the


distinction between left R-modules and right R-modules is simply a question
of notation; this is not the case if R is non-commutative.)

4.3

Noetherian Rings and Hilberts Basis Theorem

Let R be a unital commutative ring. We can regard the ring R as an Rmodule, where the ring R acts on itself by left multiplication (so that r . r0
is the product rr0 of r and r0 for all elements r and r0 of R). We then find
that a subset of R is an ideal of R if and only if it is a submodule of R. The
following result therefore follows directly from Proposition 4.2.
Proposition 4.5 Let R be a unital commutative ring. Then the following
are equivalent:
(i) (Ascending Chain Condition) if I1 I2 I3 is an ascending
chain of ideals of R then there exists an integer N such that In = IN
for all n N ;
81

(ii) (Maximal Condition) every non-empty collection of ideals of R has a


maximal element (i.e., an ideal which is not contained in any other
ideal belonging to the collection);
(iii) (Finite Basis Condition) every ideal of R is finitely-generated.
Definition A unital commutative ring is said to be a Noetherian ring if every
ideal of the ring is finitely-generated. A Noetherian domain is a Noetherian
ring that is also an integral domain.
Note that a unital commutative ring R is Noetherian if it satisfies any
one of the conditions of Proposition 4.5.
Corollary 4.6 Let M be a finitely-generated module over a Noetherian ring
R. Then M is a Noetherian R-module.
Proof Let {x1 , x2 , . . . , xk } be a finite generating set for M . Let Rk be the
direct sum of k copies of R, and let : Rk M be the homomorphism of
R-modules sending (r1 , r2 , . . . , rk ) Rk to
r1 x 1 + r2 x 2 + + rk x k .
It follows from Corollary 4.4 that Rk is a Noetherian R-module (since the
Noetherian ring R is itself a Noetherian R-module). Moreover M is isomorphic to Rk / ker , since : Rk M is surjective. It follows from Proposition 4.3 that M is Noetherian, as required.
If I is a proper ideal of a Noetherian ring R then the collection of all
proper ideals of R that contain the ideal I is clearly non-empty (since I
itself belongs to the collection). It follows immediately from the Maximal
Condition that I is contained in some maximal ideal of R.
Lemma 4.7 Let R be a Noetherian ring, and let I be an ideal of R. Then
the quotient ring R/I is Noetherian.
Proof Let L be an ideal of R/I, and let J = {x R : I + x L}. Then J
is an ideal of R, and therefore there exists a finite subset {a1 , a2 , . . . , ak } of
J which generates J. But then L is generated by I + ai for i = 1, 2, . . . , k.
Indeed every element of L is of the form I + x for some x J, and if
x = r1 a1 + r2 a2 + + rk ak
, where r1 , r2 , . . . , rk R, then
I + x = r1 (I + a1 ) + r2 (I + a2 ) + + rk (I + ak ),
as required.
82

Hilbert showed that if R is a field or is the ring Z of integers, then every


ideal of R[x1 , x2 , . . . , xn ] is finitely-generated. The method that Hilbert used
to prove this result can be generalized to yield the following theorem.
Theorem 4.8 (Hilberts Basis Theorem) If R is a Noetherian ring, then so
is the polynomial ring R[x].
Proof Let I be an ideal of R[x], and, for each non-negative integer n, let
In denote the subset of R consisting of those elements of R that occur as
leading coefficients of polynomials of degree n belonging to I, together with
the zero element of R. Then In is an ideal of R. Moreover In In+1 , for if
p(x) is a polynomial of degree n belonging to I then xp(x) is a polynomial of
degree n+1 belonging to I which has the same leading coefficient. Thus I0
I1 I2 is an ascending chain of ideals of R. But the Noetherian ring
R satisfies the Ascending Chain Condition (see Proposition 4.5). Therefore
there exists some natural number m such that In = Im for all n m.
Now each ideal In is finitely-generated, hence, for each n m, we can
choose a finite set {an,1 , an,2 , . . . , an,kn } which generates In . Moreover each
generator an,i is the leading coefficient of some polynomial qn,i of degree n
belonging to I. Let J be the ideal of R[x] generated by the polynomials qn,i
for all 0 n m and 1 i kn . Then J is finitely-generated. We shall
show by induction on deg p that every polynomial p belonging to I must
belong to J, and thus I = J. Now if p I and deg p = 0 then p is a constant
polynomial whose value belongs to I0 (by definition of I0 ), and thus p is a
linear combination of the constant polynomials q0,i (since the values a0,i of
the constant polynomials q0,i generate I0 ), showing that p J. Thus the
result holds for all p I of degree 0.
Now suppose that p I is a polynomial of degree n and that the result
is true for all polynomials p in I of degree less than n. Consider first the
case when n m. Let b be the leading coefficient of p. Then there exist
c1 , c2 , . . . , ckn R such that
b = c1 an,1 + c2 an,2 + + ckn an,kn ,
since an,1 , an,2 , . . . , an,kn generate the ideal In of R. Then
p(x) = c1 qn,1 (x) + c2 qn,2 (x) + + ck qn,k (x) + r(x),
where r I and deg r < deg p. It follows from the induction hypothesis that
r J. But then p J. This proves the result for all polynomials p in I
satisfying deg p m.
Finally suppose that p I is a polynomial of degree n where n > m, and
that the result has been verified for all polynomials of degree less than n.
83

Then the leading coefficient b of p belongs to In . But In = Im , since n m.


As before, we see that there exist c1 , c2 , . . . , ckm R such that
b = c1 am,1 + c2 am,2 + + ckn am,km ,
since am,1 , am,2 , . . . , am,km generate the ideal In of R. Then
p(x) = c1 xnm qm,1 (x) + c2 xnm qm,2 (x) + + ck xnm qm,k (x) + r(x),
where r I and deg r < deg p. It follows from the induction hypothesis that
r J. But then p J. This proves the result for all polynomials p in I
satisfying deg p > m. Therefore I = J, and thus I is finitely-generated, as
required.
Theorem 4.9 Let R be a Noetherian ring. Then the ring R[x1 , x2 , . . . , xn ]
of polynomials in the indeterminates x1 , x2 , . . . , xn with coefficients in R is a
Noetherian ring.
Proof It is easy to see to see that R[x1 , x2 , . . . , xn ] is naturally isomorphic
to R[x1 , x2 , . . . , xn1 ][xn ] when n > 1. (Any polynomial in the indeterminates x1 , x2 , . . . , xn with coefficients in the ring R may be viewed as a
polynomial in the indeterminate xn with coefficients in the polynomial ring
R[x1 , x2 , . . . , xn1 ].) The required results therefore follows from Hilberts
Basis Theorem (Theorem 4.8) by induction on n.
Corollary 4.10 Let K be a field. Then every ideal of the polynomial ring
K[x1 , x2 , . . . , xn ] is finitely-generated.

4.4

Polynomial Rings in Several Variables

A monomial in the independent indeterminates X1 , X2 , . . . , Xn is by definition an expression of the form X1i1 X2i2 Xnin , where i1 , i2 , . . . , in are nonnegative integers. Such monomials are multiplied according to the rule


X1i1 X2i2 Xnin X1j1 X2j2 Xnjn = X1i1 +j1 X2i2 +j2 Xnin +jn .
A polynomial p in the independent indeterminates with coefficients in some
ring R is by definition a formal linear combination of the form
r1 m 1 + r2 m 2 + + rk m k
where r1 , r2 , . . . , rk R and m1 , m2 , . . . , mk are monomials in X1 , X2 , . . . , Xn .
The coefficients r1 , r2 , . . . , rk of this polynomial are uniquely determined,
84

provided that the monomials m1 , m2 , . . . , mk are distinct. Such polynomials


are added and multiplied together in the obvious fashion. In particular
!
!
k
l
k X
l
X
X
X
ri m i
sj m0j =
(ri sj )(mi m0j ),
i=1

j=1

i=1 j=1

where the product mi m0j of the monomials mi and m0j is defined as described above. The set of all polynomials in the independent indeterminates
X1 , X2 , . . . , Xn with coefficients in the ring R is itself a ring, which we denote
by R[X1 , X2 , . . . , Xn ].
Example The polynomial 2X1 X23 6X1 X2 X32 is the product of the polynomials 2X1 X2 and X22 3X32 in the ring Z[X1 , X2 , X3 ] of polynomials in
X1 , X2 , X3 with integer coefficients.
Lemma 4.11 Let R be an integral domain. Then the ring R[x] of polynomials in the indeterminate x with coefficients in R is itself an integral domain,
and deg(pq) = deg p + deg q for all non-zero polynomials p, q R[x].
Proof The integral domain R is commutative, hence so is R[x]. Moreover
R[x] is unital, and the multiplicative identity element of R[x] is the constant
polynomial whose coefficient is the multiplicative identity element 1 of the
unital ring R.
Let p and q be polynomials in R[x], and let ak and bl be the leading
coefficients of p and q respectively, where k = deg p and l = deg q. Now
p(x)q(x) = ak bl xk+l + terms of lower degree.
Moreover ak bl 6= 0, since ak 6= 0, bl 6= 0, and the ring R of coefficients is an
integral domain. Thus if p 6= 0 and q 6= 0 then pq 6= 0, showing that R[x] is
an integral domain, and deg(pq) = k + l = deg p + deg q, as required.
Let p be a polynomial in the indeterminates X1 , X2 , . . . , Xn with coefficients in the ring R, where n > 1. By collecting together terms involving Xnj
for each non-negative integer j, we can write the polynomial p in the form
p(X1 , X2 , . . . , Xn ) =

k
X

pj (X1 , X2 , . . . , Xn1 )Xnj

j=0

where pj R[X1 , X2 , . . . , Xn1 ] for j = 0, 1, . . . , k. Now the right hand side


of the above identity can be viewed as a polynomial in the indeterminate Xn
with coefficients p1 , p2 , . . . , pk in the ring R[X1 , . . . , Xn1 ]. Moreover the
85

polynomial p uniquely determines and is uniquely determined by the polynomials p1 , p2 , . . . , pk . It follows from this that the rings R[X1 , X2 , . . . , Xn ] and
R[X1 , X2 , . . . , Xn1 ][Xn ] are naturally isomorphic and can be identified with
one another. We can use the identification in order to prove results concerning the structure of the polynomial ring R[X1 , X2 , . . . , Xn ] by induction on
the number n of independent indeterminates X1 , X2 , . . . , Xn . For example,
the following result follows directly by induction on n, using Lemma 4.11.
Lemma 4.12 Let R be an integral domain. Then the ring R[X1 , X2 , . . . , Xn ]
is also an integral domain.
A monomial X1i1 X2i2 Xnin is said to be of degree d, where d is some
non-negative integer, if i1 + i2 + + in = d.
Definition Let R be a ring. A polynomial p R[X1 , X2 , . . . , Xn ] is said to
be homogeneous of degree d if it can be expressed as a linear combination of
monomials of degree d with coefficients in the ring R.
Any polynomial p R[X1 , X2 , . . . , Xn ] can be decomposed as a sum of
the form
p(0) + p(1) + + p(k) ,
where k is some sufficiently large non-negative integer and each polynomial
p(i) is a homogeneous polynomial of degree i. The homogeneous polynomial
p(i) is referred to as the homogeneous component of p of degree i; it is uniquely
determined by p. A non-zero polynomial p is said to be of degree d if p(d) 6= 0
and p(i) = 0 for all i > d. The degree of a non-zero polynomial p is denoted
by deg p.
Lemma 4.13 Let R be a ring, and let p and q be non-zero polynomials
belonging to R[X1 , X2 , . . . , Xn ]. Then
deg(p + q) max(deg p, deg q), provided that p + q 6= 0,
deg(pq) deg p + deg q, provided that pq 6= 0.
Moreover if R is an integral domain then pq 6= 0 and deg(pq) = deg p + deg q.
Proof The inequality (p + q) max(deg p, deg q) is obvious. Also p(i) q (j) is
homogeneous of degree i + j for all i and j, since the product of a monomial
of degree i and a monomial of degree j is a monomial of degree i + j. The
inequality deg(pq) deg p + deg q follows immediately.
Now suppose that R is an integral domain. Let k = deg p and l = deg q.
Then the homogeneous component (pq)(k+l) of pq of degree k + l is given
by (pq)(k+l) = p(k) q (l) . But R[X1 , X2 , . . . , Xn ] is an integral domain (see
Lemma 4.12), and p(k) and q (l) are both non-zero. It follows that (pq)(k+l) 6= 0,
and thus deg(pq) = deg p + deg q, as required.
86

4.5

Algebraic Sets and the Zariski Topology

Throughout this section, let K be a field.


Definition We define affine n-space An over the field K to be the set K n
of all n-tuples (x1 , x2 , . . . , xn ) with x1 , x2 , . . . , xn K.
Where it is necessary to specify explicitly the field K involved, we shall
denote affine n-space over the field K by An (K). Thus An (R) = Rn , and
An (C) = Cn .
Definition A subset of n-dimensional affine space An is said to be an algebraic set if it is of the form
{(x1 , x2 , . . . , xn ) An : f (x1 , x2 , . . . , xn ) = 0 for all f S}
for some subset S of the polynomial ring K[X1 , X2 , . . . , Xn ].
Example Any point of An is an algebraic set. Indeed, given any point
(a1 , a2 , . . . , an ) of An , let fi (X1 , X2 , . . . , Xn ) = Xi ai for i = 1, 2, . . . , n.
Then the given point is equal to the set
{(x1 , x2 , . . . , xn ) An : fi (x1 , x2 , . . . , xn ) = 0 for i = 1, 2, . . . , n}.
Example The circle {(x, y) A2 (R) : x2 + y 2 = 1} is an algebraic set in the
plane A2 (R).
Let : K n K be a linear functional on the vector space K n (i.e., a linear
transformation from K n to K). It follows from elementary linear algebra that
there exist b1 , b2 , . . . , bn K such that
(x1 , x2 , . . . , xn ) = b1 x1 + b2 x2 + + bn xn
for all (x1 , x2 , . . . , xn ) K n . Thus if 1 , 2 , . . . , k are linear functionals on
K n , and if c1 , c2 , . . . , ck are suitable constants belonging to the field K then
{(x1 , x2 . . . , xn ) An : i (x1 , x2 , . . . , xn ) = ci for i = 1, 2, . . . , k}
is an algebraic set in An . A set of this type is referred to as an affine subspace
of An . It is said to be of dimension n k, provided that the linear functionals
1 , 2 , . . . , k are linearly independent. It follows directly from elementary
linear algebra that, if we we identify affine n-space An with the vector space
K n , then a subset of An is an m-dimensional affine subspace if and only if it
is a translate of some m-dimensional vector subspace of K n (i.e., it is of the
form v + W where v is a point of An and W is some m-dimensional vector
subspace of K n ).
87

Lemma 4.14 Let V be an algebraic set in An , and let L be a one-dimensional affine subspace of An . Then either L V or else L V is a finite
set.
Proof The affine subspace L is a translate of a one-dimensional subspace
of K n , and therefore there exist vectors v and w in K n such that L =
{v + wt : t K} (on identifying n-dimensional affine space An with the
vector space K n ). Now we can write
V = {(x1 , x2 , . . . , xn ) An : f (x1 , x2 , . . . , xn ) = 0 for all f S},
where S is some subset of the polynomial ring K[X1 , X2 , . . . , Xn ]. Now either
each polynomial belonging to S is zero throughout L, in which case L V ,
or else there is some f S which is non-zero at some point of L. Define
g K[t] by the formula
g(t) = f (v1 + w1 t, v2 + w2 t, . . . , vn + wn t)
(where vi and wi denote the ith components of the vectors v and w for
i = 1, 2, . . . , n). Then g is a non-zero polynomial in the indeterminate t, and
therefore g has at most finitely many zeros. But g(t) = 0 whenever the point
v + wt of L lies in V . Therefore L V is finite, as required.
Example The sets
{(x, y) A2 (R) : y = sin x}
and
{(x, y) A2 (R) : x 0}
are not algebraic sets in A2 (R), since the line y = 0 is not contained in either
of these sets, yet the line intersects these sets at infinitely many points of the
set.
Given any subset S of K[X1 , X2 , . . . , Xn ], we denote by V (S) the algebraic
set in An defined by
V (S) = {x An : f (x) = 0 for all f S}.
Also, given any f K[X1 , X2 , . . . , Xn ], we define V (f ) = V ({f }).
Given any subset Z of An , we define
I(Z) = {f K[X1 , X2 , . . . , Xn ] : f (x) = 0 for all x Z}.
Clearly S I(V (S)) for all subsets S of K[X1 , X2 , . . . , Xn ], and Z
V (I(Z)) for all subsets Z of An . If S1 and S2 are subsets of K[X1 , X2 , . . . , Xn ]
satisfying S1 S2 then V (S2 ) V (S1 ). Similarly, if Z1 and Z2 are subsets
of An satisfying Z1 Z2 then I(Z2 ) I(Z1 ).
88

Lemma 4.15 V (I(V (S))) = V (S) for all subsets S of K[X1 , X2 , . . . , Xn ],


and similarly I(V (I(Z))) = I(Z) for all subsets Z of An .
Proof It follows from the observations above that V (S) V (I(V (S))),
since Z V (I(Z)) for all subsets Z of An . But also S I(V (S)), and
hence V (I(V (S))) V (S). Therefore V (I(V (S))) = V (S). An analogous
argument can be used to show that I(V (I(Z))) = I(Z) for all subsets Z of
An .
Let I and J be ideals of a unital commutative ring R. We denote by
IJ the ideal of R consisting of those elements of R that can be expressed
as finite sums of the form i1 j1 + i2 j2 + + ir jr with i1 , i2 , . . . , ir I and
j1 , j2 , . . . , jr J. (One can readily verify that IJ is indeed an ideal of R.)
Proposition 4.16 Let R = K[X1 , X2 , . . . , Xn ] for some field K. Then
(i) V ({0}) = An and V (R) = ;

T
P
(ii) V (I ) = V
I for every collection {I : } of ideals
of R;
(iii) V (I) V (J) = V (I J) = V (IJ) for all ideals I and J of R.
Thus there is a well-defined topology on An (known as the Zariski topology)
whose closed sets are the algebraic sets in An .
Proof (i) is immediate.P

P
I
,
and
therefore
V
I
If

then
I


T
T V (I ). Thus
P
V
I
V (I ). Conversely if x is a point of
V (I ) then
fP(x) = 0 for allT and f P
I , andtherefore f (x) = T
0 for all f

V (I ) V
I . It follows that
V (I ) =

P I . Thus
V
I . This proves (ii).
Let I and J be ideals of R. Then I J I, I J J and IJ I J, and
thus V (I) V (I J), V (J) V (I J) and V (I J) V (IJ). Therefore
V (I) V (J) V (I J) V (IJ).
If x is a point of An which does not belong to V (I) V (J) then there exist
polynomials f I and g J such that f (x) 6= 0 and g(x) 6= 0. But
then f g IJ and f (x)g(x) 6= 0, and therefore x 6 V (IJ). Therefore
V (IJ) V (I) V (J). We conclude that
V (I) V (J) = V (I J) = V (IJ).
89

This proves (iii).


Let us define a topology on An whose open sets in An are the complements
of algebraic sets. We see from (i) that and An are open. Moreover it follows
from (ii) that any union of open sets is open, and it follows from (iii), using
induction on the number of sets, that any finite intersection of open sets is
open. Thus the topology is well-defined.
Definition The Zariski topology on an algebraic set V in An is the topology
whose open sets are of the form V \V (I) for some ideal I of K[X1 , X2 , . . . , Xn ].
It follows from Proposition 4.16 that the Zariski topology on an algebraic
set V is well-defined and is the subspace topology on V induced by the
topology on An whose closed sets are the algebraic sets in An . Moreover a
subset V1 of V is closed if and only if V1 is itself an algebraic set. (This
follows directly from the fact that the intersection of two algebraic sets is
itself an algebraic set.)
Example Any finite subset of An is an algebraic set. This follows from the
fact that any point in An is an algebraic set, and any finite union of algebraic
sets is an algebraic set.
In general, the Zariski topology on an algebraic set V is not Hausdorff.
It can in fact be shown that an algebraic set in An is Hausdorff (with respect
to the Zariski topology) if and only if it consists of a finite set of points in
An .

4.6

The Structure of Algebraic Sets

Let K be a field. We shall apply Hilberts Basis Theorem in order to study the
structure of algebraic sets in n-dimensional affine space An over the field K.
We shall continue to use the notation for algebraic sets in An and corresponding ideals of the polynomial ring that was established earlier.
The following result is a direct consequence of the Hilbert Basis Theorem.
Proposition 4.17 Let V be an algebraic set in An . Then there exists a finite
collection f1 , f2 , f3 , . . . of polynomials in n independent indeterminates such
that
V = {x An : fi (x) = 0 for i = 1, 2, . . . , k}.
Proof The set V is an algebraic set, and therefore V = V (I) for some
ideal I of K[X1 , X2 , . . . , Xn ]. Moreover it follows from Corollary 4.10 that
I is generated by some finite set {f1 , f2 , . . . , fk } of polynomials. But then
V = V ({f1 , f2 , . . . , fk }), and thus V is of the required form.
90

A algebraic hypersurface in An is a algebraic set of An of the form V (f )


for some non-constant polynomial f K[X1 , X2 , . . . , Xn ], where
V (f ) = {x An : f (x) = 0}.
Corollary 4.18 Every proper algebraic set in An is the intersection of a
finite number of algebraic hypersurfaces.
Proof The empty set in An can be represented as an intersection of two
hyperplanes (e.g., x1 = 0 and x1 = 1). Suppose therefore that the proper
algebraic set V is non-empty. It follows from Proposition 4.17 that there
exists a finite set {f1 , f2 , . . . , fk } polynomials belonging to K[X1 , X2 , . . . , Xn ]
such that V = V ({f1 , f2 , . . . , fk }). Moreover the polynomials f1 , f2 , . . . , fk
cannot all be zero, since V 6= An ; we can therefore assume (by removing
the zero polynomials from the list) that the polynomials f1 , f2 , . . . , fk are
non-zero. They must then all be non-constant, since V is non-empty. But
then
V = V (f1 ) V (f2 ) V (fk ),
as required.
Proposition 4.19 Let C be a collection of subsets of An that are open with
respect to the Zariski topology on An . Then there exists a finite collection
D1 , D2 , . . .S
, Dk of open sets belonging to C such that D1 D2 Dk is
the union DC D of all the open sets D belonging to C.
Proof It follows from the definition of the Zariski topology that, for each
open set D belonging to C, therePexists an ideal ID of K[X1 , X2 , . . . , Xn ] such
that D = An \ V (ID ). Let I = DC ID . Then
\
[
[
D =
(An \ V (ID )) = An \
V (ID )
DC
DC
DC
X

ID = An \ V (I)
= An \ V
DC

(see Proposition 4.16). Now the ideal I is finitely-generated (Corollary 4.10).


Moreover there exists a finite generating set {f1 , f2 , . . . , fk } for I with the
property that each generator fi belongs to one of the ideals ID , since if we
are given any finite generating set for I, then each of the generators can
be expressed as a finite sum of elements taken from the ideals ID , and the
collection of all these elements constitutes a finite generating set for I which
is of the required form. Choose D1 , D2 , . . . , Dk C such that fi IDi for
i = 1, 2, . . . , k. Then
I = ID1 + ID2 + + IDk ,
91

and thus
[

DC

D = A V (I) = A V

k
X

!
IDi

i=1

k
[

Di ,

i=1

as required.
We recall that a topological space is compact if and only if every open
cover of that space has a finite subcover. The following result therefore
follows directly from Proposition 4.19.
Corollary 4.20 Every subset of An is compact with respect to the Zariski
topology.

4.7

Maximal Ideals and Zorns Lemma

Definition Let R be a ring. A proper ideal I of R is said to be maximal if


the only ideals J of R satisfying I J R are J = I and J = R.
Lemma 4.21 A proper ideal I of a unital commutative ring R is maximal
if and only if the quotient ring R/I is a field.
Proof Let I be a proper ideal of the unital commutative ring R. Then the
quotient ring R/I is unital and commutative. Moreover there is a one-toone correspondence between ideals L of R/I and ideals J of R satisfying
I J R: if J is any ideal of R satisfying I J R, and if L is the
corresponding ideal of R/I then I + x L if and only if x J. We deduce
that I is a maximal ideal of R if and only if the only ideals of R/I are the
zero ideal {I} and R/I itself. It follows from Lemma 2.4 that I is a maximal
ideal of R if and only if R/I is a field.
We claim that every proper ideal of a ring R is contained in at least one
maximal ideal. In order to prove this result we shall make use of Zorns
Lemma concerning the existence of maximal elements of partially ordered
sets.
Definition Let S be a set. A partial order on S is a relation on S
satisfying the following conditions:
(i) x x for all x S (i.e., the relation is reflexive),
(ii) if x, y, z S satisfy x y and y z then x z (i.e., the relation is
transitive),
92

(iii) if x, y S satisfy x y and y x then x = y (i.e., the relation is


antisymmetric).
Neither of the conditions x y or y x need necessarily be satisfied by
arbitrary elements x and y of a partially ordered set S. A subset C of S is
said to be totally ordered if one or other of the conditions x y and y x
holds for each pair {x, y} of elements of C.
Example Let S be a collection of subsets of some given set. Then S is
partially ordered with respect to the relation (where A, B S satisfy
A B if and only if A is a subset of B).
Example The set N of natural numbers is partially ordered with respect to
the relation |, where n|m if and only if n divides m.
Let be the ordering relation on a partially ordered set S. An element u
of S is said to be an upper bound for a subset B of S if x u for all x B.
An element m of S is said to be maximal if the only element x of S satisfying
m x is m itself.
The following result is an important theorem in set theory.
Zorns Lemma. Let S be a non-empty partially ordered set.
Suppose that there exists an upper bound for each totally ordered
subset of S. Then S contains a maximal element.
We use Zorns lemma in order to prove the following existence theorem
for maximal ideals.
Theorem 4.22 Let R be a unital ring, and let I be a proper ideal of R.
Then there exists a maximal ideal M of R satisfying I M R.
Proof Let S be the set of all proper ideals J of R satisfying I J. The set S
is non-empty, since I S, and is partially ordered by the inclusion relation .
We claim that there exists an upper bound for any totally ordered subset C
of S.
Let L be the union of all the ideals belonging to some totally ordered
subset C of S. We claim that L is itself a proper ideal of R. Let a and b be
elements of L. Then there exist proper ideals J1 and J2 belonging to C such
that a J1 and b J2 . Moreover either J1 J2 or else J2 J1 , since the
subset C of S is totally ordered. It follows that a + b belongs either to J1 or
else to J2 , and thus a + b L. Similarly a L, ra L and ar L for
all r R. We conclude that L is an ideal of R. Moreover 1 6 L, since the
93

elements of C are proper ideals of R, and therefore 1 6 J for every J C. It


follows that L is a proper ideal of R satisfying I L. Thus L S, and L is
an upper bound for C.
The conditions of Zorns Lemma are satisfied by the partially ordered
set S. Therefore S contains a maximal element M . This maximal element
is the required maximal ideal of R containing the ideal I.
Corollary 4.23 Every unital ring has at least one maximal ideal.
Proof Apply Theorem 4.22 with I = {0}.

4.8

Prime Ideals

Definition Let R be a unital ring. A proper ideal I is said to be prime if,


given any ideals J and K satisfying JK I, either J I or K I.
The following result provides an alternative description of prime ideals of
a ring that is both unital and commutative.
Lemma 4.24 Let R be a unital commutative ring. An proper ideal I of R
is prime if and only if, given any elements x and y of R satisfying xy I,
either x I or y I.
Proof Let I be a proper ideal of R. Suppose that I has the property that,
given any elements x and y of R satisfying xy I, either x I or y I.
Let J and K be ideals of R neither of which is a subset of the ideal I. Then
there exist elements x J and y K which do not belong to I. But then xy
belongs to JK but does not belong to I. Thus the ideal JK is not a subset
of I. This shows that the ideal I is prime.
Conversely, suppose that I is a prime ideal of R. Let x and y be elements
of R satisfying xy I, and let J and K be the ideals generated by x and y
respectively. Then
J = {rx : r R},

K = {ry : r R},

since R is unital and commutative (see Lemma 2.5). It follows easily that
JK = {rxy : r R}. Now xy I. It follows that JK I. But I is prime.
Therefore either J I or K I, and thus either x I or y I.
Example Let n be a natural number. Then the ideal nZ of the ring Z of
integers is a prime ideal if and only if n is a prime number. For an integer j
belongs to the ideal nZ if and only if n divides j. Thus the ideal nZ is prime
94

if and only if, given any integers j and k such that n divides jk, either n
divides j or n divides k. But it follows easily from the Fundamental Theorem
of Arithmetic that a natural number n has this property if and only if n is
a prime number. (The Fundamental Theorem of Arithmetic states that any
natural number can be factorized uniquely as a product of prime numbers.)
Lemma 4.25 An ideal I of a unital commutative ring R is prime if and
only if the quotient ring R/I is an integral domain.
Proof If I is a proper ideal of the unital commutative ring R then the
quotient ring R/I is both unital and commutative. Moreover the zero element
of R/I is I itself (regarded as a coset of I in R). Thus R/I is an integral
domain if and only if, given elements x and y of R such that (I +x)(I +y) = I,
either I + x = I or I + y = I. But (I + x)(I + y) = I + xy for all x, y R,
and I + x = I if and only if x I. We conclude that R/I is an integral
domain if and only if I is prime, as required.
Lemma 4.26 Every maximal ideal of a unital commutative ring R is a prime
ideal.
Proof Let M be a maximal ideal of R. Then the quotient ring R/M is a
field (see Lemma 4.21). In particular R/M is an integral domain, and hence
M is a prime ideal.

4.9

Affine Varieties and Irreducibility

Definition A topological space Z is said to be reducible if it can be decomposed as a union F1 F2 of two proper closed subsets F1 and F2 . (A subset
of Z is proper if it is not the whole of Z.) A topological space Z is said to
be irreducible if it cannot be decomposed as a union of two proper closed
subsets.
Lemma 4.27 Let Z be a topological space. The following are equivalent:
(i) Z is irreducible,
(ii) the intersection of any two non-empty open sets in Z is non-empty,
(iii) every non-empty open subset of Z is dense.
Moreover a subset A of a topological space Z is irreducible (with respect to
the subspace topology) if and only if its closure A is irreducible.

95

Proof The topological space Z is irreducible if and only if the union of any
two proper closed subsets of Z is a proper subset of Z. Now the complement
of any proper closed set is a non-empty open set, and vica versa. Thus on
taking complements we see that Z is irreducible if and only if the intersection
of any two non-empty open subsets of Z is a non-empty subset of Z. This
shows the equivalence of (i) and (ii).
The equivalence of (ii) and (iii) follows from the fact that a subset of Z
is dense if and only if it has non-empty intersection with every non-empty
open set in Z.
Let A be a subset of Z. It follows directly from the definition of the
subspace topology on A that A is irreducible if and only if, given any closed
sets F1 and F2 such that A F1 F2 then either A F1 or A F2 . Now if
F is any closed subset of Z then A F if and only if A F . It follows that
A is irreducible if and only if A is irreducible.
It follows immediately from Lemma 4.27 that a non-empty irreducible
topological space is Hausdorff if and only if it consists of a single point.
Lemma 4.28 Any irreducible topological space is connected.
Proof A topological space Z is connected if and only if the only subsets of Z
that are both open and closed are the empty set and the whole set Z. Thus
suppose that the topological space Z were not connected. Then there would
exist a non-empty proper subset U of Z that was both open and closed. Let
V = Z \ U . Then U and V would be disjoint non-empty open sets. It would
then follow from Lemma 4.27 that Z could not be irreducible.
Lemma 4.29 Let V be an algebraic set, and let V1 be a proper algebraic
subset of V . Then there exists f K[X1 , X2 , . . . , Xn ] such that f (x) = 0 for
all x V1 but f 6 I(V ).
Proof The inclusion V1 V implies that I(V ) I(V1 ). Now V = V (I(V ))
and V1 = V (I(V1 )). Thus if V1 is a proper subset of V then I(V ) 6= I(V1 ),
and hence there exists f I(V1 ) such that f 6 I(V ). Then f is the required
polynomial.
Proposition 4.30 A non-empty algebraic set V in An is irreducible (with
respect to the Zariski topology) if and only if the ideal I(V ) is a prime ideal
of K[X1 , X2 , . . . , Xn ].

96

Proof Suppose that the algebraic set V is irreducible. Let f and g be


polynomials in K[X1 , X2 , . . . , Xn ] with the property that f g I(V ). Then
V V (f )V (g), since, given any point of V , one or other of the polynomials
f and g must be zero at that point. Let V1 = V V (f ) and V2 = V V (g).
Then V1 and V2 are algebraic subsets of V , and V = V1 V2 . Therefore either
V = V1 or V = V2 , since the irreducible algebraic set V cannot be expressed
as a union of two proper algebraic subsets. It follows that either f I(V )
or else g I(V ). Thus I(V ) is prime, by Lemma 4.24.
Conversely, suppose that V is reducible. Then there exist proper algebraic
subsets V1 and V2 of V such that V = V1 V2 . It then follows from Lemma 4.29
that there exist polynomials f and g in K[X1 , X2 , . . . , Xn ] such that f (x) = 0
for all x V1 , g(x) = 0 for all x V2 , and neither f nor g belongs to I(V ).
But then f (x)g(x) = 0 for all x V , since V = V1 V2 , and hence f g I(V ).
Thus the ideal I(V ) is not prime.
Definition An affine algebraic variety is an irreducible algebraic set in An .
Theorem 4.31 Every algebraic set in An can be expressed as a finite union
of affine algebraic varieties.
Proof Let C be the collection of all ideals I of K[X1 , X2 , . . . , Xn ] with the
property that the corresponding algebraic set V (I) cannot be expressed as a
finite union of affine varieties. We claim that C cannot contain any maximal
element.
Let I be an ideal of K[X1 , X2 , . . . , Xn ] belonging to C. Then the algebraic
set V (I) cannot itself be an affine variety, and therefore there must exist
proper algebraic subsets V1 and V2 of V such that V (I) = V1 V2 . Let
I1 = I(V1 ) and I2 = I(V2 ). Then I(V (I)) I1 and I(V (I)) I2 , since
V1 V (I) and V2 V (I). Also I I(V (I)). It follows that I I1 and
I I2 . Moreover V (I1 ) = V1 and V (I2 ) = V2 , since V1 and V2 are algebraic
sets (see Lemma 4.15), and thus V (I1 ) 6= V (I) and V (I2 ) 6= V (I). It follows
that I 6= I1 and I 6= I2 . Thus I is a proper subset of both I1 and I2 .
Now V1 and V2 cannot both be finite unions of affine varieties, since V (I)
is not a finite union of affine varieties. Thus one or other of the ideals I1 and I2
must belong to the collection C. It follows that no ideal I belonging to C can
be maximal in C. But every non-empty collection of ideals of the Noetherian
ring K[X1 , X2 , . . . , Xn ] must have a maximal element (see Proposition 4.5).
Therefore C must be empty, and thus every algebraic set in An is a finite
union of affine varieties, as required.
We shall show that every algebraic set in An has an essentially unique
representation as a finite union of affine varieties.
97

Lemma 4.32 Let V1 , V2 , . . . , Vk be algebraic sets in An , and let W be an


affine variety satisfying W V1 V2 Vk . Then W Vi for some i.
Proof The affine variety W is the union of the algebraic sets W Vi for
i = 1, 2, . . . , k. It follows from the irreducibility of W that the algebraic sets
W Vi cannot all be proper subsets of W . Hence W = W Vi for some i,
and hence W Vi , as required.
Proposition 4.33 Let V be an algebraic set in An , and let V = V1 V2
Vk , where V1 , V2 , . . . , Vk are affine varieties, and Vi 6 Vj for any j 6= i.
Then V1 , V2 , . . . , Vk are uniquely determined by V .
Proof Suppose that V = W1 W2 Wm , where W1 , W2 , . . . , Wm are affine
varieties, and Wi 6 Wj for any j 6= i. Now it follows from Lemma 4.32 that,
for each integer i between 1 and k, there exists some integer (i) between 1
and m such that Vi W(i) . Similarly, for each integer j between 1 and m,
there exists some integer (j) between 1 and k such that Wj V (j) . Now
Vi W(i) V ((i)) , But Vi 6 Vi0 for any i0 6= i. It follows that i = ((i))
and Vi = W(i) . Similarly Wj V (j) W( (j)) , and thus j = ( (j)) and
Wj = V (j) . We deduce that
: {1, 2, . . . , k} {1, 2, . . . , m}
is a bijection with inverse , and thus k = m. Moreover Vi = W(i) , and thus
the varieties V1 , V2 , . . . , Vk are uniquely determined by V , as required.
Let V be an algebraic set, and let V = V1 V2 Vk , where V1 , V2 , . . . , Vk
are affine varieties, and Vi 6 Vj for any j 6= i. The varieties V1 , V2 , . . . , Vk are
referred to as the irreducible components of V .

4.10

Radical Ideals

Definition Let R be a unital commutative ring. An ideal I of R is said to


be a radical ideal if every element x of R with the property that xm I for
some natural number m belongs to I.
Lemma 4.34 Every prime ideal of a unital commutative ring R is a radical
ideal.
Proof Let I be a prime ideal. Suppose that x R satisfies xm I. If m = 1
then we are done. If not, then either x I or xm1 I, since I is prime.
Thus it follows by induction on m that x I. Thus I is a radical ideal.
98


Lemma 4.35 Let I be an ideal of a unital commutative ring R, and let I
m
denote the set of all elements
x of R with the property that x Ifor some
natural number m. Then I is a radical ideal of R. Moreover I = I if and
only if I is a radical ideal of R.

Proof Let x and y be elements of I. Then there exist natural numbers m


and n such that xm I and y n I. Now
m+n

(x + y)

m+n
X
i=0

m+n
i

xi y m+ni ,

(where x0 = 1 = y 0 ), and moreover, given any value of i between 0 and


m + n, either i m or m + n i n, so that
either xi
I or y m+ni I.

Therefore (x + y)m+n I, and thus x + y I. Also


x I and rx I
for all
r

R.
Thus
I
is
an
ideal
of
R.
Clearly
I is a radical ideal, and

I = I if and only if I is a radical ideal.

The ideal I is referred to as the radical of the ideal I.


Lemma 4.36 Let Z be a subset of An . Then I(Z) is a radical ideal of the
polynomial ring K[X1 , X2 , . . . , Xn ]. Moreover Z = V (I(Z)) if and only if Z
is an algebraic set in An .
Proof Note that if g and h are polynomials belonging to K[X1 , X2 , . . . , Xn ]
which are zero throughout the set Z then the same is true of the polynomials
g + h, g and f g for all f K[X1 , X2 , . . . , Xn ]. Therefore I is an ideal of
K[X1 , X2 , . . . , Xn ]. Moreover g m is identically zero on Z if and only if the
same is true of g. Therefore the ideal I(Z) is a radical ideal. If Z = V (I(Z))
then Z is clearly an algebraic set. Conversely, if Z is an algebraic set then
Z = V (S) for some subset S of K[X1 , X2 , . . . , Xn ], and therefore
V (I(Z)) = V (I(V (S))) = V (S) = Z,
by Lemma 4.15, as required.
Lemma 4.37 Let S be a subset of the polynomial ring K[X1 , X2 , . . . , Xn ],
and let I be the ideal generated by S. Then V (S) = V (I) = V ( I), where

I is the radical of the ideal I. Thus every algebraic set in An is of the form
V (I) for some radical ideal I of K[X1 , X2 , . . . , Xn ].

99

Proof The ideal I(V (S)) of K[X1 , X2 , . . . , Xn ] contains the set S. Therefore

I I(V (S)), where I is the ideal generated by S. Moreover if f I then


f m I for some natural number m, and thus f m I(V (S)). But I(V (S))
is a radical ideal (see Lemma 4.36). Therefore f I(V (S)). Thus

S I I I(V (S)).
It follows that

V (I(V (S))) V ( I) V (I) V (S).


But V (I(V (S))) = V (S) (see Lemma 4.15). Therefore V (S) = V (I) =
V ( I), as required.

4.11

Commutative Algebras of Finite Type

Definition Let K be a field. A unital ring R is said to be a K-algebra


if K R, the multiplicative identity elements of K and R coincide, and
ab = ba for all a K and b R.
It follows from this definition that a unital commutative ring R is a Kalgebra if K R and K and R have the same multiplicative identity element.
Note that if L: K is a field extension, then the field L is a unital K-algebra.
Definition Let K be a field, and let R1 and R2 be K-algebras. A ring
homomorphism : R1 R2 is said to be a K-homomorphism if (k) = k for
all k K.
Given any subset A of a unital commutative K-algebra R, we denote by
K[A] the subring of R generated by K A (i.e., the smallest subring of R
containing K A). In particular, if a1 , a2 , . . . , ak are elements of R then we
denote by K[a1 , a2 , . . . , ak ] the subring of R generated by K {a1 , a2 , . . . , ak }.
If R = K[A] then we say that the set A generates the K-algebra R.
Note that any element of K[a1 , a2 , . . . , ak ] is of the form f (a1 , a2 , . . . , ak )
for some polynomial f in k independent indeterminates with coefficients in
K. Indeed the set of elements of R that are of this form is a subring of R,
and is clearly the smallest subring of R containing K {a1 , a2 , . . . , ak }.
Definition Let K be a field. A unital commutative ring R is said to be a Kalgebra of finite type if K R, the identity elements of K and R coincide, and
there exists a finite subset a1 , a2 , . . . , ak of R such that R = K[a1 , a2 , . . . , ak ].

100

Lemma 4.38 Let K be a field. Then every K-algebra of finite type is a


Noetherian ring.
Proof Let R be a K-algebra of finite type. Then there exist a1 , a2 , . . . , ak
R such that R = K[a1 , a2 , . . . , ak ]. Now it follows from the Hilbert Basis
Theorem that the ring K[X1 , X2 , . . . , Xk ] of polynomials in the independent
indeterminates X1 , X2 , . . . , Xk with coefficients in K is a Noetherian ring (see
Corollary 4.10). Moreover R
= K[X1 , X2 , . . . , Xk ]/a, where a is the kernel
of the homomorphism
: K[X1 , X2 , . . . , Xk ] R
that sends f K[X1 , X2 , . . . , Xk ] to f (a1 , a2 , . . . , ak ). (Note that the homomorphism is surjective; indeed the image of this homomorphism is a
subring of R containing K and ai for i = 1, 2, . . . , k, and is therefore the
whole of R.) Thus R is isomorphic to the quotient of a Noetherian ring, and
is therefore itself Noetherian (see Lemma 4.7).
If K(): K is a simple algebraic extension then K() is a K-algebra of
finite type. Indeed K() is a finite-dimensional vector space over K (see
Theorem 3.4). If a1 , a2 , . . . , ak span K() as a vector space over K then
clearly K() = K[a1 , a2 , . . . , ak ].

4.12

Zariskis Theorem

Proposition 4.39 Let K and L be fields, with K L. Suppose that L: K


is a simple field extension and that L is a K-algebra of finite type. Then the
extension L: K is finite.
Proof The field L is a K-algebra of finite type, and therefore there exist
elements 1 , 2 , . . . , m of L such that L = K[1 , 2 , . . . , m ]. Also the field
extension L: K is simple, and therefore L = K() for some element of
K. Now, given any element of L there exist polynomials f and g in K(x)
such that g() 6= 0 and = f ()g()1 . Indeed one may readily verify
that the set of elements of L that may be expressed in the form f ()g()1
for some polynomials f, g K[X] with g() 6= 0 is a subfield of L which
contains K {}. It is therefore the whole of L, since L = K(). It follows
that there exist polynomials fi and gi in K[X] such that gi () 6= 0 and
i = fi ()gi ()1 for i = 1, 2, . . . , m. Let e(x) = g1 (x)g2 (x) . . . , gm (x). We
shall show that if the element of L were not algebraic over K then every
irreducible polynomial with coefficients in K would divide e(x),
101

Let p K[X] be an irreducible polynomial with coefficients in K, where


p() 6= 0. Now L = K[1 , 2 , . . . , m ], and therefore every element of L is
expressible as a polynomial in 1 , 2 , . . . , m with coefficients in K. Thus
there exists some polynomial Hp in m indeterminates, with coefficents in K,
such that
p()1 = Hp (1 , 2 , . . . , m ).
Let d be the total degree of H. One can readily verify that
e()d Hp (1 , 2 , . . . , m ) = q(),
for some polynomial q(x) with coefficients in K. But then p()q() = e()d ,
and therefore is a zero of the polynomial pq ed . If it were the case that
were not algebraic over K then this polynomial pq ed would be the zero
polynomial, and thus p(x)q(x) = e(x)d . But it follows from Proposition 2.14
that an irreducible polynomial divides a product of polynomials if and only
if it divides at least one of the factors. Therefore the irreducible polynomial
p would be an irreducible factor of the polynomial e, and so would be an irreducible factor of one of the polynomials g1 , g2 , . . . , gm . We see therefore that
if were not algebraic over K then the polynomial e would be divisible by
every irreducible polynomial in K[X]. But this is impossible, because a given
polynomial in K[X] can have only finitely many irreducible factors, whereas
K[X] contains infinitely many irreducible polynomials (Lemma 2.13). We
conclude therefore that must be algebraic over K. But any simple algebraic field extension is finite (Theorem 3.4). Therefore L: K is finite, as
required.
Lemma 4.40 Suppose that K A B, where A and B are unital commutative rings, and B is both a K-algebra of finite type and a finitely generated
A-module. Then A is also a K-algebra of finite type.
Proof There exist 1 , 2 , . . . , m B such that B = K[1 , 2 , . . . , m ],
since B is a K-algebra of finite type. Also there exist 1 , 2 , . . . , n B such
that
B = A1 + A2 + + An ,
since B is a finitely generated A-module. Moreover we can
P choose 1 = 1.
But then there exist elements qi of A such that q = ni=1 P
qi i for q =
1, 2, . . . , n. Also there exist elements ijk of A such that i j = nk=1 ijk k
for i, j = 1, 2 . . . , n. Let
S = {qi : 1 q m, 1 i n} {ijk : 1 i, j, k n},

102

let A0 = K[S], and let


B0 = A0 1 + A0 2 + + A0 n .
Now each product i j is a linear combination of 1 , 2 , . . . , n with coefficients ijk in A0 , and therefore i j B0 for all i and j. It follows from
this that the product of any two elements of B0 must itself belong to B0 .
Therefore B0 is a subring of B. Now K B0 , since K A0 and 1 = 1.
Also q B0 for q = 1, 2, . . . , m. But B = K(1 , 2 , m ). It follows that
B0 = B, and therefore B is a finitely-generated A0 -module.
Now any K-algebra of finite type is a Noetherian ring (Lemma 4.38). It
follows that A0 is a Noetherian ring, and therefore any finitely-generated
module over A0 is Noetherian (see Corollary 4.6). In particular B is a
Noetherian A0 -module, and therefore every submodule of B is a finitelygenerated A0 -module. In particular, A is a finitely-generated A0 -module.
Let 1 , 2 , . . . , p be a finite collection of elements of A that generate A as an
A0 -module. Then any element a of A can be written in the form
a = a1 1 + a2 2 + + ap p ,
where al A0 for l = 1, 2, . . . , p. But each element of A0 can be expressed as
a polynomial in the elements qi and ijk with coefficients in K. It follows
that each element of A can be expressed as a polynomial in the elements qi ,
ijk and l (with coefficients in K), and thus A = K[T ], where
T = S {l : 1 l p}.
Thus A is a K-algebra of finite type, as required.
Theorem 4.41 (Zariski) Let L: K be a field extension. Suppose that the
field L is a K-algebra of finite type. Then L: K is a finite extension of K.
Proof We prove the result by induction on the number of elements required
to generate L as a K-algebra. Thus suppose that L = K[1 , 2 , . . . , n ], and
that the result is true for all field extensions L1 : K1 with the property that L1
is generated as a K1 -algebra by fewer than n elements (i.e., there exist elements 1 , 2 , . . . , m of L1 , where m < n, such that L1 = K1 [1 , 2 , . . . , m ]).
Let K1 = K(1 ). Then L = K1 [2 , 3 , , n ]. It follows from the induction hypothesis that L: K1 is a finite field extension (and thus L is a
finitely-generated K1 -module). It then follows from Lemma 4.40 that K1 is
a K-algebra of finite type.
But the extension K1 : K is a simple extension. It therefore follows from
Proposition 4.39 that the extension K1 : K is finite. Thus both L: K1 and
K1 : K are finite extensions. It follows from the Tower Law (Proposition 3.1)
that L: K is a finite extension, as required.
103

4.13

Hilberts Nullstellensatz

Proposition 4.42 Let K be an algebraically closed field, let R be a commutative K-algebra of finite type, and let m be a maximal ideal of R. Then
there exists a surjective K-homomorphism : R K from R to K such that
m = ker .
Proof Let L = R/m, and let : R L denote the quotient homomorphism.
Then L is a field (Lemma 4.21). Now m = ker and 1 6 m, and therefore
|K 6= 0. It follows that m K is a proper ideal of the field K. But
the only proper ideal of a field is the zero ideal (Lemma 2.4). Therefore
m K = {0}. It follows that the restriction of to K is injective and maps
K isomorphically onto a subfield of L. Let K1 = (K), and let : K K1
be the isomorphism obtained on restricting : R L to K. Then L: K1
is a field extension, and L is a K1 -algebra of finite type. It follows from
Zariskis Theorem (Theorem 4.41) that L: K1 is a finite field extension. But
then L = K1 , since the field K1 is algebraically closed (Lemma 3.7). Let
= 1 . Then : R K is the required K-homomorphism from R to K.
Theorem 4.43 Let K be an algebraically closed field, and let R be a commutative K-algebra of finite type. Let a be a proper ideal of R. Then there
exists a K-homomorphism : R K from R to K such that a ker .
Proof Every proper ideal of R is contained in some maximal ideal (Theorem 4.22). Let m be a maximal ideal of R with a m. It follows from
Proposition 4.42 that m = ker for some K-homomorphism : R K.
Then a ker , as required.
Theorem 4.44 (Weak Nullstellensatz) Let K be an algebraically closed field,
and let a be a proper ideal of the polynomial ring K[X1 , X2 , . . . , Xn ], where
X1 , X2 , . . . , Xn are independent indeterminates. Then there exists some point
(a1 , a2 , . . . , an ) of An (K) such that f (a1 , a2 , . . . , an ) = 0 for all f a.
Proof Let R = K[X1 , X2 , . . . , Xn ]. Then R is a K-algebra of finite type. It
follows from Theorem 4.43 that there exists a K-homomorphism : R K
such that a ker . Let ai = (Xi ) for i = 1, 2, . . . , n. Then (f ) =
f (a1 , a2 , . . . , an ) for all f R. It follows that f (a1 , a2 , . . . , an ) = 0 for all
f a, as required.
Theorem 4.45 (Strong Nullstellensatz) Let K be an algebraically closed
field, let a be an ideal of the polynomial ring K[X1 , X2 , . . . , Xn ], and let f

104

K[X1 , X2 , . . . , Xn ] be a polynomial with the property that f (x1 , x2 , . . . , xn ) =


0 for all (x1 , x2 , . . . , xn ) V (a), where
V (a) = {(x1 , x2 , . . . , xn ) An (K) : g(x1 , x2 , . . . , xn ) = 0 for all g a}.
Then f r a for some natural number r.
Proof Let R = K[X1 , X2 , . . . , Xn ], and let S denote the ring R[Y ] of polynomials in a single indeterminate Y with coefficients in the ring R. Then S can
be viewed as the ring K[X1 , X2 , . . . , Xn , Y ] of polynomials in the n + 1 indeterminate indeterminates X1 , X2 , . . . , Xn , Y with coefficients in the field K.
The ideal a of R determines a corresponding ideal b of S consisting of those
elements of S that are of the form
g0 + g1 Y + g2 Y 2 + + gr Y r
with g0 , g1 , . . . , gr a. (Thus the ideal b consists of those elements of the
ring S that can be considered as polynomials in the indeterminate Y with
coefficients in the ideal a of R.)
Let f R be a polynomial in the indeterminates X1 , X2 , . . . , Xn with the
property that f (x1 , x2 , . . . , xn ) = 0 for all (x1 , x2 , . . . , xn ) V (a), and let c
be the ideal of S defined by
c = b + (1 f Y ).
(Here (1 f Y ) denotes the ideal of the polynomial ring S generated by the
polynomial 1 f (X1 , X2 , . . . , Xn )Y .) Let V (c) be the subset of (n + 1)dimensional affine space An+1 (K) consisting of all points (x1 , x2 , . . . , xn , y)
An+1 (K) with the property that h(x1 , x2 , . . . , xn , y) = 0 for all h c. We
claim that V (c) = .
Let (x1 , x2 , . . . , xn , y) be a point of V (b). Then g(x1 , x2 , . . . , xn ) = 0 for
all g a, and therefore (x1 , x2 , . . . , xn ) V (a). But the polynomial f has
the value zero at each point of V (a). It follows that the polynomial 1 f Y
has the value 1 at each point of V (b), and therefore
V (c) = V (b) V (1 f Y ) = .
It now follows immediately from the Weak Nullstellensatz (Theorem 4.44)
that c cannot be a proper ideal of S, and therefore 1 c. Thus there exists
a polynomial h belonging to the ideal b of S such that h 1 (1 f Y ).
Moreover this polynomial h is of the form
h(X1 , X2 , . . . , Xn , Y ) =

r
X
j=0

105

gj (X1 , X2 , . . . , Xn )Y j ,

where g1 , g2 , . . . , gn a.
Let g a be defined by g =

r
P

gj f rj . Now g f r = g f r h + f r (h 1).

j=0

Also
r

gf h=

r
X

gj f rj (1 f j Y j ) (1 f Y ),

j=0

since the polynomial 1 f j Y j is divisible by the polynomial 1 f Y for all


positive integers j. It follows that gf r (1f Y ). But the polynomial gf r
is a polynomial in the indeterminates X1 , X2 , . . . , Xn , and, if non-zero, would
be of degree zero when considered as a polynomial in the indeterminate Y
with coefficients in the ring R. Also any non-zero element of the ideal (1
f Y ) of S is divisible by the polynomial 1 f Y , and is therefore of strictly
positive degree when considered as a polynomial in the indeterminate Y
with coefficients in R. We conclude, therefore that g f r = 0. But g a.
Therefore f r a, as required.

106

Вам также может понравиться